MSK

Pataasin ang iyong marka sa homework at exams ngayon gamit ang Quizwiz!

Case A 17-year-old boy presents with a 5-day history of intermittent fever, joint pain, and redness and swelling of the joints. The patient gives a history of pain in the right knee and ankle. 3 days ago, he had pain and swelling in his left knee, but now it has improved. On examination, temperature is 102° F, pulse is 108/min, RR is 20/min, and BP is 110/80 mm Hg. The patient's right knee is swollen, tender, and warm. There is a limitation of range of motion due to pain. The right ankle appears swollen and warm. Other system exams are normal. Lab tests are ordered; during the follow-up exam, you note elevated erythrocyte sedimentation rate (ESR) and rising ASO titers. Question How long should the patient receive secondary prophylactic antibiotics?

Correct answer: 5 years Explanation The correct response is 5 years. Rheumatic fever is an inflammatory condition that is precipitated by group A streptococcal infection. It is more common in children, but it also occurs in adults. The clinical manifestations include arthritis, carditis, fever, subcutaneous nodules, erythema marginatum, and chorea. It is more commonly diagnosed in close living quarters, such as a dormitory. The diagnosis is established by finding antecedent Streptococcal infection (+ culture OR rising ASO or anti-DNAase titers), plus fulfillment of the modified Jones criteria. Modified Jones criteria Must have 2 major findings, or 1 major and 2 minor findings:

Case A 33-year-old Caucasian man with a history of severe type I Diabetes presents with a red and swollen pinky toe on his left foot. Further details from the patient include that he was running to answer his phone at home and stepped on an exposed carpet tack. This occurred about 2 days ago. He washed it out and covered it as soon as he could. Since then he has noted a significant increase in redness and severe restriction of movement of this toe; these characteristics have worsened tremendously in the last 12 hours. In addition, he admits to "feeling feverish" for the past day and experienced significant night sweats the previous night. He denies formally measuring his temperature. He has tried to take acetaminophen 500 mg 2 tablets every 8 hours without any noticeable relief. Pertinent physical examination findings include an oral temperature of 102.8° F, significant erythema and edema extending the entire 5th toe of the left foot that is accompanied with restricted range of motion due to tenderness and swelling. There is a small amount of purulent drainage at the site of the wound. Question Considering the most likely diagnosis based on the history and physical examination, what is the total length of time this patient will need to be on antibiotics?

Correct answer: 6 weeks Explanation The patient will need to be on antibiotics for at least 6 weeks. It is highly likely that the patient above has an acute case of osteomyelitis. In simplest terms this is an infection of the bone involved. Various etiologies could lead to this condition that may include direct trauma, an infection from somewhere else in the body, a chronic ulcer or wound, or even bacteremia. It has been noted that specific patient populations are more prone to develop osteomyelitis: these include diabetics, hemodialysis patients, sickle cell disease patients, IV drug abusers, or even elderly patients. Symptoms that should make a provider be suspicious of osteomyelitis includes pain and /or tenderness of the affected area/bone, swelling and warmth of the area, significant fever, general discomfort, drainage of purulent material from the site, as well has sweating and chills. Management of any case of osteomyelitis is to most simply eliminate the infection and prevent development any type of chronic infection; chronic cases of osteomyelitis can have severely detrimental effects on patients including but not limited to: permanent deformity, possible fracture, and long term chronic issues. The patient above being a brittle diabetic will also further complicate the treatment plan for this case of osteomyelitis. Actions commonly taken in treatment include drainage, antibiotic treatment, splinting or cast immobilization, or even surgery. Prescribing antibiotics is the first step in treating osteomyelitis. What agent used and for how long is directed by the type of organism involved as well as the patient's current health state. Cultures are obtained either through aspiration or even a bone biopsy to identify the organism involved. Antibiotic duration is not clear cut in terms of treatment of osteomyelitis. However, the recommendation for any diabetic patients with a diagnosis such as this requires a duration of at least a 6 week time period.

Question A 73-year-old man presents with an inability to actively raise his left arm. This started 1 month ago, prior to which his shoulder pain had improved. He has had shoulder pain for more than 6 months and it has kept him from sleeping on his left side and has often woken him up. There was no specific injury that he can recall, although he felt a pop in the shoulder a month ago while taking out the trash. On a physical exam, what findings would you expect to observe?

Correct answer: A positive (+) drop arm test Explanation This is a torn rotator cuff and drop arm test is positive in this condition. Typically history includes pain for several months, with difficulty sleeping on the affected side and waking from the pain. Active range of motion is limited, but passive range of motion is usually full (unless associated with a frozen shoulder prior). There is usually tenderness over the greater tuberosity. Nerve damage is rare with cuff tears. Reference: Andrews JR. Rotator Cuff Tear. In: Griffin LY, ed. Essentials of Musculoskeletal Care, 3rd ed. Rosemont, IL: American Academy of Orthopaedic Surgeons; 2005:205.

Correct answer: Uveitis Explanation The correct response is uveitis. The patient is presenting with signs and symptoms indicative of ankylosing spondylitis (AS). Characteristic symptoms include back pain that is worse in the morning and improves with exercise. Back stiffness and extreme fatigue are also reported. AS is more common in male patients. As with other inflammatory arthritides, patients usually present at a younger age (18-35 years) relative to patients with mechanical causes of back pain. Unlike mechanical back pain, the pain associated with inflammatory spondylitis usually improves with movement. AS is a clinical diagnosis, but many patients will have elevated c-reactive protein and erythrocyte sedimentation rates. Diagnostic imaging classically reveals inflammatory and sclerotic changes affecting the sacroiliac joints and spine. Vertebral fusion is also seen as a result of progressive joint erosion. Extra-articular conditions associated with AS include iritis and uveitis (inflammations of the iris and uvea, respectively) as well as inflammatory bowel disease. Heart disease is also more common in patients with inflammatory arthritides. Diabetes mellitus and peptic ulcer disease are relatively more common in patients with rheumatoid arthritis. Psoriatic skin lesions are seen in patients with psoriatic arthritis. Scleroderma is an autoimmune disease that does not coexist with increased frequency in any of the inflammatory arthritides.

Correct answer: Acetaminophen Explanation Acetaminophen is correct. The patient has symptoms of knee pain and x-ray findings of joint narrowing with osteophyte formation. These findings are consistent with osteoarthritis. Acetaminophen is the recommended first-line treatment for osteoarthritis. Studies have shown that their efficacy is equal to NSAIDs. Naproxen is incorrect. Naproxen is an NSAID. While NSAIDs are used in the treatment of osteoarthritis, this patient has a history of Crohn's disease. Patients with Crohn's disease should avoid NSAIDs, as they may exacerbate the disease. Oxycodone is incorrect. Narcotic medications should be avoided in chronic conditions such as osteoarthritis. Methotrexate is incorrect. Methotrexate may be used in the treatment of rheumatoid arthritis, but not osteoarthritis. Prednisone is incorrect. While prednisone could alleviate the symptoms of osteoarthritis, it has a side effect profile and should be avoided.

Case You are evaluating a 34-year-old man who was brought to the emergency department after being the driver in a single car motor vehicle accident. Preliminary radiology studies show a comminuted fracture of the right tibia, but right now the patient describes a dramatically increasing amount of pain felt in that extremity. He describes the pain as being a 10/10, becoming worse with each passing minute, and as a deep, achy, burning pain. You quickly examine the right leg and note pallor, a tense "wood-like" feeling of the extremity, diminished sensation, and muscle weakness. Question These findings are consistent with what diagnosis?

Correct answer: Acute compartment syndrome Explanation This patient is suffering from compartment syndrome. Compartment syndrome should always be considered an urgent and extremely emergent situation due to the consequences that can ensue. Acute compartment syndrome (ACS) most often develops after the patient has suffered from some type of significant trauma, especially involving a long bone fracture. In fact, fractures account for more than 75% of cases of ACS. More often ACS is seen in patients under the age of 35 years of age and also has a higher incidence in young men, especially after they experience fracture of the tibial diaphysis or distal radius; risks of developing ACS are increased if the patient suffers from a comminuted fracture. The patient described above fits these scenarios that show a higher risk of developing ACS. Often, the signs and symptoms of ACS occur in a stepwise fashion; an important clue is a rapid progression of symptoms and signs in a short amount of time. Symptoms include pain out of proportion to the apparent injury, persistent deep ache or burning pain, or even paresthesias. Signs that would be evident include pain with passive stretch of muscles in the affected compartment, a tense compartment with a firm "wood-like" feeling, pallor, diminished sensation, muscle weakness, and a late finding of paralysis. Patients with a dislocation will commonly have a visibly deformed joint, as well as evidence of swelling, discoloration, and intense pain or immovability. This would most likely had been found on the initial survey of the patient as they entered the emergency department. Another possibility is peripheral vascular ischemia (PVI), although this is not a likely diagnosis. Many times patients with PVI have significant peripheral vascular history, as well as having a substantial smoking history. These patients' symptoms occur suddenly and are not associated with any significant event or trauma; symptoms and signs are commonly referred to suffering from the 4P's: pain, pallor, pulseless, and paresthesias. The patient above is not as likely to be suffering from this pathology. Deep venous thrombosis (DVT) is suspected if a patient has evidence of extremity swelling, pain, or erythema of the involved extremity, along with significant existing risk factors that increase the likelihood of a DVT: history of prolonged bed rest/hospitalization/immobility, malignancy, pregnancy, or even lower extremity trauma. Even though the patient above has just recently suffered trauma to his lower extremity, a DVT is not likely to be causing the issues he is currently experiencing. Cellulitis would be likely if the examiner observed such signs/symptoms as edema, erythema, and evidence of a warm plaque that may have vesicles or bullae. References

Case A 9-year-old girl presented to her pediatrician several months ago with arthralgias, myalgias, weight loss, and morning stiffness of her joints. She has since been diagnosed with Juvenile Rheumatoid Arthritis (JRA) and has been taking nonsteroidal anti-inflammatory (NSAIDs) medications to treat the symptoms. Her parents have recently taken her to see a rheumatologist in the hopes of making sure they are considering every form of therapy for the child despite positive results from the NSAIDs thus far. Question What first-line adjunctive treatment should be recommended?

Correct answer: Addition of physical and occupational therapy Explanation The correct answer is the addition of physical and occupational therapy. The prognosis of JRA is typically good, but it needs to be treated early and aggressively in order to prevent osteopenia, asymmetrical skeletal development, systemic manifestations, and chronic eye disease. Aggressive therapy refers to pharmacologic therapy as well as lifestyle and dietary changes. The goal of physical therapy (PT) is to strengthen muscles, decrease the impact loading on joints, and increase range of motion. Patients are given instructions for using moist heat on the joints and are taught both passive and active assisted exercises to perform 2-3 times daily. Occupational therapy (OT) is used to improve the patient's body mechanics, posture, and other modalities to protect their joints. Patients are given instructions by an occupational therapist about proper posture, improvement in activities of daily living, body mechanics, and how to protect their joints in general. It is important for patients with JRA to use exercise during all stages of their disease. During the early stages, patients should use passive range of motion in order to maintain range of motion during the most painful period. As pain is improved and inflammation decreases, patients should slowly increase to active forms of exercise. Salicylates can be used as first-line treatment for patients with JRA, but would not be used in conjunction with an NSAID. NSAIDs have become preferred over salicylates in recent years due to the decreased frequency of dosing, their liquid form, and diminished side effects. While NSAIDs are commonly used as a first-line agent and the patient has shown improvement, JRA is a disease that has historically shown better results with early and aggressive treatment. Aggressive treatment refers to the use of a combination of various forms of treatment, such as pharmacologic treatments, in conjunction with lifestyle modifications. Disease-modifying antirheumatic agents (DMARDs) are considered a second-line form of treatment for patients with JRA unless the patient has contraindications to NSAIDs and salicylates. DMARDs are not given in conjunction with an NSAID but may be used as combination therapy; most commonly, 2 DMARDs are used together. Combination therapy is the first-line treatment in patients with erosions on radiograph or other signs of more aggressive disease at the time of diagnosis. Folic acid is given to children with JRA when they are started on MTX, a purine inhibitor, during combination therapy with another DMARD for patients with severe disease. MTX is an effective DMARD and toxicity is decreased when used in conjunction with folic acid.

Case A 58-year-old woman with diabetes presents with a 1-month history of left shoulder pain. She rates the pain an 8/10 and describes it as a constant ache. The pain is worse at night, and it is gradually becoming more severe. Her ROM is becoming compromised. It is becoming more difficult for her to put on her coat and take care of her yard. She denies any redness, bruising, or swelling. She cannot recall any direct injury to her shoulder, and she denies any previous trauma or surgery. On physical exam, both active and passive ROM are compromised. Question What is the most likely diagnosis?

Correct answer: Adhesive capsulitis Explanation The most likely diagnosis is adhesive capsulitis. It is often associated with diabetes, ischemic heart disease, and thyroid disorders. It can also result from immobility following a traumatic shoulder injury. Patients complain of a slow, gradual onset of shoulder pain that can be severe, even at night. The pain is generalized and referred to the superolateral aspect of the shoulder and upper arm. Examination reveals no swelling at the glenohumeral joint. Passive and active ROM are greatly limited in all movements. Acromioclavicular sprain is common in younger people and athletes. The classic cause is the direct blow to the acromion with the humerus in the adducted position. This drives the acromion medially and inferiorly. Falling on an outstretched hand or elbow can also cause an acromioclavicular sprain. Fracture of the clavicle is one of the most common injuries. It is usually caused by a fall on the lateral aspect of the shoulder. Less commonly, it is caused by a direct blow or falling on the outstretched hand. Although complications are uncommon, they can occur. With this type of injury lung and neurovascular compromise can occur. AP X-ray is the best way to ascertain a clavicle fracture. Glenohumeral dislocations in younger people are most commonly caused by direct trauma and sports injuries. Patients with this type of injury are usually unwilling to move the affected arm, and they tend to cradle it with the unaffected arm. About 80-90% of shoulder dislocations are anterior shoulder dislocations. An excessive external rotation or abduction force usually causes this type of injury. A bulge may be noticeable where the humeral head rests, with an emptiness beneath the acromion. Posterior shoulder dislocation usually happens when the humeral head is driven posteriorly with force. Rotator cuff tendonitis (tendinopathy) is associated with pain and difficulty actively abducting or rotating the arm. Overuse injuries often cause irritation of the tendon sheath, causing tendonitis. Symptoms are usually pain and weakness made worse by overhead activities and abduction and external rotation. The degree of trauma depends on the type of tear. With full thickness tears, the degree of trauma is significant, as with a major fall, MVA, or shoulder dislocation. Partial thickness tears can be caused by a lesser degree of trauma; overuse injuries can cause tears as well. MRI is the imaging of choice.

Case A 45-year-old man presents for evaluation of ongoing left hip pain which he has been unable to control at home, since falling from a roof 8 weeks ago. The patient was evaluated at the ER right after the fall and had a left hip X-ray done that was unremarkable. He states: "I have had a minor limp since the fall but now I can barely stand on the leg." He rates the pain 10/10 when weight bearing or moving the leg. He states the pain radiates to his groin and anteriomedial thigh. He denies fever, urinary complaints, numbness, or weakness. Physical exam reveals tenderness on palpation of the left hip and increased pain with active and passive range of motion in all planes. Distal pulses are normal; sensation is fully intact; and calf compartments are soft and compressible without a palpable mass. When asked to stand on his left leg, he is unable to do so secondary to pain and has resorted to using a wheel chair to maneuver. He has no chronic medical problems and takes only Ibuprofen for pain. You obtain an MRI of the hip, which reveals evidence of osteonecrosis. Question What is most appropriate next step?

Correct answer: Admit to the hospital for an inpatient orthopedic evaluation. Explanation The correct answer is to admit this patient to the hospital for orthopedic consultation. This patient presents with avascular necrosis of the left hip, and there are several concerning factors in this patient's presentation. First, the diagnosis of avascular necrosis (AVN) in a man at the age of 45 is likely to require surgical repair;1 second, his pain level of 10/10 will likely be difficult to control on an outpatient basis; and lastly, the patient is unable to bear weight, making him a significant fall risk. While it is unclear if the patient will require emergent surgery at this time, it is appropriate to admit this patient for an immediate orthopedic consultation. There is no evidence of secondary infection at this time that would prompt the initiation of antibiotics. There is no evidence that is concerning for a DVT in this patient that would require initiation of anticoagulation therapy. Additionally, there is the possibility of the patient undergoing operative care, and therefore, you would not want to anticoagulate him at this time. Osteonecrosis is not a rheumatologic disease, and involvement of rheumatology is not necessary.

Case A 62-year-old man presents with left knee pain. The patient states that he has had knee pain intermittently over the last 5 years, but it has gradually worsened over the last 6 months. He notes that his knee feels somewhat stiff in the morning, but improves when he begins moving around. However, after standing at his job for several hours, he develops an aching pain that is only relieved by rest. He has a history of hypertension and is a recovering alcoholic. His BMI is 26. On X-ray, there is joint space narrowing with the presence of osteophytes. Question What is the strongest risk factor for the patient's condition?

Correct answer: Age Explanation The correct answer is age. The patient is complaining of unilateral knee pain that had insidious onset, worsens with activity, is alleviated by rest, and is associated with morning stiffness. These symptoms, when paired with the characteristic X-ray findings of joint space narrowing and osteophyte formation, indicate osteoarthritis. Of the above, age is the only significant risk factor for osteoarthritis. Osteoarthritis is rare in patients under age 40. Male gender is incorrect. While younger patients with osteoarthritis are more often male, women in the patient's age group are more at risk for osteoarthritis. Hypertension is not associated with osteoarthritis. While history of alcohol use may contribute to vitamin deficiencies that could potentially increase risk for osteoarthritis, it is not the strongest risk factor for the condition. BMI of 26 is incorrect. A BMI at this level places the patient in the category of being overweight. Obesity is linked with osteoarthritis, and being overweight could have contributed to the patient's diagnosis. However, the patient's age would be considered the stronger risk factor.

Case A 44-year-old woman presents for annual examination. Her friend's mother fractured her hip 1 year ago and had signs of osteoporosis. The patient would like to know if she should be screened for osteoporosis. She has past medical history of hypertension. She denies smoking and drinks 1-2 glasses of wine per week. Her last menstrual period was 2 weeks ago. Question When should the patient begin bone mineral density (BMD) screening

Correct answer: Age 65 Explanation Age 65 is correct. The National Osteoporosis Foundation recommends bone mineral density testing for all women over 65 years old unless they exhibit risk factors for osteoporosis. While female gender is a risk for osteoporosis, the patient does not exhibit other risk factors such as current smoking, excessive alcohol intake, or chronic steroid use. She also does not have conditions that cause secondary osteoporosis, such as hyperparathyroidism and hypothyroidism. The patient should begin screening at age 65 unless her clinical condition changes. Immediately, at the onset of menopause, age 50, and age 55 are all incorrect. Screening may be done in these time periods but is only recommended if the patient is at high risk for osteoporosis or has a disease that may contribute to secondary osteoporosis.

Case A 76-year-old man presents with a 2-day history of a red, hot, and swollen ankle. He denies fever, chills, headache, and any other symptoms. He is unable to put his weight on the foot, and he denies any trauma. He is a diabetic with poor glucose control. Laboratory analysis shows negatively birefringent crystals in the synovial fluid; there is no evidence of bacterial growth. The patient receives treatment, and his condition improves rapidly, but he has a similar attack 6 months later. Question What treatment is used in the prophylaxis of this condition?

Correct answer: Allopurinol Explanation This patient has gout, which is a form of inflammatory arthritis; it is the body's response to the deposition of uric acid crystals in the joints. Gout presents as acute monoarticular arthritis in 90% of patients. In early gout, usually only 1 or 2 joints are involved (usually, the smaller lower extremity joints). Podagra, or inflammation of the first metatarsophalangeal joint, is the initial joint manifestation involved in about half of all cases, but podagra may also be observed in patients with pseudogout, reactive arthritis, gonococcal arthritis, psoriatic arthritis, and sarcoidosis. The attacks usually begin abruptly, and they can reach maximum intensity in 6-12 hours. The joints are red, hot, and extremely tender. Untreated, the characteristics of gout change over time and the attacks become more polyarticular. Although more joints may become involved, inflammation in a given joint may become less intense. Attacks occur more frequently and last longer; eventually, patients may develop a chronic polyarticular arthritis, which can be symmetrical and resemble rheumatoid arthritis. Tophi, which are collections of uric acid crystals in the soft tissues, occur frequently in untreated patients. They can be found in multiple locations, including the fingers, toes, and olecranon bursae; they can also be found along the olecranon, where they may appear to be rheumatoid nodules. Tophi tend to develop after 10 years in untreated patients who develop chronic gouty arthritis. Acute flares of gout can occur in situations that lead to increased levels of serum uric acid (e.g., the use of alcohol, overindulgence of certain foods rich in protein, trauma, hemorrhage, or the use of medications that elevate levels of uric acid). Allopurinol is the best answer for this patient who is elderly and diabetic with a history of gout. This drug is used in the prevention of future attacks; it is not used to treat acute flare-ups. Patients with frequent gout attacks may begin prophylactic treatment with allopurinol to prevent further episodes. Allopurinol blocks xanthine oxidase, thereby reducing the production of uric acid; therefore, it should be used in patients who overproduce uric acid. It is the most effective agent to lower serum uric acid levels. The side effects include rash, gastrointestinal symptoms, headache, urticaria, interstitial nephritis, and (rarely) hypersensitivity syndrome. It was originally developed as a chemotherapeutic agent. Alcohol can interfere with its effectiveness. Sulfinpyrazone, a uricosuric agent, is an alternative agent to aid in the prevention of attacks; however, it is not used as frequently since it can cause bone marrow suppression. Prednisone, a glucocorticoid, is similar in efficacy to other agents and has no greater risk of adverse effects in most patients, although other options may be preferred in patients with concomitant infection, prior glucocorticoid intolerance, diabetes, and those who are in a postoperative period in whom glucocorticoids may increase risk of impaired wound healing. NSAIDs have an anti-inflammatory effect that works by inhibiting cyclooxygenase, which acts to produce leukotrienes from arachidonic acid. Indomethacin is the drug of choice unless the patient is elderly or has underlying health problems, but other NSAIDs (e.g., ibuprofen, naproxen, sulindac, and ketoprofen) may be used. Aspirin cannot be used because it can elevate uric acid levels. Some of the side effects of NSAID therapy include gastropathy, nephropathy, and liver dysfunction. It can also cause fluid overload in patients with congestive heart failure, so its use should be limited in these patients. Colchicine is an antimitotic drug that is often used in patients who cannot take NSAIDs or corticosteroids. Due to its side effects (e.g., nausea, vomiting, diarrhea and, following intravenous administration, bone marrow suppression, renal failure, and death), however, it is not used as a first-line treatment.

Case A 25-year-old man presents with back pain and stiffness. He states that he has had longstanding issues with back pain, although he denies any trauma to his back. He has noticed, along with the pain, an increasing presence of stiffness and general fatigue. He feels that these issues have gradually worsened over the last several months, and they are more persistent recently. The pain is much worse first thing in the morning, and he rates it a 7/10 on a numerical pain scale. Radiation occasionally occurs into the buttock areas bilaterally and the patient feels the symptoms markedly lessen with activity. Physical examination shows marked forward stooping of the thoracic and cervical spine, with the lower spine showing presence of a substantial reduction in lateral flexion. Question What is high on your differential diagnosis?

Correct answer: Ankylosing spondylitis Explanation The patient history and physical examination being described above is most likely a case of ankylosing spondylitis (AS). This is a chronic inflammatory disease that consists of many signs of symptoms, specifically significant back pain and progressive spinal stiffness. Many times patients with AS will also have transient or persistent peripheral arthritis as well as other manifestations such as anterior uveitis. Typically this is seen in male patients who are in the age range of 20-30 years old, although some may start having issues as early as their late teens. The goals of management of AS is to maximize long-term health-related quality of life: relief of symptoms, maintenance of function, prevention of complications from spinal disease, and minimize extraspinal and extraarticular manifestations and comorbidities. Spondylolisthesis is described as a forward or backward displacement of a single or multiple vertebrae. Patients with this condition may very well not have any symptoms, but if signs or symptoms are present, they could include walking abnormalities, postural problems, constant pain in the lumbar region, and (especially) leg pain. Often, this pain is worsened with activity and relieved with rest. Osteoarthritis is not as likely due to the fact that this is generally seen as a disease of aging; 90% of patients over the age of 40 years old have some degree of radiologic evidence of osteoarthritis. It also affects more women than men. Symptoms and signs usually affect the PIP joints, DIP joints, carpometacarpal joint of the thumb, the hip, knee, or secondary in origin to a joint that has other injuries (mechanical overuse, metabolic disease, or mechanical injury.) Onset is insidious. There is stiffness of 15 minute duration, with pain made worse with activity or weight bearing, and it is relieved by rest. Flexion contracture may be evident as well as potentially Bouchard or Heberden nodes. No ankylosis is present, but there may be limitation of the joint, crepitus, or effusion may be evident. Rheumatoid arthritis is not as likely in this case scenario; ithas an insidious onset with characteristic morning stiffness and symmetrically affecting small joints of the hands and feet. Psoriatic arthritis also is not consistent with the patient scenario above due to the fact that psoriasis usually precedes the arthritis component is around 80% of the cases. This form of arthritis affects the DIP and SI joints. Usually, this is asymmetric, and it may present with a classic 'sausage' appearance of the affected digits. References McPhee SJ, Papadakis MA, Rabow, MW. 2014 Current Medical Diagnosis & Treatment.

Case A 27-year-old man presents with knee instability. During a game of basketball, the man jumped up for the ball and landed incorrectly. He felt and heard a "pop". He experienced instant swelling and pain; when he attempted to apply weight, it felt like his knee was going to "give out". On exam, he has a positive Lachman sign and anterior drawer sign. McMurray's sign, posterior drawer sign, and varus and valgus stress test are all negative. Question What type of injury does this patient have?

Correct answer: Anterior cruciate ligament Explanation The patient has an anterior cruciate ligament (ACL) injury. Applying the Lachman test and anterior drawer sign will increase the translation between the femur and tibia; an intact ACL would have decreased translation and an endpoint. It is very important to exam the unaffected knee first in order to compare it to the affected joint. Patients typically say they felt something "pop", and they have difficulty with ambulation. An MRI can confirm the diagnosis and the extent of the tear. A prompt referral to the orthopedist is warranted. A posterior cruciate ligament injury is suspected if there is a positive sag sign and posterior drawer sign. Usually, the mechanism of injury is when the knee is flexed and there is associated force (e.g. hitting a dashboard during a car accident). A medial collateral ligament injury is suspected when a valgus stress produces a knee laxity. It presents with tenderness to palpation of the medial aspect of the knee. If there is gross laxity, then an ACL injury should also be suspected because the ACL also provides some medial stability. A mechanism of injury would be a football player being tackled from the lateral side of the knee. A lateral collateral ligament injury is suspected with a positive varus stress. These are more common with high-velocity injuries because the opposing leg provides some protection. A meniscal injury usually presents with a positive McMurray's sign and Thessaly test; the patient typically complains of joint line tenderness or a knee locking sensation. Meniscal injuries usually occur together with ACL tears because the mechanisms of injury are the relatively the same.

Case A 20-year-old male university student presents with weakness in the right hand. The student is a member of the university rowing club, and his right hand is the main rowing hand. The onset of weakness has been progressive over the past 2 months since beginning a more intensive training program at the rowing club. On physical exam, the patient is very muscular and well-developed. There is tenderness over the ventral surface of the right forearm approximately 3 inches distal to the cubital fossa near the midline. Tapping in the area produces no numbness in the hand. Weakness is evident on attempted flexion of the interphalangeal joint of the thumb and the distal interphalangeal joints of the index and middle finger. All other tests reveal normal motor function; there are no sensory deficits. Question What is the most likely diagnosis for this patient?

Correct answer: Anterior interosseous nerve entrapment Explanation The correct answer is entrapment of the anterior interosseous nerve as it leaves the pronator teres muscle and enters the deep anterior compartment of the forearm. It occurs due to hypertrophy of the pronator teres muscle. Problems affecting the anterior interosseus nerve can be distinguished from those of the proximal median nerve because the former has no sensory fibers; it therefore has only motor findings. In this condition, there is weakness in the distribution of the flexor pollicis longus and flexor digitorum profundus involving the 2nd and 3rd digits. The classical physical finding is the inability to form the "OK", or pinch sign, with the thumb and the index finger. Also, inability to flex the wrist or clench the fist may be seen. Non-surgical treatment includes rest, splints, and anti-inflammatory medication; surgical treatment involves decompression. Median nerve entrapment above the elbow could produce all of the findings noted. However, one would also expect to find sensory deficits in the hand and weakness of thumb opposition. Pronator teres tendonitis could produce the tenderness noted in this setting, but would not affect the thumb and fingers. The diagnosis of radial nerve palsy is made when there is evidence of wrist-drop and sensory loss over the first dorsal interosseous. The patient may feel that his grasp is weak, secondary to loss of wrist extension necessary for powerful grip. The radial nerve may be affected if the nerve is compressed against the middle third of the humerus. Ulnar nerve entrapment would produce sensory deficits in the hand and weakness of distal interphalangeal joint flexion of the ring and 5th finger.

Question A 19-year-old man presents with pain and deformity of his right dominant shoulder after a sudden jerking movement to the same from a wrestling competitor approximately 1 hour ago. He states he felt a clunking sensation when it happened. He was unable to continue wrestling and has pain with movement of the right shoulder. What diagnostic studies should be performed?

Correct answer: Anterior/Posterior (AP) and axillary or transscapular lateral radiographs Explanation AP and axillary lateral or transscapular lateral (Y-scapula) views should be obtained. It is imperative that a lateral view of the shoulder joint be obtained to assess the position of the humeral head. Posterior dislocations can be easily missed if the lateral view is not obtained. Reference:

Case Ico-delete Highlights A 24-year-old woman presents with fever, joint pain, fatigue, hair loss, and a rash over the bridge of her nose and cheeks. Physical exam reveals patchy alopecia, a malar rash, and symmetrical joint tenderness. Results of initial testing include an elevated erythrocyte sedimentation rate (ESR) and a positive antinuclear antibody test. Her serum creatinine was elevated, and urine test was positive for protein. Question Ico-delete Highlights Which commonly ordered test is more likely be positive given this patient's probable diagnosis?

Correct answer: Antibodies to double-stranded DNA Explanation The correct response is antibodies to double-stranded DNA. The patient's presentation is consistent with a diagnosis of systemic lupus erythematosus (SLE). SLE is a non-organ specific, autoimmune disease that can markedly vary from patient to patient. Most patients have a combination of complaints that include constitutional symptoms, as well as skin, hematologic, renal, and musculoskeletal systems. Patients generally develop several different autoantibodies. SLE is more common in women of childbearing age and in African American and African Caribbean persons. Patients can experience relapses and remissions. Antinuclear antibody (ANA) develops in practically all patients with SLE at some point in the disease course. Â Positive ANA is one of the Systemic Lupus International Collaborating Clinics criteria (a revision of the American College of Rheumatology's 11 criteria). SLICC also requires at least four positive criteria, but these must include at least one of 11 clinical criteria (eg, malar rash) and at least one of six immunologic criteria. ANA is usually one of the immunologic tests ordered when SLE is suspected, and, if positive, is often followed up by other specific tests. Antibodies to double-stranded DNA are highly specific for SLE, and seen in about 70% of patients. It is another of the SLICC immunologic criterion. One clinical criteria, biopsy-proven lupus nephritis is thought to indicate SLE if the patient is also positive for anti-double-stranded DNA antibodies or positive for ANA. Anti-Smith (Sm) antibody is also highly specific for SLE, but the test has low sensitivity. Anti-Sm antibodies are seen in about 30% of patients with SLE. Anti Ro/SSA and anti-La/SSB antibodies are more commonly associated with Sjogren syndrome. They are seen in about 30% and 20% respectively of patients with SLE (often in patients with ANA-negative disease). Anti-cyclic citrullinated (CCP) peptide antibody is usually for diagnosing rheumatoid arthritis, although it may be highly specific for patients with SLE and erosive arthritis. Lupus erythematosus (LE) cell test is now considered obsolete. It is less sensitive, time-consuming, and has been replaced by ANA testing.

Case A 27-year-old Caucasian man returns to the emergency department with unbearable left lower leg pain; he does so approximately 6 hours after initial discharge. While playing lacrosse, the patient sustained a closed, mid-shaft tibial fracture. After casting and an anti-inflammatory, his pain was noted to be mild (2 out of 10 on 1 - 10 scale) at time of discharge. He reports his pain is increasing dramatically (it is now rated at 9 out of 10) and is unresponsive to his prescribed narcotic, acetaminophen, icing, and elevating his leg. He also describes a feeling of tingling and numbness throughout the lower left extremity. Question What is the most likely cause of this patient's severe pain?

Correct answer: Compartment syndrome Explanation The most likely diagnosis is compartment syndrome, which is an emergency. Without prompt identification and treatment, blood flow is reduced and can result in tissue necrosis and permanent nerve and muscle damage. Compartment syndrome can occur after a trauma, such as a fracture or burn. As increasing fluid pressures build in the area of injury, the restrictive fascia prevents outflow. In this patient's case, his cast may have been applied too tightly and should be removed immediately. If his symptoms do not resolve, he may need a surgical fasciotomy. Key features of compartment syndrome include a history of trauma (especially to an extremity), edema, and pain out of proportion to history/findings. Pulselessness is a late finding. Claudication is a condition in which the patient experiences lower extremity pain secondary to poor arterial blood supply. It is associated with peripheral arterial disease. Claudication should classically worsen with movement (such as prolonged walking) and improve with rest. This patient's pain did not improve with rest and his history is inconsistent with peripheral arterial disease. Malingering or drug-seeking behaviors should be evaluated in any patient complaining of pain out of proportion to history, as the patient may be attempting to deceive the provider. This patient's history was not concerning for prior history of psychiatric illness or substance abuse. The objective physical exam findings on this patient (reduced capillary refill and sensation, edema and shiny, taut skin) help substantiate a diagnosis of compartment syndrome. Sepsis is concerning condition that could cause edema and pain in an extremity. However, the patient had a history of a closed fracture (much less likely to develop sepsis than with an open fracture) and is afebrile. This patient could be at a future risk of sepsis, but in this scenario, he does not present with systemic symptoms of profound infection. Depending upon the mechanism of injury, this patient may have fractured both his tibia and fibular. Sometimes, a 2nd fracture can be missed in initial evaluation. However, a fibular fracture would not explain this patient's severe pain. In contrast, a fibular fracture would be expected to respond to immobility (with the cast), ice, analgesics, and elevation.

Case A 40-year-old African-American woman presents with diffuse headache and joint pain. The headache started few days ago; it is dull and becoming progressively worse. Joint pain is localized in fingers, starts in the morning, and improves during the day; it returns when she tires. 2 months ago, she was treated in the ED because of several weeks of lasting fatigue, low-grade fevers, joint pain, hair loss, and oral ulcers. Her laboratory tests were normal, except for positive VDRL and antinuclear antibody tests results. Urine and blood cultures were negative for evidence of infection, and her chest X-ray was normal. Because she felt better after 10-day tapering course of prednisone, she did not appear to the scheduled control. Today, your examination reveals an ill-appearing woman in distress. Her temperature is 39 C. Her fingers are swollen and red, and she has malar rash and oral ulcers. Question What additional test should you order as the most specific for her condition?

Correct answer: Antibodies to the Sm antigen Explanation The diagnosis of SLE is based on clinical features and the presence of autoantibodies. Current criteria for the diagnosis are 4 or more of the following: malar rash, discoid rash, photosensitivity, oral ulcers, arthritis, serositis, renal disorder, neurologic disorder, immunologic disorder, and antinuclear antibodies ("1997 Update of the 1982 American College of Rheumatology Revised Criteria for Classification of Systemic Lupus Erythematosus"). This patient meets the criteria. She has a history of positive ANA and alopecia; there is the presence of a neuropsychiatric disorder, oral ulcers, arthritis, and fever, with infection ruled out. Antibodies to the Sm antigen in the presence of characteristic clinical picture are the most specific for SLE. VDRL is often false positive in SLE patients (in about 1/4 patients). Clinically, VDRL positive patients present differently than VDRL negative. Noninflammatory synovial fluid will contribute to the diagnosis of SLE only in the presence of compatible clinical manifestations. Synovial fluid analysis is not necessary in this patient. Urine protein/creatinine ratio demonstrating kidney disease favors the diagnosis of SLE in the presence of compatible clinical manifestations. It is not necessary in this patient. Cerebrospinal fluid analysis is not indicated in a case of a headache without focal neurological signs and a clear clinical picture of SLE. A complement test may be used to monitor the course and treatment efficacy in patients with autoimmune disorders. Patients with active SLE may have lower-than-normal levels of the complement proteins C3 and C4, particularly in the presence of lupus nephritis. Complement system test is not specific for SLE.

ase A 32-year-old woman presents with a 2-day history of sharp pleuritic-type chest pain. She indicates that the pain worsens when she lies down and feels better when she sits upright. She also admits to having polyarticular arthritis involving the small joints of her hands. Her obstetric history is positive for 2 first trimester spontaneous abortions. You order a CXR, which reveals pericarditis. Question What laboratory test could you order to help assist in your diagnosis?

Correct answer: Antinuclear antibodies (ANA) Explanation The clinical picture is suggestive of systemic lupus erythematosus (SLE). Antinuclear antibodies are positive in 95-100% of patients with SLE. Patients with SLE can present with an abnormal CBC (anemia, leukopenia, and thrombocytopenia), but it is not diagnostic in SLE. Rheumatoid factor can be present in 20% of patients with SLE, but it is not diagnostic for SLE. The parathyroid hormone would be normal in a patient with SLE. Antineutrophil cytoplasmic antibody is only 0-1% positive in patients with SLE.

Case A 42-year-old African-American woman presents with acute deterioration of her mental state; she is hallucinating and talking about people hiding under her bed. She cannot sleep, and she refuses to eat. Several months ago, her family practitioner told her that she might have lupus erythematosus; however, she refused further investigations. On examination, she appears agitated and is disorientated to place and time; she has impaired memory, and she confirms the presence of auditory hallucinations. Question What test would best confirm the diagnosis of cerebral systemic lupus erythematosus?

Correct answer: Antinuclear antibodies in blood Explanation Antinuclear antibody (ANA) testing is used for serologic testing for SLE. There are 2 major types of ANA: autoantibodies to DNA and histones and autoantibodies to extractable nuclear antigens. The first group concerns antibodies against single and double-stranded DNA (dsDNA); significant levels of anti-dsDNA antibodies are considered confirmatory in the diagnosis of SLE (anti-histone antibodies are considered confirmatory in the diagnosis of drug-induced SLE). Most instances of the second group (autoantibodies to extractable nuclear antigens) are specific to a disease, e.g., autoantibody to Smith antigen is considered to be specific for SLE. Other examples include: autoantibodies to ribonucleoproteins, SSA/Ro, or SSB/La, Scl-70, Jo-1, PM1, etc. Oligoclonal bands are immunoglobulins seen in serum, plasma, or cerebrospinal fluid (CSF). The presence of oligoclonal bands in CSF with their absence in serum shows that immunoglobulins are produced within the central nervous system. Since 80-90% patients with multiple sclerosis have oligoclonal bands, they are considered an indicator in the diagnosis of multiple sclerosis. They are not confirmatory for SLE. Antineuronal antibodies are commonly associated with paraneoplastic neurological diseases (Anti-Hu, Yo, Ri amphiphysin, Tr, CV2, and Ta antibodies). Some of these antibodies are specific for certain types of cancer or neurological syndromes. They are not confirmatory for SLE. Protein levels in CSF are not specific; they may be elevated in several conditions (e.g., diabetes mellitus, brain tumor, brain abscess, meningitis, multiples sclerosis, hemorrhage, Guillain-Barre syndrome); they may be normal in viral infections, or they may be low (recent lumbar puncture, chronic CSF leakage, water intoxication). Complement system plays a role in many diseases with an immune component (asthma, glomerulonephritis, multiple sclerosis, inflammatory bowel disease, etc); therefore, its levels are not specific for SLE. References

Case A 14-month-old girl presents in the middle of the night. Her mother tells you that she has been fussy and has not been feeding well for the past 48 hours. In the last 4 hours, she has developed a high fever; it was 102 degrees Fahrenheit when measured at home. Her mother also notices that the child appears reluctant to move her right leg; the girl appears to keep it rotated outward. Examination reveals a distressed, febrile child. Her cardiovascular, abdominal, and respiratory exams appear normal. Her right leg is held fixed in external rotation and partial flexion, with minimal hip joint effusion. A needle aspirate of the joint fluid reveals the following; WBC count - 60,000/cc, neutrophils 95% No crystals Gram stain is positive for Gram-positive cocci in clusters Question What is the most important step in preventing destruction of the joint?

Correct answer: Arthrotomy and irrigation of joint space Explanation The correct answer is to perform an arthrotomy and irrigate the joint space. The child has septic arthritis, which is proven by the history and joint fluid analysis showing a white count of >50,000 with predominant neutrophils. In addition, Gram stain shows organisms likely to be staphylococcus. Septic arthritis in a patient so young is an emergency. Without drainage, accumulation of pus in the joint results in rapid destruction of the articular cartilage and permanent disability. Infants often present with non-specific symptoms, and the classical symptoms of septic arthritis may appear late in the course of the disease. Early administration of antibiotics is incorrect. While antibiotics are important, they are not the primary means of preventing disability in a child with septic arthritis. High dose acetaminophen administration is incorrect. High doses of acetaminophen are hepatotoxic, and it will only reduce pain. Repeat needle aspiration is incorrect. The initial aspirate leaves no doubt as to the diagnosis of septic arthritis; therefore, a repeat needle aspiration is unnecessary. Administration of indomethacin/allopurinol is incorrect. Both drugs are used in gout; they are not used in septic arthritis.

Question A 17-year-old boy presents following an injury to his right dominant upper arm while playing football a half hour ago. While attempting to throw a pass, an opposing player grabbed his arm and tackled him. The opposing player landed directly on the thrower's flexed elbow, while driving the thrower's shoulder into the ground.The injured player's mother heard a "cracking sound" in the stands. He had immediate pain and exhibited a deformity in the distal 1/3rd of his right arm. Besides the inspection findings of edema, ecchymosis, and deformity, what specific physical examination technique should be performed?

Correct answer: Assessment of wrist extension and dorsal hand sensation Explanation Since radial nerve injuries can be associated with this fracture, checking the neurological status of the radial nerve by asking the patient to extend his wrist (which should not be painful at the fracture site) and checking the sensory distribution on the dorsal hand is imperative. The axillary nerve is proximal to the fracture and is unlikely to be affected. Compartment syndrome, although possible, is a later finding and not one of the first concerns during the physical exam. Lymphadenopathy is not an acute finding due to a fracture. Even gentle minimal passive range of motion of the elbow and arm is painful, so full passive range of motion should NOT be attempted.

ase A 52-year-old Caucasian man presents with pain in his left lower extremity. The patient is well known to your practice. Upon further questioning, the patient describes doing construction work 3 months prior to presentation; he jumped onto his feet from an elevated height of 6 ft. Since this episode, he has noticed increasing left sided hip and knee pain. He describes the pain as radiating into the left groin and front middle thigh area. The pain is relieved with sitting and aggravated by walking and climbing up stairs. The patient denies any paresthesias, numbness, bowel/bladder dysfunction, fever, night sweats, or chills. Pertinent medical history includes alcoholism for the past 15 years; there have been 3 episodes of acute pancreatitis in the last 4 years. They required hospitalization. Physical examination reveals a patient with a new onset of a limp as well as restricted and painful active and passive joint movement of the left lower extremity. Question What is highest on your differential diagnosis at this time?

Correct answer: Avascular necrosis Explanation Avascular necrosis (AVN), which is sometimes also referred to as osteonecrosis, is osseous cell death resulting from vascular compromise. Common sites that are affected include the proximal or distal femoral head or even the ankle, shoulder, or elbow. Common causes include but are not limited to corticosteroid use, alcoholism, trauma, systemic lupus erythematosus, pancreatitis, gout, and sickle cell disease. The patient presented shows several risk factors for development of avascular necrosis, placing this very high on the most likely diagnosis. Occasionally, AVN will be asymptomatic and found incidentally on radiographic images. Symptoms will vary depending on the joint that is affected; pain in that joint is most common. Patients may experience groin pain that is worsened with weight bearing actions. This pain will usually progress to being present while at rest. On physical examination, patients who walk with a new limp are usually highly suspicious of having AVN progression and deterioration. Patients will have tenderness around that affected bone tissue; they will also have significant restriction as well as pain with active and passive joint movements. Neurologic deficits may also be present due to nerve compression from the necrosis. Severely advanced AVN can present with joint deformity or even muscle wasting. Multiple myeloma typically initially presents as spine, rib, or proximal long bone pain; it is more commonly in older adults, with the median age of presentation being 65 years. Many of these patients will present with signs and symptoms relating to pathology, including anemia, infections, kidney failure, spinal cord compression, or even hyper-viscosity syndrome. Osteoporosis is not common in this patient population group; therefore, it is much lower on the differential diagnosis. Its occurrence is higher in Caucasian women, with other risk factors such as tobacco or alcohol abuse, excess/deficiency in hormones, malignancy, and even some genetic disorders. Osteoporosis is very commonly asymptomatic until actual fractures occur from this pathology. Many times it will create back pain of varying degrees, depending if a fracture is present or not. Although the patient may have some signs of osteoarthritis on future imaging studies, it is most likely not the main cause of what has brought him in to see you now. Rheumatoid arthritis has an insidious onset, with characteristic morning stiffness, and it symmetrically affects small joints of the hands and feet. References

Case A 38-year-old man presents with insidious onset of non-specific left groin pain for 6 weeks. The pain is worse with weight bearing, and there is pain at rest. He has noticed having night pain for the past 4 weeks. The patient has a history of high dose corticosteroid use throughout his life for recurrent lung infections. On exam, there is decreased range of motion with internal rotation of the left hip. Subchondral radiolucency is noted on plain radiographs. Question What is the most likely diagnosis?

Correct answer: Avascular necrosis Explanation Avascular necrosis is correct because the hip is the most commonly affected joint for this disease. Subchondral radiolucency, also known as crescent sign, is pathopneumonic for this disease. Although many times the patients can be asymptomatic, if symptoms are present, they are usually non-specific hip or groin pain. Limited motion with internal rotation and abduction is seen on physical exam. Worsening pain with weight bearing is common. He also has a history of high dose corticosteroid use, which is one of the most common risk factors for this disease. Femoral neck fracture is incorrect; it usually occurs in young athletes or elderly patients with a history of osteoporosis. These types of fractures associated with multiple injuries and avascular necrosis often results. Usually, there would be history of an injury, especially in the elderly population. Plain radiographs should show shortening or femoral neck angulation. Most patients have severe pain. Osteoarthritis is incorrect; it usually occurs over time from wear and tear. Plain radiographs would show decrease in joint space. Osteoarthritis is usually more common in older patients. Polymyalgia rheumatica is incorrect; the hallmark is pain and stiffness in the shoulders and hips. This frequently occurs along with Giant cell arteritis. This usually affects people over the age of 50. Trochanteric bursitis is incorrect; there is usually pain when lying on the affected side. Physical exam would show pain with palpation over the greater trochanter as well as pain with resisted abduction or adduction depending on if the deep or superficial bursa is affected. Trochanteric bursitis more commonly affects women.

ase An 86-year-old man presents with a history of a dislocated right hip. For the past 3 weeks, he has noticed pain where his hip dislocated when putting weight onto it. He reports a history of alcohol abuse and use of steroids for his rheumatoid arthritis. Upon physical exam, there is limited range of motion in the right hip but he is afebrile. An X-ray of the right hip shows sclerosis of the bone; however, a synovial fluid analysis is normal. Question What is the most likely diagnosis?

Correct answer: Avascular necrosis Explanation The clinical picture is suggestive of avascular necrosis. Avascular necrosis commonly happens after a traumatic injury such as a dislocation. Additionally, alcohol use and steroid use are noted, which can be risk factors for avascular necrosis. Lastly, the findings demonstrate sclerosis of the bone on the X-ray. Legg-Calvé-Perthes Disease is not the correct answer, as Legg-Calvé-Perthes Disease occurs commonly in the age range of 4-9. Patients with Legg-Calvé-Perthes Disease have risk factors, such as maternal smoking during pregnancy and low birth weight. Osteochondritis dissecans is not correct because osteochondritis dissecans is a lesion of subchondral bone that causes a separation and instability overlying articular cartilage. Additionally, osteochondritis dissecans more commonly occurs in adolescents. Septic arthritis is not the correct answer because septic arthritis most commonly presents with a fever, which this patient does not have. Additionally, a synovial fluid analysis of 145 WBC/mm3 is in a normal range and therefore rules out septic arthritis. Sesamoiditis is not correct because sesamoiditis affects the sesamoid bones under the first metatarsophalangeal joint. References Papadakis MA, McPhee SJ, Rabow MW. Current Medical Diagnosis &Amp; Treatment. New York: McGraw-Hill Medical; 20

Case A 32-year-old man is working on his farm when a wild horse suddenly jerks the bridle, forcing the man's right arm over his head and backwards. There is a "clunking" sound, and the man is in immediate pain and unable to move his arm. There is a deformity of the shoulder with a depressed area (dimple) noted in the anterior shoulder. Question What nerve must be carefully assessed in this patient before and after it is fixed because of the possible injury to it?

Correct answer: Axillary Explanation With a glenohumeral dislocation, as in this man, a risk of neurapraxia of the axillary nerve is possible. Neurovascular function, particularly that of the axillary nerve, must be carefully assessed before and after reduction of the dislocation. Anterior-inferior displacement of the humeral head into the quadrangular space has the potential of injury to the axillary nerve. The musculocutaneous nerve supplies the coracobrachialis muscle, leaves the axilla by piercing that muscle, and is not exposed to the force of dislocation. The median nerve arises from the lateral and medial cords of the brachial plexus, passes downward on the lateral side of the axillary artery, and also is not exposed to the force of dislocation. The radial nerve, the longest branch of the brachial plexus, lies behind the axillary artery and also is out of harm's way. The ulnar nerve arises from the medial cord of the brachial plexus, descends in the interval between the axillary artery and vein, and is also not subject to the force of dislocation.

Case You are evaluating a 28-year-old man who is brought in by ambulance for a right knee injury sustained during an MVA. The patient is in obvious distress, holding his right knee and screaming. You are told by EMS that the patient was the restrained driver of a car that was rear-ended at high speed. You are unable to obtain any additional history. Examination already shows the development of a right knee effusion. The skin is intact, and there does not appear to be any deformity. Examination of laxity is limited, secondary to the patient's pain. It appears that distal sensation is intact but you cannot palpate a dorsalis pedis or posterior tibial pulse. You order an emergent MRI for further evaluation. Question In the meantime, what test would be best to help confirm the patient's diagnosis?

Correct answer: Bilateral ankle-brachial indices Explanation The correct answer is bilateral ankle-brachial indices. This patient's history and exam are concerning for a knee dislocation. Knee dislocations are classified according to the direction that the tibia is displaced in relation to the femur. Anterior knee dislocation is the most common type, and often due to a high-velocity injury such as an MVA. While rare, dislocations of the knee have significant complications involving both structural damage to the knee (disruption of tendons, ligaments, and cartilage), as well as neurovascular injuries (injury to the popliteal and/or tibial arteries, and damage to the peroneal nerve most commonly). The Apley grind test is used to evaluate a meniscus injury. It is done by having the patient lay prone with their knee bent at 90 degrees. The examiner then applies downward pressure by pushing down on the bottom of the patient's foot while rotating the foot. The test is positive if pain is reproduced. The Lachman test is performed by having the patient supine and the knee flexed at 30 degrees. One hand firmly holds the distal femur, and the second-hand pulls the proximal tibia anteriorly. Excessive anterior movement of the tibia indicates an injury to the anterior cruciate ligament. Valgus stress is done with the patient in the supine position. One hand supports the lateral aspect of the knee while the other hand grasps the patient's ankle and applies valgus stress (move the ankle joint to the lateral side). This test is used to assess if there is laxity of the medial collateral ligament. The McMurray test is used to evaluate for a meniscus injury. It is performed on a supine patient by bringing the knee up to 90 degrees of flexion. The examiner's hand is then placed on the medial aspect of the knee, and the other hand externally rotates the foot and then extends the leg. If a "click" is appreciated, the test is positive for a medial meniscus injury. The opposite is done to assess for a lateral meniscus injury.

Question In your new practice, you have inherited 3 adolescent patients who had different early childhood malignancies that were treated in different ways according to the type and grade of cancer. They have been in remission now for over 5 years. In reviewing their charts, you consider the need to monitor for late effects of childhood cancer. What secondary malignancy is commonly associated with radiation therapy?

Correct answer: Bone and soft tissue sarcoma Explanation Radiation is associated with the development of cancer within the radiation field, notably, breast cancer, thyroid cancer, skin cancers, brain tumors, and most commonly bone and soft tissue sarcomas. The cumulative risk for secondary malignancy at 20 years of age in pediatric cancer survivors is 3% to 10%, which is 5 to 20 times greater than for the general population. Survivors should be evaluated yearly for the development of secondary malignancies. Hepatoblastoma has been associated with familial polyposis coli; any survivors should be screened for colon cancer. Patients that were treated with cyclophosphamide have a higher risk of developing bladder cancer, with a cumulative dose directly related to increased risk. The occurrence of non-glomerular hematuria identifies this group. It is not known which major cyclophosphamide metabolite drives bladder carcinogenesis. Survivors of Hodgkin's disease are likely to develop secondary malignancies such as acute myeloid leukemia, non-Hodgkin's lymphoma, breast cancer, and other solid tumors. The risk for some of these malignancies increases with time. Children with a genetic form of retinoblastoma, which includes all who have bilateral disease and about 10% who have unilateral disease, have a 50% chance of developing a secondary malignancy. References

Case A 40-year-old woman presents occasional constipation that she relates to her diet. She is married with 2 sons. Aside from mild anxiety, she is dealing appropriately with her duties as a librarian. She does not smoke, she consumes alcohol only recreationally, and she has a body weight of 187 lb. She is well-nourished and communicates well. Lung, cardiac, abdominal, and neurological examinations are unremarkable. Thyroid is normal in size and consistency. Menses are normal, and drug history is negative except for "hormone pills" for contraception. Family history is remarkable for a mother with hypothyroidism and osteoporosis. Question What strategy is recommended for screening osteoporosis in this patient?

Correct answer: Bone densitometry at age 65 Explanation The US Preventive Services Task Force (USPSTF, 2002) recommends screening for osteoporosis among women 65 or older and women 60-64 with risk factors for osteoporotic fractures. This patient is 40 years old, outside the age range to be at risk for osteoporosis, which is ≥50. Bone densitometry, or dual-energy X-ray absorptiometry (DEXA scan), from the femoral neck is currently the best screening measure used. Body weight less than 154 lb and no current use of estrogen therapy are major factors associated with an increased risk of osteoporosis and fracture. In the presented case, an appropriate decision on the timing of initial bone densitometry should be based on the patient's osteoporosis risk factors at age 65. However, the most appropriate choice among the others is bone densitometry at age 65.

Case A 56-year-old medical secretary/transcriptionist presents with paresthesias in the first three fingers of both hands; it awakens her from sleep. She has had her symptoms for almost 1 year, but now the problem is getting worse. She says that she is worried because her father died from myocardial infarction after having pain in his arms. There is swelling of both of her wrists. Her EKG is normal. Question What is the most likely diagnosis?

Correct answer: Carpal tunnel syndrome Explanation Carpal tunnel syndrome is a chronic repetitive strain injury (CRSI). The median nerve is impinged at the wrist, resulting in tingling and numbness, which often awakens the person from sleep. Lateral epicondylitis is known as tennis elbow and is also a CRSI (chronic repetitive strain injury). Tendonitis occurs at insertions of the forearm extensor muscles at the lateral epicondyle of the elbow. Medial epicondylitis is also a CRSI and known as golfer's elbow. It is due to tendonitis at insertions of the wrist flexors and pronators at medial epicondyle of the elbow giving varying degrees of paresthesias in hands and fingers. Thoracic outlet syndrome is due to compression of the neurovascular bundle of the upper limb in the upper thoracic region. There is tingling, aching, and burning in the distribution of the ulnar nerve. This also may be due to CRSI. Phantom pain (phantom limb) is an entity in which patients feel pain at the site of an amputated limb.

Question The attached drawing demonstrates the wrist flexion test (Phalen's test). Refer to the image. If paresthesia in the hand occurs after resting the elbows on a table and allowing the wrists to fall into complete volar flexion for 1 minute, what condition does this indicate?

Correct answer: Carpal tunnel syndrome Explanation Carpal tunnel syndrome is a median nerve compression neuropathy that occurs at the wrist where the nerve passes beneath the transverse carpal ligament. Patients frequently present with numbness, pain, and tingling of the thumb, index, middle, and ring fingers. In long-standing cases, there may be atrophy of the median innervated thenar muscles. The wrist flexion test (Phalen's test) produces paresthesia in the hand in patients with carpal tunnel syndrome. Cubital syndrome is a compression neuropathy of the ulnar nerve in the cubital tunnel in the elbow. Sensory loss occurs in the small and ring fingers, as well as the dorsal ulnar aspect of the hand. Pronator syndrome is due to compression of the median nerve in the proximal forearm. Patients have vague forearm pain, easy fatigability, paresthesia, numbness in the thumb, index, and long fingers, and weakness of the hand. The Phalen test is usually negative, distinguishing this entity from the carpal tunnel syndrome. Anterior interosseous nerve entrapment produces no sensory deficit, but it results in the inability to flex the interphalangeal joint of the thumb and the distal interphalangeal joint of the index finger. Posterior interosseous entrapment may occur in the supinator muscle at the Arcade of Frohse; it results in an inability to extend the metacarpophalangeal joints of the fingers or thumb, or abduct the thumb with the wrist in the neutral position. There is no sensory deficit.

Question A 3-year-old boy has velvety lax skin, hyperextensible joints, and mitral valve prolapse. What is the most likely diagnosis?

Correct answer: Ehlers-Danlos syndrome Explanation There are several different types of Ehlers-Danlos syndrome (EDS). The clinical vignette above describes the classical, type I variant. The velvety, distensible skin, hyperextensibility of many joints, mitral valve prolapse, and hernia are all associated with the type I variant. Although the exact nature of the biochemical defect is just now being sorted out, all types of EDS are due to mutations in collagen genes. In recent experimental studies, a mutation in the gene encoding the α1(V)-chain of type-V collagen, was identified. Type- V collagen is associated with type-I collagen and is probably involved in assembly of type-I collagen fibers. Type-I collagen is abundant in ligaments of joints and in the dermis, thus the characteristic and unusual symptoms in the skin and joints.

Case A 32-year-old man presents due to low back pain. He reports a 3-year history of chronic low back pain that progressively worsened. He notes a lot of stiffness with any movements that involve his low back. He is especially stiff upon morning awakening and now has to ask his wife to help him get out of bed. He denies a history of trauma. He works in retail, with some bending and light lifting duties. He has taken acetaminophen repeatedly—even exceeding recommended doses—with no relief. He knows his paternal grandfather had some type of serious arthritis in his back, but he does not know the diagnosis. Other than his chronic back pain, the patient reports that he is in good health with no chronic conditions. He takes no regular medications, except for the analgesics mentioned. He has no allergies and has never had surgery. On physical exam, some limited range of motion is observed in his back and hips, and the patient reports discomfort with the motions. Otherwise, his exam is normal, with no visible edema, muscle wasting, or bony abnormality. You obtain a plain radiograph, and the report indicates a classic "bamboo spine" appearance and some evidence of sacroiliitis. His sedimentation rate and c-reactive protein levels are elevated. Question What medication would be an appropriate first-line treatment for this patient's condition?

Correct answer: Celecoxib (Celebrex) Explanation This patient's presentation is consistent with a diagnosis of ankylosing spondylitis (AS), a progressive inflammatory arthritis that primarily affects the axial skeleton. NSAIDs are the primary first-line treatment, so celecoxib (Celebrex) would be reasonable. Cyclooxygenase-2 (COX-2) inhibitors, such as celecoxib, have not been necessarily shown to have better efficacy, but are suggested to have fewer gastrointestinal side effects than traditional NSAIDs. Regular use of NSAIDs is recommended. Other potential treatments include medications such as methotrexate, sulfasalazine, and newer "biologics," like infliximab, etanercept, and adalimumab. Acetylcysteine (Mucomyst) is a medication used for acetaminophen overdose. The patient's current symptoms do not suggest acute acetaminophen toxicity. Allopurinol (Aloprim) is a medication used primarily for gout. While gout can cause joint pain, this patient's presentation is not consistent with gout. Cyclobenzaprine (Flexeril) is an older muscle relaxant medication. Although the patient reports "stiffness," his symptoms are due to joint damage, not muscle spasm. This medication would not help his AS. Milnacipran (Savella) is a medication approved for fibromyalgia. While chronic pain is found in both AS and fibromyalgia, none of this patient's diagnostic studies are consistent with fibromyalgia.

ase A 55-year-old woman presents with a 3-week history of dull pain in the left femur. She does not remember any trauma to her leg. After seeing an abnormality on the X-ray, you obtain a bone biopsy and see poorly-to-well-formed cartilage-like cells with a bizarre appearance. Question What is the most likely diagnosis?

Correct answer: Chondrosarcoma Explanation Since there are cartilage-like cells in the biopsy, it is very likely a chondromatous tumor, the widely spread differentiation points to a sarcoma. It predominantly occurs in the pelvis, femur, and shoulder girdle in middle-aged to older adults. It may be primary, arising from cartilage cells, or secondary to a preexisting benign lesion. In osteomalacia (failure to calcify the bone matrix), there would be a decreased amount of well-calcified bone. An old fracture would show unorganized meshwork of woven bone (callus). In Paget's disease, also known as osteitis deformans, there would be dense, well-calcified, erratically shaped bony spicules with abnormal grouped cement lines, which form a mosaic pattern. A bone cyst would show a big, solitary, cystic cavity.

Question A 12-year-old boy presents with left shoulder pain and deformity after a fall from his skateboard while attempting a half-pipe maneuver an hour ago. He heard a "cracking" sound and was unable to continue skateboarding. He has pain with movement of the arm in any direction. On exam, you note tenderness over the superior central shoulder with a palpable deformity and "tenting" of the skin. What is his most likely diagnosis?

Correct answer: Clavicle fracture Explanation With a superior central deformity and tenderness, along with tenting of the skin, the most likely diagnosis is clavicle fracture. AC separation would have deformity laterally on the shoulder. Glenohumeral dislocation would have shoulder contour deformity. Rotator cuff tear does not have acute deformity. Frozen shoulder is a slow, progressive chronic problem and not an acute injury.

Question The patient is a 40-year-old woman who is G5 P4. On ultrasound at 24 weeks, the fetus is diagnosed as small for gestational age (SGA). Normal heart, stomach, situs, and cord vessels are identified. The fetus is identified as having a protruding myelomeningocele during the sonographic examination. During the examination, the sonographer noted reduced movement in the lower limbs. This male fetus has associated malformations that are common with this malformation. What is associated with a lumbar myelomeningocele?

Correct answer: Clubfoot Explanation Clubfoot is often seen with a lumbar myelomeningocele, possibly due to the lack of nerve stimulation to the muscles. The basic deformity is a subluxation at the talocalcaneonavicular joint. Several factors, including family history and intrauterine crowding, have been blamed as the causative factors. It is associated with meningomyelocele, arthrogryposis multiplex congenita, and tibial hemimelia. Initially, conservative therapy is tried; failure paves way for surgical correction. A clubhand could be seen with a higher defect. Spinal defects may or may not lead to reduced movement in the limbs. Amniotic band syndrome is a combination of many disfiguring manifestations, such as congenital distal ring constrictions, intrauterine amputations, and acrosyndactyly, which are the most common manifestations. The basic pathology is the intrauterine entrapment of the fetal parts by the mesodermal fibrous bands of the ruptured amnion.

Question A 16-year-old boy is brought to the Emergency Department by his mother following a fall from a skateboard onto his left outstretched hand (non-dominant) 30 minutes ago. He screamed on the fall and continues to hold his left forearm near his wrist with his right hand. He holds the left arm against his body for support. There is a "dinner fork" deformity, with edema and ecchymosis at the wrist. He is neurovascularly intact, and the skin is closed. With this type of deformity, which of the following fractures is the one most likely in this case?

Correct answer: Colles fracture Explanation The most common fracture of the wrist is the distal radius Colles fracture. This fracture is angulated dorsally. The Smith fracture is the opposite of the Colles in that it is angulated volarly. A Torus fracture is seen in the younger aged patient (3-9) and is just a buckling of the cortex of the injured bone. Salter-Harris fractures (I through VI) are seen in patients with open physes. This patient's distal radius physis is closed by this age. A die-punch fracture is a compression fracture of the articular surface of the distal radius and does not have the angulation deformity of the Colles fracture.

Case An 18-year-old man presents 30 minutes after falling on his outstretched arm while skateboarding. He is guarding his left forearm near his wrist with his right hand, and he has his left arm against his body for support. There is a "dinner fork" deformity with edema and ecchymosis at the wrist. He is neurovascularly intact and the skin is closed. Question With this type of deformity, what fracture is most likely?

Correct answer: Colles' fracture Explanation This patient presents with a Colles' fracture. This fracture causes the "dinner fork" appearance of the wrist; the hand and distal fracture fragment of the distal radius (or distal radius and ulna) are dorsally angulated. In Smith fractures, there is palmar displacement of the distal fragment of radius; this is in contrast to the dorsal displacement seen in Colles' fractures. A torus fracture is the buckling of the cortex of the bones of the wrist (radius or ulna); it usually lacks an angulation deformity. Salter-Harris fractures are seen in growing children. This patient is likely skeletally mature. A die-punch fracture may also be angulated, but it is a compression of the radial articular surface.

Case A 48-year-old woman presents with a 6-day history of lower back pain and right hip pain. The patient has had intermittent bouts of lower back pain that she attributes to heavy weight-lifting. She had a serious episode 6 months ago that required 2 weeks to resolve. Usually, the pain improves with ice and does not need medications. The pain is worse in the morning and with driving. There are no paresthesias or numbness; she does not have pain on coughing or sneezing. She does not have any bladder problems, bowel problems, or gait disturbances. She reports no history of fever, weight loss, neoplasm, or trauma. She only takes ibuprofen. On physical examination, the patient is not obese. She has a normal gait and a full range of motion while in a squatting position. She has no lumbar shift on standing. The sacroiliac joints, greater trochanters, and pubic symphysis are nontender. She has limited lumbar side flexion due to pain. Her reflexes, strength, and sensation of the lower extremity are normal. Results of straight-leg raise are within normal limits. Right hip range of motion is also within normal limits. Costovertebral angle percussion is nontender. There is spasm of the erector spinae (greater on the right). There is absence of a lower abdominal pulsatile mass. Question What is the next step of management?

Correct answer: Conservative therapy with activity modification Explanation The correct response is conservative therapy with activity modification. The vast majority of patients who present with mechanical lower back pain improve spontaneously or with conservative measures. Activity modification, physical therapy, oral analgesics, and educational training fall in this category. Patients should be encouraged to continue with activities of daily living as tolerated. Activity modification helps reduce pain to a tolerable level while maintaining sufficient activity so as not to disrupt activities of daily living. Instructions should be given to avoid heavy lifting or prolonged activity, and patients should change positions frequently to avoid intradiscal pressures. Had the patient been working outside the home, the physician would encourage her to continue her occupational activity. Radiological testing is generally not required in patients presenting with mechanical lower back pain. The Quebec Task Force of Spinal Disorders (QTFSD) recommends radiographic testing in case of neurologic deficits, age greater than 50 years or less than 20 years, fever, trauma, or history of neoplasm. In the setting of an objective evaluation demonstrating neurological compromise or disc disruption, MRI of the spine may be useful in detecting anatomic lesions. A diagnosis based on MRI, in the absence of objective clinical findings, may not be the cause of the patient's pain; studies have shown that nearly 1/3 of asymptomatic individuals will have abnormalities on lumbosacral MRIs. Laboratory testing is typically unnecessary in patients with lower back pain. In the event that the pain lasts longer than 1 month or is associated with systemic signs (e.g., fever, night sweats, and chills), the physician may be prompted to test for evidence of a systemic cause of the pain (e.g., infection or ankylosing spondylitis). An erythrocyte sedimentation rate or a C-reactive protein level may be useful in the setting of an infection; however, the patient denied fever, thereby dismissing the need for lab testing. Bed rest is not indicated in the setting of mechanical lower back pain. Continuing ordinary activities within the limits permitted by pain results in a more rapid recovery than either bed rest or back mobilizing exercises. Electromyography (EMG) can be used to evaluate the local neurological function in an ambiguous presentation. Electromyography is useful for objectively assessing patients with lower back pain with neurological symptoms or signs in the lower limb(s). It helps in ascertaining whether those symptoms/signs are due to lumbosacral radiculopathy versus peripheral polyneuropathy, myopathy, or peripheral nerve entrapment, among other conditions. It may provide information about the nerves, including the root level. It may have some use should there be no clear revelations in the history and physical examination.

Case Police bring in a 28-year-old male prisoner who was in a fight with another man 2 hours prior to examination. During the fight, your patient struck the other man in the mouth; he sustained a laceration over his first and second knuckle of his right hand. A prompt initial inspection reveals a laceration that is 2.5 cm in length; there is no foreign body and the depths of the wound do not appear to involve bone or tendon. There is noted tenderness of the head of the first metacarpal bone, but no crepitus. There is no limitation to passive or active motion. The patient's last tetanus booster was 1 year ago. His physical examination, aside from his hand, is within normal limits. Question What should be the immediate course of action for this patient?

Correct answer: Copiously irrigate the wound. Explanation This patient's injury is sometimes called a clenched fist injury due to the position of the fist when the injury occurs. This injury may also be called a closed fist injury or "fight bite." Although many occurrences of this type of injury do not initially appear severe, the overall consequences of this injury are very deceiving. The possibility of significant infection is an issue due to there being over 42 different organisms that one may be exposed to from a human bite. Another factor that makes this injury serious is that the injury itself does not often bleed heavily and the underlying injury is hidden by soft tissue. Just range of motion of the injured joint could seal the bacteria that may be introduced during the impact; severe complications can range from cellulitis, osteomyelitis, septic arthritis, or even tenosynovitis. Delay in seeking treatment will only worsen these complications. Treatment should begin promptly and consists of an extreme irrigation of copious amounts of sterile water or an antiseptic solution. Typically this is done with a high-pressure stream of water or solution into the wound. A prompt referral to a hand surgeon would then be appropriate. Incision and drainage of the wound may also be considered, and splinting may occur for stabilization after proper evaluation and cleaning. Bite wounds, however, should never be sutured. Intravenous antibiotics may be administered for the first 24 hours. Once the patient has been discharged, they are usually switched to oral antibiotics for 5-7 days. Antibiotic choice is dependent on those used by the hand surgeons in the community. Not treating the patient as described above would very highly likely lead to devastating consequences with significant morbidity. References Raukar NP, Raukar GJ, Savitt DL. Extremity Trauma. In: Knoop KJ, Stack LB, Storrow AB, Thurman R, eds. The Atlas of Emergency Medicine. 4th ed. New York

Case An 11-year-old girl presents with an 8-month history of right hip and thigh pain. The patient's health has been good overall, with the exception of her being diagnosed as obese. You decide to order X-rays because you suspect the patient might have slipped capital femoral epiphysis (SCFE). Question If this patient is diagnosed with SCFE, what should be the initial management and what definitive treatment should follow?

Correct answer: Crutches, then surgical pinning in situ Explanation The correct answer is crutches, then surgical pinning in situ. SCFE is a disorder that occurs when there is separation of the proximal femoral epiphysis through the growth plate. It usually occurs during early adolescence and is more common in males. Occasionally, there can be a metabolic cause, such as thyroid or pituitary diseases; obesity is also a known risk factor. Patients usually present with pain and/or a limp that has been going on for some prolonged period of time. Pain can be referred to the thigh or the medial side of the knee in some cases. Diagnosis is made based on X-ray findings; both anteroposterior and lateral X-rays should be taken. Treatment is important due to the risk of avascular necrosis in about 30% of these patients. When a patient presents with non-acute SCFE, such as in this case, the definitive treatment is to pin the slip. Until this can take place, patients should not bear any weight on the affected side, hence the crutches. 'No crutches, then surgical pinning in situ' is not the correct answer. The surgical correction is the most definitive treatment of a patient with chronic SCFE, but they need to be non-weight bearing until surgery can take place. This limits the long-term effects that the slip can have on the patient. 'No crutches, then traction' is not the correct answer. Crutches are needed in order to keep the patient non-weight bearing until surgery can take place. This limits long-term sequelae. Traction can be an appropriate treatment in patients who have an acute case of SCFE. The patient's SCFE is not likely to be acute given the presence of symptoms for 8 months. 'Crutches, then traction' is not the correct answer. Crutches in the immediate time period are pertinent to limit weight bearing. However, traction is an appropriate treatment in patients who have an acute case of SCFE. The patient's SCFE is not likely to be acute given the presence of symptoms for 8 months. 'SCFE is self-limiting and will resolve within 3-6 months' is not the correct answer. SCFE is not self-limiting, and NOT treating the patient will likely result in avascular necrosis of the affected side. Treatment is imperative in these patients.

Case A 5-year-old girl presents after falling off a shopping cart, tripping, and then falling onto her right arm. On examination, temp is 98.7, pulse 97, respirations 18, blood pressure 127/80 mm Hg. She is alert, oriented, and in no acute distress. Significant findings related to the right arm, which was mildly swollen, deformed, and diffusely tender. There was decreased range of motion of the right elbow due to pain. Sensation was intact. A radiographic examination was performed. Question What is the most common long-term complication associated with the radiology finding?

Correct answer: Cubitus varus Explanation Cubitus varus is an elbow deformity with a decreased carrying angle (so that, with the arm extended at the side and the palm facing forward, the forearm and hand are held at less than 5 degrees). There is deviation of the forearm toward the midline of the body. It is the most common complication of a supracondylar fracture of the elbow and is usually caused by poor reduction of the fracture with medial displacement, internal rotation, and extension of the distal fragment. The distal fragment then tilts into a varus position. The use of AP, lateral, and oblique view X-ray and CT scan to determine the position of the distal fragment and ensure that its rotation is less than 10 degrees is helpful in preventing this. Cubitus varus occurs in various studies ranging from 3 to 57%. It produces a cosmetic deformity, but it is usually of little functional significance. Proximal migration of the fracture fragment as well as nonunion with displacement can lead to a valgus deformity, known as cubitus valgus. Most supracondylar fractures (95%) are extension fractures with posterior displacement of the distal fragment. Flexion fractures (5%), which are much rarer, cause the distal fragment to move forward. In the x-ray, the fracture line runs transversely just above the condyles. 5% of supracondylar fractures are associated with vascular injury. This is more likely with posterolateral displacement of the distal fracture fragment which then displaces the neurovascular bundle over a spike on the metaphysis. This presents as a pulseless pink hand. Less than 1% of children with vascular compromise develop a compartment syndrome due to injury of the brachial artery. This can be masked by a median nerve injury. Contracture occurs as a result of inadequate arterial perfusion and venous stasis. Within 4-6 hours, irreversible muscle necrosis begins. The resulting edema further impairs the circulation. The area with the most degeneration of the muscle in the forearm is the middle third of the muscle belly of the pronator and flexor muscles. Fibrosis and contracture follow. If blood supply does not improve by closed reduction, then open reduction and surgical exploration of the artery is indicated. Nerve injuries occur in 5-19% of these fractures and most often are neurapraxias. In these injuries, nerve continuity is maintained, but there is focal demyelination and a conduction block. In type III fractures up to 50% can show median nerve deficits. The most commonly involved nerve is the anterior interosseous branch of the median nerve, but the radial and ulnar nerves can be involved. Motor function generally recovers occurs in 3-4 months and sensory function in 6 months, both as remyelination occurs. Myositis ossificans is an extremely rare complication of supracondylar fracture in children. Calcification in a hematoma in the brachialis muscle over the anterior aspect of the elbow joint, around the periosteum and the capsular ligaments may all contribute to this new bone formation. Early findings consist of slight warmth with limitation of movement due to muscle spasm. This progresses to a consolidating stage in which a firm lump may be palpated in front of the elbow. In advanced stages, a firm bony lump surrounds the elbow with total loss of movement. Serial radiographs are obtained to distinguish it from osteosarcoma where the calcification extends from center to periphery. Surgery is indicated only if it obstructs movements or impinges on nerves.

Case A 45-year-old man presents to his orthopedic office for a 1-month history of worsening pain in his left groin and buttock. He had a surgical hip fracture repair approximately 4 months ago, 2 days after a fall in a bowling alley. His post-operative course was unremarkable, with normal healing and return to activities. However, he now notes increasing difficulty with walking and has had to limit activities again due to pain. He has tried "all" of the over-the-counter pain medications without relief. His past medical history is unremarkable, aside from the surgical fracture repair. He has no chronic medical conditions, no other surgeries, and takes no chronic medications. He has no medication allergies. On physical exam, the patient is asked to walk across the room. He limps and appears to be in pain. The entire hip region is evaluated and no erythema, ecchymosis, or edema is noted. The incision site is healing well, with no tenderness and no fluctuant tissue. With range of motion tests, the patient reports pain, especially with internal rotation of the left hip. He is sent for several tests. Blood culture is negative. X-ray shows a crescent sign over the left femoral head. An MRI shows a low-signal intensity on the left femoral head, and increased uptake is seen also in this region with nuclear bone scan. The fracture site shows normal post-operative changes. Question What best describes the underlying mechanism responsible for this patient's complication from his hip fracture?

Correct answer: Damaged vascular supply to femoral head Explanation This patient's condition is suspicious for avascular necrosis (or the more preferred term, osteonecrosis) of the hip. Avascular necrosis (AVN) is a relatively common complication following a traumatic hip dislocation and/or fracture and results in damaged vascular supply to the femoral head, with subsequent bone death. Some potential conditions on the differential may include arthritis, malignancy, and osteomyelitis. The history and physical exam may be similar in all these conditions, but the history of hip fracture and the diagnostic studies support a diagnosis of AVN. Definitive treatment is surgical, with a variety of approaches, from core decompression to bone grafting. Autoimmune destruction of synovial membrane is one of the mechanisms associated with rheumatoid arthritis. Damage to bone marrow, resulting in decreased white blood cell immunity, is not a process specific to one condition. Because only a minimal portion of one bone is affected, this patient's marrow production of red and white blood cells would be expected to be normal. Development of traumatic arthritis is another potential complication following hip fracture and/or dislocation. However, this patient's current findings show bone death, not joint inflammation. He could still be at risk for developing post-traumatic arthritis, but this is not the current explanation of his symptoms.

Question A 47-year-old grossly obese woman presents with left non-dominant shoulder pain and limited motion. The pain began about a month ago, and her shoulder has progressively lost motion during that time until now she cannot reach overhead with that arm. There is no history of trauma; it does not wake her at night; and she can sleep on the left side. Her past medical history is significant for Diabetes Type I and hypothyroidism. She is on regular insulin and NPH insulin, as well as levothyroxine (Synthroid). What would you expect to find on exam?

Correct answer: Decrease in active and passive range of motion (by at least 50%) Explanation This is adhesive capsulitis (frozen shoulder). Diabetes and hypothyroidism are common risk factors. Pain usually precedes the progressive stiffness. Age is commonly between 40 and 60 years old. Loss of both active and passive range of motion by at least 50% is the hallmark of this disorder. Motion is painful, especially at the extremes. Tenderness is common at the deltoid insertion and not over the SC joint. The rotator cuff muscles are not as weak with isometric testing, as if the rotator cuff muscle-tendon were inflamed. This is a disorder of the glenohumeral joint capsule and not the rotator cuff muscles. There may be some weakness due to atrophy from non-use because of the pain and stiffness.

Case A healthy 37-year-old woman sustained a hairline fracture of the left tibia. Her leg is put in a cast and immobilized. Six weeks later, the cast is removed from her leg. A repeat X-ray shows healing of the fracture line. Question What histological changes would be most likely to be observed in this patient's gastrocnemius muscle?

Correct answer: Decrease in mitochondrial enzyme activity Explanation Due to long-term restriction of activity, the gastrocnemius muscle is expected to undergo disuse atrophy. The muscle fibers decrease in size and there is a decrease in the number of myofibrils. Histologically, there is a decrease in mitochondrial enzyme activity, signaling a change in the muscle's oxidative capacity. There is no increase in fibrous tissue or fat in inactive muscles. Calcium changes affect muscle contractility. Changes in calcium levels are not observed in muscle wasting. Satellite cells are normally quiescent cells in skeletal muscle that become active in regeneration and hypertrophy. This is not observed with deconditioning due to inactivity.

Question An 80-year-old woman presents with a history of a fall. She is unable to bear any weight on her left leg and is in considerable pain. An X-ray of the hip reveals a fracture in the neck of the femur and osteoporosis of the bone. What puts a patient at risk for common forms of osteoporosis?

Correct answer: Deficient calcium intake Explanation Refer to the table. Osteoporosisis a general term for increased porosity of the skeleton resulting in reduction of bone mass. The common primary forms of this disease are senile and postmenopausal osteoporosis. Peak bone mass is achieved in young adulthood. It is determined largely by Hereditary factors Physical activity Muscle strength Diet Hormonal states Age-related bone loss which may average around 0.7% is a normal, predictable biological phenomenon. Post-menopausal osteoporosis is characterized by a hormone dependent acceleration of bone loss. Women lose as much as 35% of cortical bone and 50% of their trabecular bone. Therefore, it is of no surprise that 1 out of 2 post-menopausal women suffer from an osteoporotic fracture, whereas 1 in 40 men do. ↓Estrogen ↓ ↑ Interleukin-1 from monocytes ↓ Most potent known stimulator of osteoclast recruitment and activity ↓ Compensatory osteoblastic activity (but it does not keep pace) → HIGH TURNOVER form of osteoporosis. As one grows older, the skeleton is populated by osteoblasts with diminished capacity to make bone resulting in LOW TURNOVER OSTEOPOROSIS Biopsy Single energy absorptiometry Dual energy absorptiometry Quantitative CT scan After young adulthood, bone resorption tends to exceed new bone formation; as a result, bone mass gradually decreases as one ages. The osteoblasts of the aged do not divide or synthesize matrix as vigorously as those of the young. The osteoblasts and osteoprogenitor cells also do not respond to growth factors as well in the aged as they do in the young. Experiments have consistently shown that exercise, particularly resistance-type exercise, can increase bone mass; immobility increases the rate at which bone is resorbed. Low calcium intake during periods when the bone is growing rapidly can reduce the maximum bone mass an individual ever attains. Patients who only attain a relatively small bone mass are at greater risk for osteoporosis. Estrogen deficiency is a major factor in osteoporosis; this is demonstrated by the fact that a much larger percentage of women suffer from osteoporosis than men, and that estrogen supplementation early in menopause can protect women from this disease. High levels of parathormone and/or low levels of vitamin D have not been shown to be important etiologic factors in the common forms of osteoporosis.

Case A 43-year-old woman presents with a reddish-pink eruption on her neck, upper back, shoulders, and arms. The skin lesions appear annular and do not hurt or itch. The patient is concerned about having a contagious disease, but does not feel sick. Her body temperature is 38.4°C, her pulse is 65, and her blood pressure is 150/70. She has a history of hypertension and gastritis, which she has been treating for several years. She exercises regularly, mostly outdoors, and uses sunscreen whenever she feels it is needed. Question What is the most appropriate next step in the management of this patient?

Correct answer: Determine the medication history of the patient Explanation The correct answer is to determine the medication history of the patient,because the patient has a history of hypertension and gastritis, and presents annular, not-itching, and not-painful skin lesions. The lesions described are typical of subacute cutaneous lupus erythematosus (SCLE), a condition that can be induced by certain antihypertensive agents or some proton pump inhibitors used to treat gastritis. The answer of request anti-Ro/SSA and anti-La/SSB serological test is incorrect, because this screening test is very specialized and should be requested only when there is a justified reason to suspect diagnosis of systemic or cutaneous lupus disease. The answer prescribe a topical antibiotic cream is incorrect, because even though the patient presents with a slightly elevated body temperature, she does not present with symptoms of skin infection. Her rash does not cause burning, pain, swelling, or itching, typical signs of a bacterial infection. The answer of request her immunization records is incorrect, because there is no reason to suspect a viral infection. The answer instruct patients about sun-avoidance techniques is incorrect, because even though it is necessary for the patient's education, it should be done only after a making a diagnostic decision.

Case A 3-year-old boy is brought to the pediatrician after his mother noticed that he has been "walking abnormally." She also says that the boy gets tired easily and rarely goes out to play. He has always had a tendency to fall often. On examination, the calf muscles are enlarged. The boy walks with an awkward gait on his toes. Gower's sign is positive. Lab studies reveal elevated creatine phosphokinase levels. You order genetic testing and muscle biopsy; they show a mutation in the dystrophin gene and absence of dystrophin, respectively. Question What is the most probable diagnosis?

Correct answer: Duchenne muscular dystrophy Explanation The correct answer is Duchenne muscular dystrophy (DMD). DMD is an X-linked recessive progressive muscular dystrophy characterized by proximal muscle weakness and wasting of voluntary muscles. Pseudodystrophy results in enlargement of calf and deltoid muscles. The symptoms are evident by 3 to 7 years of age. Being X-linked, boys are more often affected, and females are usually carriers. One-third of DMDs are due to a new mutation, and 10% of these are due to gonadal mosaicism. DMD is caused by a mutation in the dystrophin gene at locus Xp21. Diagnosis is by elevated creatine phosphokinase levels, absent dystrophin on muscle biopsy, and mutation detected by genetic testing. Becker muscular dystrophy is similar to DMD, except that the diagnosis is usually made in adulthood and progresses at a slower rate. Dystrophin is usually present, but in a lesser percentage than normal. Limb girdle dystrophy affects the shoulder and hip muscles and is characterized by slowly progressive muscular weakness. Congenital dystrophy is present at birth and is characterized by slowly progressive muscular weakness and joint deformities. Myotonic dystrophy can occur at any age and is characterized by muscular dystrophy, along with other findings such as cataracts, myotonia, etc. The mutation in the dystrophin gene and absence of dystrophin in the muscle favor a diagnosis of DMD and not the other dystrophies. References References:

Case The mother of a 3-year-old boy brings her child to the family doctor because the child was slow to begin walking, has difficulty in walking and standing, and fatigues easily. She recalls that the youngest brother of her mother had similar problems and died in his late teens. Tests of the child reveal creatinuria and elevated serum levels of creatine phosphokinase (CPK). A histological specimen taken from a muscle biopsy from this patient is shown in the image. Question Ico-delete Highlights What is the most likely diagnosis?

Correct answer: Duchenne muscular dystrophy Explanation This child is suffering from Duchenne muscular dystrophy (DMD). This X-linked condition is maternally transmitted to male children only. It usually manifests in the first decade of life as difficulty in standing and walking. Muscle weakness initially appears in the muscles of the hip girdle and upper legs. Later, the condition involves muscles of the arms and respiratory system, with death usually occurring before age 20. Patients fail to produce the protein dystrophin. This is a member of the spectrin superfamily, with a MW = 427,000. Dystrophin is found underneath the sarcolemma in normal muscle, especially where membrane folds are formed, e.g., at myotendinous and neuromuscular junctions. It also interacts with actin filaments, stabilizing their side-side association as well as binding to the amino-terminus. Muscle from DMD patients shows myocytes of variable diameter (compared to the diameters of b and c) and unusual infiltration of leukocytes (at a). Physical symptoms of DMS occur most often before age 6 and include fatigue, learning difficulties and muscle weakness. It begins most commonly in the legs and pelvis area with progressive difficulty walking. Muscles of DMD patients are more fragile and more easily ruptured. In addition, DMD patients show reduced levels of enzymes normally found in the sarcoplasm with concomitant increases in serum levels of creatine phosphokinase (CPK), lactate dehydrogenase, and glucose phosphate isomerase. Urinary levels of creatine are usually elevated. One of the normal functions of dystrophin is stabilization of the sarcolemma against the physical stresses caused by frequent folding during cycles of contraction and relaxation.Myosin and myoglobin levels would be reduced due to loss on sarcomeres. Lysosomal hydrolases would be elevated because of increased activity from phagocytic cells. Becker muscular dystrophy is similar to DMD in its genetics and clinical course, although it is less common and less severe. In such instances, dystrophin is formed in lower quantities and has a slightly lower molecular weight than normal dystrophin. Nemaline myopathy is usually caused by autosomal recessive mutations that lead to accumulations of rod-shaped bodies just beneath the membrane of muscle cells. It probably represents mutations in the protein α-actinin, leading to bundling of Z-line proteins. Myasthenia gravis is an autoimmune disease wherein patients develop antibodies to the acetylcholine receptor of the neuromuscular junction. Routine histological biopsies of muscle tissue are apparently normal. It does not usually present in young children, but when seen in patients under 40, it is more common in women than in men. Mitochondrial myopathy usually presents in young adults initially as proximal muscle weakness, especially in the oculomotor muscles. The disease is commonly characterized by peripheral accumulations of mitochondria (demonstrable only by special stains) in muscle cells.

Case You are evaluating a 12-year-old boy who presented to the emergency department 3 days ago with a right ankle injury. He was running out for a pass in a pickup football game when he "rolled over" his ankle. Radiographs were negative. He has been treating with rest, icing, compression, and elevation (RICE). Past medical history is unremarkable, without prior ankle injuries. Today, he has mild pain and swelling of the outer aspect of the right ankle near the lateral malleolus with mild tenderness on motion, especially inversion, with a mild limitation of motion due to pain. He can almost fully bear weight on the ankle. There is no joint instability. Question What is your next step in managing this injury?

Correct answer: Early mobilization Explanation Ankle sprains are among the most common sports injuries, with inversion injuries resulting in lateral sprains being the most common type. The above represents a grade I sprain since there is no joint instability. After an initial period of rest, icing, compression, and elevation, if the ankle is stable, mobilization with range of motion exercises progressing to ankle strengthening should begin as soon as possible in order to avoid atrophy and joint stiffness. Protected weight-bearing may occur by using crutches and an ankle stabilizer. Even if this patient had a radiographically undetected nondisplaced Salter Harris I fracture of the distal fibula detectable on MRI, which is relatively uncommon, treatment would not change. Therefore, MRIs are rarely needed in the management of a simple ankle sprain unless symptoms do not improve with treatment. Hard casting would not be used for a grade I sprain and would impair mobilization. Hard casting immediately after an injury is contraindicated, as swelling may cause compartment syndrome. An opiate would not be an appropriate choice for mild pain; further, NSAIDs have anti-inflammatory as well as analgesic properties and can reduce swelling, whereas opiates cannot. A rigid brace would not be appropriate for a grade I sprain and would hinder mobilization. References

Case A 14-year-old boy is seen for a sports physical for the freshman basketball team. Past history is significant for a high degree of myopia bilaterally, first diagnosed at age 4 years, and a dislocated shoulder at age 10 years that was easily reduced. Family history is significant for several unidentified ancestors having died in their forties of an unidentified cardiovascular disorder. Physical examination revealed normal vital signs. Height is 6' 1" and weight 145 lbs. The upper to lower segment ratio is 0.65 (decreased). Arm span was 76". The palate is highly arched and mild pectus excavatum is present. A 2/6 early diastolic murmur is present and best heard at the second intercostal space at the right sternal border. Arachnodactyly of the fingers and toes and generalized loose jointedness and pes planus are also present. Question As to whether he should be permitted to play basketball, what should be advised?

Correct answer: Echocardiography needs to be performed, with follow-up thereafter Explanation The presentation is highly suggestive of Marfan syndrome, with skeletal findings (decreased upper/lower segment ratio, dolichostenomelia, arachnodactyly, pectus deformity, pes planus, and hyperextensible joints), ocular manifestations (here, high myopia, but the most characteristic finding is ectopia lentis), and a suggestive family history of early cardiovascular death and cardiac findings in the patient. The most important factor in determining this young man's fitness for competitive athletics is identification of the nature and extent of any cardiac involvement (e.g., mitral valve prolapse, aortic regurgitation, dilated aortic root). While elucidating family history would be helpful regarding the diagnosis of Marfan syndrome, it will not identify the cardiac abnormality (if there is one) in this patient. A prior, nonrecurrent shoulder dislocation is not a contraindication to athletic participation. Heart murmurs can be innocent or indicative of mild, moderate, or severe cardiac (or noncardiac) conditions. In and of themselves, they are not a contraindication to athletic participation. The cause must first be determined. One cannot decide upon the appropriateness or not of athletic participation until a cardiac evaluation is performed.

Question A 43-year-old woman who works as a secretary and spends most of her day typing on the computer has symptoms of carpal tunnel syndrome. What diagnostic test is most useful in confirming your diagnosis?

Correct answer: Electromyogram Explanation Electrophysiologic (electromyogram) testing is the most useful confirmatory test for carpal tunnel syndrome (CTS), but these tests must be used with caution. Some patients have no clinical signs or symptoms but will have an abnormal EMG; as many as 5-10% of patients with CTS will have normal results. In general, the diagnosis of CTS is based on your H&P, with EMG use to confirm your suspicion. A positive Phalen's and Tinel's sign would demonstrate paresthesia in a median nerve distribution, a positive Tinel's sign via percussion of the median nerve at the wrist, and positive Phalen's after passive flexion of the hand at the wrist. They are used to help in diagnosis but are not very sensitive or specific tests. Electrodiagnostic studies are often needed to confirm carpal tunnel syndrome. CT is used more often for boney pathology of the wrist. MRI may show swelling within the carpal tunnel but is not diagnostic. Plain films should be taken if there is limited range of motion of the wrist but is not diagnostic of CTS. ESR may be slightly elevated with CTS but is not specific for it.

Case A 65-year-old woman presents with morning stiffness and pain in her shoulders. She reports that she has lost a few pounds over the past month and has had intermittent fevers. The physical examination reveals tenderness over her deltoids with reduced shoulder abduction. Laboratory results reveal elevated erythrocyte sedimentation rate (ESR) value. Question What additional laboratory findings would be expected in this patient?

Correct answer: Elevated C-reactive protein Explanation The correct response is an elevated C-reactive protein. The clinical presentation is suggestive of polymyalgia rheumatica; patients usually present with pain and stiffness of the pectoral and pelvic girdles. They may also have constitutional symptoms, such as weight loss and fever. On examination, the affected shoulder or thigh muscles are tender, and there is a painful restriction of hip and shoulder movements. Laboratory results in these patients usually reveal an elevated erythrocyte sedimentation rate (ESR) and elevated C-reactive protein. The serum creatine kinase is within normal limits, and a normochromic normocytic anemia may be present, consistent with anemia of chronic disease. Microcytic microchromic anemia is more consistent with the diagnosis of iron deficiency anemia. Electromyography is normal, and a muscle biopsy, if done, is also normal. Management is with typically with corticosteroids.

Case A 26-year-old man, with no significant past medical history, presents to the emergency department following a motor vehicle accident that occurred 2 hours ago. He was seated in the front passenger space and seatbelt-restrained. He states that his right lower extremity struck the dashboard during the impact. Although the pain in the right lower extremity was initially tolerable, it has rapidly intensified over the past 30 minutes. The orally-administered morphine no longer is helping to reduce his pain; he also notes a burning dysesthesia to the area. His physical examination reveals severe tenderness upon passive ranges of motion, swelling, firmness, tenderness to palpation and squeezing of the affected area. The skin is warm, and distal pulses are intact bilaterally. Question What is the most appropriate intervention for this patient at this time?

Correct answer: Emergent surgical fasciotomy Explanation This patient's diagnosis is consistent with compartment syndrome of the affected extremity; this occurs when increased pressure within a limited space compromises the circulation and function of the tissues within that space. Muscle death and nerve damage in the setting of compartment syndrome are caused by prolonged elevation of tissue pressures; this can result from external forces (e.g., a cast or tight dressing that compress a compartment). It can also result from an increase in the volume of a compartment that exceeds the limits of the surrounding fascia's ability to stretch. This may be the result of hemorrhage into a compartment or edema caused by reperfusion injury. A fasciotomy should be performed as soon as the diagnosis is clinically confirmed, or after the determination of elevated tissue pressure in relation to the diastolic blood pressure. Surgical fasciotomy reduces intracompartmental pressure; long incisions of the skin and fascia are made to allow tissue swelling to occur without a concordant elevation of tissue pressure. Medical management should be initiated while the surgical procedure is being arranged; this includes supplemental oxygen and supporting the blood pressure in the hypotensive patient. Constrictive casts or dressings should be removed. The affected limb should be placed at the level of the heart; elevation higher than the heart reduces the arteriovenous pressure gradient. A lower extremity sonogram is not warranted at this time since a vascular occlusion is not suggested in this patient. The application of topical analgesia or alterations of analgesic interventions should not take priority above the definitive treatment of compartment syndrome.

Case A 67-year-old presents with a 2-year history of numbness in the lower extremities. The most relevant complaint is gait impairment and instability. Neurologic examination shows a disturbed sense of position with preserved tactile and temperature sense, normal muscular force, and normal reflexes. Electrophysiologic evaluation excludes a sensory polyneuropathy Question What is the most likely cause of the patient's symptoms?

Correct answer: Ependymoma Explanation The choices listed are all intrinsic cord tumors, the most common of which is the correct answer. This patient has an ependymoma. Ependymomas make up approximately 60% of intramedullary spinal cord tumors. They are relatively slow-growing and may develop anywhere along the entire length of the spinal cord. However, a large percentage of these tumors are found at the lower sections of the cord. They develop from ependymal cells and can be seen in all age groups; however, the most common age group is between 20 and 40 years. Men and women are affected equally. Astrocytomas are the 2nd most common type of spinal cord tumor. They are found more often in children than in adults. Other rare types include developmental tumors, oligodendogliomas, lipomas, and hemangioblastomas. The usual presentation is pain, and classically it is aggravated in supine or flat position. As the tumor advances, abnormal sensation and eventually weakness follow, until finally the normal bladder and bowel function are lost. Surgery is the treatment of choice. Radiation therapy is sometimes given if the total surgical removal of the tumor is not possible or if the tumor appears to grow rapidly. References References:

Case A 51-year-old-woman presents due to menstrual irregularity, hot flashes, and mood changes. Physical examination reveals an atrophic vagina and breasts that have decreased in size. You explain that those symptoms and signs are physiologic and related to her age. The patient decides not to start hormonal replacement therapy (HRT). Question 8 years later a dual-energy X-ray absorptiometry (DEXA) reveals a T score of -3, reflecting a very low bone density. Change in the serum level of what hormone or electrolyte is most likely responsible for the alterations detected by the DEXA scan?

Correct answer: Estrogen Explanation Estrogen level fall around and after menopause due to age-linked decline in number of ovarian follicles. This permanent loss of estrogen may result in the increase of bone re-absorption named osteoporosis. Normally, estrogen inhibits apoptosis in osteoblasts (bone-forming cells) and induces apoptosis in osteoclasts (bone-resorbing cells). Therefore, the absence of this hormone will trigger bone reabsorption, leading to loss of bone mass and interconnections in trabecular bone. Levels of calcium (Ca++), cholecalciferol (vitamin D), and parathyroid hormone (PTH) are normal in menopause-related osteoporosis. A complex of Ca++ and phosphate (PO43−) compose the inorganic matrix of bone. Vitamin D is one of the fat soluble vitamins, along with vitamins A, E, and K. The active form of vitamin D, 1,25 (OH)2D3, facilitates Ca++ absorption from the intestine, bone growth, and mineralization. Vitamin D is ingested in the diet or manufactured in the skin through ultraviolet light activation. The conversion of the inactive metabolite 25(OH)D3 to the active metabolite 1,25(OH)2D3 occurs is the kidney. PTH increases the release of Ca++ and PO43− from bone to blood. In the kidney, it increases reabsorption of Ca++ while decreases reabsorption of PO43−, and stimulate the conversion of 25(OH)D3 (inactive) to 1,25(OH)2D3 (active). Like the estrogen level, progesterone levels also fall after the menopause. However, the decrease in progesterone is not responsible for the signs and symptoms of menopause; it is also not responsible for the signs and symptoms of osteoporosis.

Case An 89-year-old Caucasian man is brought in by his daughter due to pain in his left shin. He bumped his leg on a coffee table about 3 weeks ago; he developed some mild discomfort, bruising, and a small gash in the skin. It had seemed as if his condition was healing well, but his condition has worsened over the past few days. Now he is moaning due to pain, and he says it hurts to walk on the leg. He describes the pain as "horrible" and an 8 on a 1 - 10 pain scale. The daughter reports the pain keeps him up at night and is unresponsive to ibuprofen and narcotic pain pills. He denies fever and chills. On physical exam, the patient is in obvious pain and is assisted to the exam table with limited weight bearing on his left leg. The lower extremities are examined; significant findings include healing and a scabbed lesion of approximately 3 cm in length across mid-tibia, with surrounding erythema and edema. Tenderness is elicited along the shin, extending well past the area of erythema. Homan's sign is negative. Distal pulses, temperature, coloration, knee range of motion, and lower extremity reflexes are symmetric and normal. Right lower extremity is normal. Question What test finding would be most diagnostic for this patient's likely condition?

Correct answer: Evidence of tibial edema and necrosis on magnetic resonance imaging (MRI) Explanation Evidence of tibial edema and necrosis on magnetic resonance imaging (MRI) is the most diagnostic for this patient's condition. Along with this patient's history and physical, this MRI finding indicates bone involvement and osteomyelitis, a condition that can develop in the elderly after trauma. Staphylococcus aureus is a common cause, but biopsy reveals the causal organism(s). Osteomyelitis can be acute or chronic, and it can affect all ages of patients. Certain groups, such as diabetics with foot ulcers and peripheral vascular disease or intravenous drug users, may be especially at risk. Diagnosis is based upon history, physical, and a combination of diagnostic tests. Elevated C-reactive protein and erythrocyte sedimentation rate both indicate inflammation in the body. However, these tests do not point to the specific etiology of the inflammation. An elevated white blood cell count (WBC) likewise indicates infection and/or inflammation, but it does not help differentiate cellulitis versus osteomyelitis. Plain radiography (X-rays) is often recommended early in the workup of a suspected case of osteomyelitis, both because of low cost and widespread availability. If positive for bone involvement, a diagnosis can be made. However, most of the time, especially in early osteomyelitis, the plain films are non-specific and may simply show soft tissue swelling. Further testing would be recommended.

ase A 22-year-old Asian man with no significant past medical history presents with a dull pain. It is insidious in onset and has been felt deep in the lower lumbar gluteal region for the past 5 months. It is accompanied by low-back morning stiffness of up to a few hours' duration. The stiffness improves with activity and returns following inactivity. For the past 2 months, the pain has ascended; it has become more persistent and bilateral. He notes that nocturnal exacerbation of pain forces him to rise and move around. He also admits to bilateral hip and ankle pain, anorexia, malaise, and an intermittent low-grade fever. His physical exam reveals a stooped forward-flexed position, limitation of anterior and lateral flexion and extension of the lumbar spine, a positive Schober test, restricted ranges of motion of his hips, and reduced chest expansion. A lower back and hip X-ray is performed. It yields the attached image. Question What additional clinical manifestation would be most likely in this patient?

Correct answer: Eye pain and conjunctival injection Explanation The correct response is eye pain and conjunctival injection. This patient's most likely diagnosis is ankylosing spondylitis (AS). The most common extra-articular manifestation is acute anterior uveitis, which occurs in 40% of patients and can antedate the spondylitis. Attacks are typically unilateral, causing pain, photophobia, and increased lacrimation. These tend to recur, often in the opposite eye. Cataracts and secondary glaucoma are not uncommon sequelae. Up to 60% of patients have inflammation in the colon or ileum. This is usually asymptomatic, but frank IBD occurs in 5 - 10% of patients with AS. Hematuria and dysuria suggest a genitourinary pathology. A positive straight leg raise test indicates a likelihood of her herniated lumbar disc. Subcutaneous nodule formation in the setting of joint pain and stiffness suggest rheumatoid arthritis. This patient however is diagnosed with a spondyloarthropathy. Peripheral enthesitis occurs in approximately 33% of patients. These lesions are painful and tender upon examination and may be associated with swelling of the tendon or ligament insertion. Warmth of joints is not typical. Additionally, the most common and characteristic peripheral sites of enthesitis are the insertion of the Achilles tendon on the calcaneus and the insertion of the plantar fascia on the calcaneus. Warm and swollen joints are more typical of rheumatic, crystal-induced arthropathy or joint infection.

Case A 34-year-old man was the driver in a single car motor vehicle accident. Preliminary radiologic studies show a comminuted fracture of the right tibia. Currently, the patient is describing a substantially increasing amount of pain felt in the injured extremity. He describes the pain as being a 10/10; it seems as if it is becoming worse with each passing minute. With anguish on his face, he describes it as a deep, achy, burning pain. You quickly examine the right leg; you note pallor, a tense 'wood-like' feeling of the extremity, diminished sensation, and muscle weakness. Question Given the most likely diagnosis, what would be the most appropriate clinical intervention at this time?

Correct answer: Fasciotomy Explanation The correct response is fasciotomy. This patient is suffering from acute compartment syndrome. Compartment syndrome should always be considered an urgent and extremely emergent situation due to the consequences that can ensue. Acute compartment syndrome (ACS) most often develops after the patient has suffered some type of significant trauma and especially involving a long bone fracture. Fractures account for more than 75% of cases of ACS. More often, ACS is seen in patients under the age of 35 years of age; it also has a higher incidence in young men, especially after they experience fracture of the tibial diaphysis or distal radius; risks of developing ACS is even more increased if the patient suffers from a comminuted fracture. Our patient described above fits the scenarios that show a higher risk of developing ACS. Many times the signs and symptoms of ACS occur in a stepwise fashion; an important clue is a rapid progression of symptoms and signs in a short amount of time. Symptoms include pain out of proportion to the apparent injury, a persistent deep ache or burning pain, or even paresthesias. Signs that would be evident include pain with passive stretch of muscles in the affected compartment, a tense compartment with a firm 'wood-like' feeling, pallor, diminished sensation, muscle weakness and a late finding of paralysis. Adequate decompressive fasciotomy is the procedure that must be performed. Just prior to surgery the affected leg in the patient above should be immobilized with the ankle in slight plantar flexion; doing so helps decrease the deep compartment pressure and does not cause increased pressure in the anterior compartment. Urgent treatment of compartment syndrome can cause is a necessity to avoid the detrimental consequences that will occur as a result of this pathology. Such complications include permanent loss of function. Observation only is the inappropriate choice for this reason. All bandages and casts must be removed to prevent worsening of the ACS, therefore placement of a cast is incorrect; it has been estimated that complete removal of a cast will lead to a decrease of 85% in pressure from the patient's baseline. Both intravenous heparin and/or antibiotic therapy are inappropriate options at this time.

Case A 32-year-old Caucasian woman presents with pain, stiffness, fatigue, and swelling of the fingers. These symptoms have been present over the last 6 months. Over the course of several visits, you rule out any neuromuscular abnormalities. Laboratory work includes complete blood count to rule out infectious diseases. Erythrocyte sedimentation rate is mildly elevated. Thyroid function studies are normal. On physical exam, you note bilateral tenderness to palpation of more than 12 points, including the upper edge of the trapezii; neck muscle insertion at the occiput deltoids, infraspinatus, and second costochondral junction; inferior to the lateral (elbow) epicondyles, trochanters, and lower lumbar area; medial fat pads of the knees; and the medial and lateral insertions of the Achilles tendons. Question What is the most likely diagnosis?

Correct answer: Fibromyalgia Explanation Fibromyalgia is a soft tissue, non-articular pain disorder characterized by chronic, generalized musculoskeletal aches, pains, and stiffness that occur primarily in muscles and their attachments. It is associated with specific sites of exaggerated tenderness. Palpation of the upper edge of the trapezii; neck muscle insertion at the occiput, deltoids, infraspinatus, and second costochondral junction; inferior to the lateral (elbow) epicondyles, trochanters, and lower lumbar area; medial fat pads of the knees; and the medial and lateral insertions of the Achilles tendons reveal 12 or more tender points. The first treatment is patient education. The client will feel better just knowing they have a diagnosable syndrome that can be managed. The use of antidepressant drugs can help. Drugs like amitriptyline, fluoxetine, chlorpromazine, or cyclobenzaprine are used. Exercise is another important aspect of treatment. A healthy diet can also help the client feel in control of their condition. Corticosteroids and opioids should not be used and are not helpful. Cellulitis is the invasion of the skin, its appendages, and/or subcutaneous layer by way of thermal, mechanical, chemical, or physiologic injury and the colonization or infection with pathogenic bacteria of both aerobe and anaerobic nature. Lesions are usually due to immune system response, and purulence is present. Gout occurs most commonly in men (95%). Presentation is usually acute with a swollen, red, big toe. Women may develop gouty arthritis in the perimenopausal phase as estrogen levels drop. In many cases, if the disorder is left untreated, urate crystals are deposited, usually in and around the joints of the extremities. The needle-shaped monosodium urate (MSU) crystals can be deposited in the parenchyma of organs such as the kidney. In some clients, deposits of uric acid form within the urinary collecting tubules. Polymyalgia Rheumatica symptoms persist for more than 2 weeks and include morning stiffness lasting more than 2 hours of the proximal limb muscles, shoulder, and hip girdles. Reiter syndrome, also called reactive arthritis (ReA), is not consistent with other spondyloarthropathies. It is a triad of arthritis, nongonococcal urethritis, and conjunctivitis. The syndrome characteristically is an acute, asymmetric arthritis of the lower extremities that may involve the sub talus or present a dactylitis (sausage digit). It may be isolated to joints or include systemic illness, characterized by fatigue, fever, and weight loss. Classic symptoms accompanying, or intercurrent manifestations, involve the following 3 systems: Mucutaneous disease ranging from painless oral or genital ulceration to a keratoderma blenorrhagicum, a psoriatic skin lesion typically on the palms and soles. Urethritis or cervicitis (including prostatitis and salpingitis) Ocular manifestations ranging from sporadic conjunctivitis to debilitating uveitis Myalgia, joint stiffness, and low back ache. Most cases are positive for HLA B27, except in HIV patients.

Case A 42-year-old woman presents with a history of chronic fatigue, pain around her neck, shoulders, and lower back. She is also experiencing chronic headaches and irritable bowel symptoms. Upon physical exam, no abnormal findings were found, with the exception of trigger points that produced pain around areas such as the trapezius, lateral epicondyle of her elbow, and pain around the medial fat pad of her knee. Laboratory findings showed a normal ESR, negative RF factor, and a negative ANA. Question What is the most likely explanation of the findings?

Correct answer: Fibromyalgia Explanation The clinical picture is suggestive of fibromyalgia. This patient presents with symptoms that are found in patients with fibromyalgia, such as common trigger points noted in her physical exam. Additionally, lab findings help rule out other options, as patients with fibromyalgia have normal lab findings. Osteoporosis is not the correct answer, because the patient does not have a T score of less than -2.5. Rheumatoid arthritis is not the correct answer, because the patient has a negative RF factor. Systemic Lupus Erythematous is not the correct answer, because the patient has a negative ANA. Polymyalgia rheumatica is not the correct answer, because the patient has a normal ESR

Question Ico-delete Highlights If the musculocutaneous nerve were to become damaged, what function would be lost?

Correct answer: Forearm flexion Explanation The major muscles innervated by the musculocutaneous nerve are the brachialis and biceps brachii. Therefore, forearm flexion would be lost. Forearm extension is controlled by the triceps brachii which is innervated by the radial nerve. The wrist flexors are innervated mostly by the median and ulnar nerves. Wrist extensors are innervated by the radial nerve. The pronators (pronator quadratus and pronator teres) are innervated by the anterior interosseous and median nerves

Case A 28-year-old man presents at an urgent care clinic first thing in the morning. He notes a left ankle sprain. The patient describes walking in the woods the day before while wearing flip-flops; he inadvertently stepped into a hidden hole. The patient immediately noticed pain with weight bearing on the left ankle, making it mildly difficult for him to walk out of the woods. Ambulation is only slightly difficult at this time. He describes it as a 6 - 7/10 on a 1 - 10 pain scale. He has noted mild swelling and some tenderness to touch. He has treated this with elevation, ice for 20 minutes at a time, and ibuprofen 200 mg every 8 hours. Physical examination reveals a moderate degree of ecchymosis of the left ankle, with mild mechanical instability, and slight restriction of range of motion. Question Based on the history and physical examination findings, how would you classify this ankle sprain based on the traditional ankle sprain grading system?

Correct answer: Grade II Explanation The scenario above is describing the Grade II ankle sprain. This injury typically involves an incomplete tear of a ligament. Patients will experience moderate pain, swelling, tenderness and ecchymosis. There will be mild to moderate joint instability during exam and some restriction of the range of motion as well as loss of function. Ambulation and weight bearing are painful. Grade I ankle sprain results from only mild stretching of a ligament with potentially microscopic tears. Patients will present with mild swelling and tenderness; there will not be evidence of joint instability, and the patient will be able to bear weight and ambulate fairly easily. Grade III ankle sprain involves a complete tear of a ligament. There will be severe pain, swelling, tenderness, and ecchymosis. Significant instability will be seen on exam, as well as loss of function, with the inability to bear weight or ambulate whatsoever. Grades 0 or IV do not exist in the common grading system of ankle sprains.

Case A 61-year-old Caucasian man presents with upper extremity pain. He describes the pain as coming on gradually, limiting the movement of his arm. He also has pain that wakes him up at night. He states there was no trauma or past history of trauma. He is not involved any particular sports, exercise regimen, or other activities where he uses the motion of his shoulder repeatedly. He has a sedentary job. The pain just started and is gradually getting worse. Question What is the most likely diagnosis?

Correct answer: Frozen shoulder Explanation Suspect a frozen shoulder, also known as adhesive capsulitis, if shoulder pain comes on gradually, limiting the motion at both the glenohumeral and scapulothoracic joint. Often there will be limitation of abduction, external rotation, and forward flexion. The hallmark of frozen shoulder is a spontaneous onset of pain that worsens gradually and insidiously, followed by loss of range of motion. The pain may radiate, but pain below the elbow is not common. Pain at night is also a common symptom. Acute subacromial bursitis, which is painful enough to cause marked limitation of movement, happens more often in younger patients. The point of tenderness is often over the subacromial bursa, along with the swelling and warmth. This is noticed most when the client is lifting objects and combing their hair. Tendinitis or tenosynovitis in the rotator cuff, biceps, or biceps tendon sheath often precedes subacromial bursitis. The pain following a severe trauma to the shoulder may come from a subluxation or dislocation. Often the client will state that the shoulder just doesn't feel right, something popped, or something is out of place. Fibrositis syndrome is a form of rheumatism, which can cause pain in multiple locations, including the upper extremity. Some doctors do not believe these syndromes exist. Others have demonstrated that there is exaggerated, reproducible tenderness at specific anatomic locations in clients with this condition. Overuse syndromes may involve the elbow. Muscle strains may be vocational or recreational due to weight lifting, throwing, or racquet sports. There is pain and acute tenderness at the origin of the extensor muscles of the forearm, which attach just distal to the lateral epicondyle of the humerus. Overuse syndromes can include a joint that is used in a repetitive motion over a period of time.

Case A 33-year-old woman presents with joint pain and morning stiffness that lasts more than an hour. Her past medical history is significant for postpartum depression 2 years ago. On examination, you find her body mass index (BMI) is 18 and she displays signs of symmetrical small joint inflammation. You are waiting for X-rays and labs to confirm the diagnosis of rheumatoid arthritis. In the meantime, she is asking about risk factors, particularly the possibility of worsening her joint condition if she becomes pregnant again. Question What is the primary risk factor in this patient's case?

Correct answer: Gender Explanation The correct response is gender. Rheumatoid arthritis affects women 3 times more often than men. Although it may present at any age, rheumatoid arthritis is most frequently diagnosed between the ages of 40 and 50. This patient's age is not a risk factor for the disease. Depression is usually a consequence of the arthritis, not its cause. Arthritis is more common in obese patients; this patient is underweight (low BMI). Pregnancy is not a risk factor for rheumatoid arthritis; it actually has ameliorating effects on the disease activity.

Case A 43-year-old woman presents after being diagnosed with breast cancer 2 years ago. Tests have indicated that the cancer has spread. You discover, upon examination, that she has a visibly protruding vertebra. Question What term best describes this deformity?

Correct answer: Gibbus Explanation Gibbus is an angular deformity of a collapsed vertebra. The deformity may be protruding outward and may be caused by metastatic cancer, tuberculosis, or other infections of the spine. Scoliosis is the lateral curvature of the spinal column. This can be functional or structural when it compensates for other abnormalities. Scoliokyphosis is the combined lateral and posterior curvature of the spine. Kyphosis is the posterior (backward) curvature of the spine as viewed from the side. It is a rounded thoracic convexity. This is common in aging and in women. Lordosis is the curvature of the lumbar spine. It is the accentuation of the normal lumbar curve. It may exist as a compensation for other deformities (e.g., kyphosis).

Question A 19-year-old boy presents with pain and deformity of his right dominant shoulder after a sudden jerking movement to the same from a wrestling competitor approximately 1 hour ago. He was unable to continue wrestling and has pain with any movement of the right shoulder. On exam you see a loss of normal shoulder contour anteriorly. There is no Acromioclavicular joint tenderness. What is the most likely diagnosis

Correct answer: Glenohumeral dislocation Explanation This is a glenohumeral dislocation (shoulder dislocation). With an acute dislocation there is considerable pain with any movement. There is loss of normal shoulder contour (more so with anterior dislocation rather than posterior dislocation). With a cuff tear there is gross weakness to resistance and gravity. Shoulder (AC) separation would have tenderness over the injury site as would a clavicle fracture or sternoclavicular subluxation.

Case A 50-year-old woman presents with a painful red foot. She does not have a significant past medical history. 2 nights ago, she went to an office party. She ate and drank throughout the evening and later spent a great deal of time on the dance floor. Throughout the night, she felt fine and continued to enjoy herself. Over the next 2 days, she began to have increased pain and swelling in her right foot. The pain became so severe that she could not put her foot flat on the ground to walk. On examination, it is noted that her largest toe on her right is red, swollen, and painful to touch. She cannot bend the toe or bear her full weight on that foot. Her blood pressure is 122/84 mm Hg, and pulse is 80/min; respiration is 14/min, and temperature is 38.2° C. A CBC is done and is within normal range. An analysis of her synovial fluid shows an elevated white blood cell count and crystals are birefringent, with strong negative elongation when viewed under polarized light. Question What is the most likely diagnosis?

Correct answer: Gout Explanation Gout is a form of inflammatory arthritis that is the body's response to the deposition of uric acid crystals in the joints. It presents as acute monoarticular arthritis in 90% of patients. Gout has a peak incidence between the age of 30 and 50 years, and it is more common in men than in women. In early gout, usually only 1 or 2 joints are involved. The smaller joints of the lower extremities are usually the ones that are involved. For example, the first toe is often involved. There are 4 clinical phases of gout. These are: asymptomatic hyperuricemia, acute gouty arthritis, inter-critical gout, and chronic tophaceous gout. Podagra, or inflammation of the first metatarsophalangeal joint, is the initial joint manifestation involved in about half of all cases. However, podagra may be observed in patients with pseudogout, reactive arthritis, gonococcal arthritis, psoriatic arthritis, and sarcoidosis. The attacks usually begin abruptly and can reach maximum intensity in 6 - 12 hours. The joints are red, hot, and extremely tender. Untreated, the characteristics of gout change over time. The attacks become more polyarticular. Although more joints may become involved, inflammation in a given joint may become less intense. Attacks occur more frequently and last longer. Eventually, patients may develop a chronic polyarticular arthritis, which can be symmetrical and resemble rheumatoid arthritis. Tophi, which are collections of uric acid crystals in the soft tissues, occur frequently in untreated patients. They can be found in multiple locations, including the fingers, toes, and in the olecranon bursae. Tophi tend to develop after 10 years in untreated patients who develop chronic gouty arthritis. Acute flares of gout can occur in situations that lead to increased levels of serum uric acid, such as the use of alcohol, overindulgence of certain foods, trauma, hemorrhage, or the use of medications that elevate levels of uric acid. It is important to note that it is the abrupt change in uric acid levels and not the actual amount that can trigger an attack. Patients may be tested to determine if they are hyperexcreters of uric acid. By measuring levels on a monthly basis, it is possible to adjust their medications accordingly. A stress fracture can result in a painful, swollen, and red toe, but there would not be any crystals present in the synovial fluid. Pseudogout has a similar presentation; however, the crystals in the synovial fluid would be positively birefringent. These crystals are made of calcium pyrophosphate and are rod-shaped with blunt ends, whereas urate crystals are shaped more like needles. Cellulitis in this patient would most likely yield an elevation of her WBC count. There is also no evidence in her history of trauma to the foot, such as a bite, cut, or scratch. Septic arthritis may also present with a red, swollen, and painful toe. However, the synovial fluid would most likely show the presence of a bacterial organism, not crystalline structures

Case Ico-delete Highlights A 48-year-old man is awakened from his afternoon nap by an agonizing pain at the base of his big toe on the right side. On examination the first metatarsal phalangeal joint is swollen, red, tender, and warm to touch. The patient also appeared to have an inflammed knee and as a result, a decision was made to tap the right knee rather than the first metatarsal joint. Analysis of the fluid from the right knee shows: Leukocytes 68,000/mm3 Neutrophils 75% Crystal analysis Negatively birefingent Gram stain Negative Question What is the most likely diagnosis?

Correct answer: Gout Explanation Gout is an inflammatory arthritis associated with hyperuricemia. Uric acid crystals are deposited in the joint. These crystals appear as needle-shaped, negatively birefringent crystals. The condition is commonly seen in middle-aged men. Onset is sudden with agonizing pain, swelling, and redness of the joint. There is often a family history of gout. In osteoarthritis, as well as rheumatoid arthritis, there would not be crystals in neutrophils in the joint fluid. In septic arthritis there would be bacteria, but no crystals in the fluid. Reactive arthritis is a sterile synovitis, which occurs 1 to 2 weeks after an infection. In this case, like with septic arthritis, there are no crystals in the fluid.

Case Ico-delete Highlights A 23-year-old Caucasian man presents with lower back pain. The pain has been occurring for several months. The pain is worst in the morning and improves with activity. He has associated stiffness. There is no known injury to the area, and the pain has not responded to rest and heat. He has also noticed blurred vision and photophobia. On examination, there is decreased range of motion in the back. Uveitis is noted. Question Based on the patient's findings, which laboratory study would likely be positive?

Correct answer: HLA-B27 Explanation HLA-B27 is correct. The patient has lower back pain with uveitis, which is suggestive of ankylosing spondylitis. HLA-B27 is positive in 90% of Caucasian patients and 50% of African-American patients with ankylosing spondylitis. Anti-CCP antibodies and rheumatoid factor are incorrect. Anti-CCP antibodies and rheumatoid factor are found in rheumatoid arthritis and will be negative in ankylosing spondylitits. Anti-nuclear antibodies is incorrect. Anti-nuclear antibodies are negative in ankylosing spondylitits and are found in conditions such as systemic lupus and scleroderma. Anti-phospholipid antibodies is incorrect. Anti-phospholipid antibodies are found in anti-phospholipid syndrome and would not be present in ankylosing spondylitits.

Case A 67-year-old woman presents with painful distal interphalangeal joints (DIPJ) in both hands. Past medical history is positive for a 10-year history of osteoarthritis that has been treated with NSAIDs. On clinical exam, both hands present with enlargements of the DIPJs that are not warm or red, but are tender to palpation. Question What is the most likely diagnosis?

Correct answer: Heberden's nodes Explanation Heberden's nodes are detectable bony enlargements of the distal interphalangeal joints (DIPJ) of the hands. The joint may be limited in function and tender to palpation. Bouchard's nodes are bony enlargements of the proximal phalangeal joints (PIPJ). The joint may be limited in function and tender to palpation. A mucoid (mucinous) cyst arises from the joint capsule in the distal or proximal phalangeal joints. It contains degenerative myxomatous fibrous tissue from degenerative arthritis affecting the joint. Gout generally affects the joints of the foot (primarily the first metatarsophalangeal joint). It appears as a swollen, highly painful joint that is warm to the touch. The joint aspiration shows monosodium urate crystals. An inflamed bursa is secondary to a trauma to joints associated with a bursa. Some of the joints that can be affected are the elbow, the shoulder, and the first metatarsophalangeal joint

Case A 63-year-old man, with long-standing right shoulder pain from impingement syndrome, presents because he cannot lift his arm after pulling the starter cord on his lawn mower 1 month ago. An AP view of his right shoulder is ordered. Question What finding on the plain films may indicate rotator cuff arthropathy?

Correct answer: High-riding humeral head (subluxation) Explanation With large, long-standing tears of the rotator cuff, AP radiographs may reveal a high-riding humerus relative to the glenoid (subluxed), indicative of rotator cuff arthropathy. Widening of the acromioclavicular (AC) joint is indicative of AC joint injury (separation grade II). Cystic changes in the humeral head are indicative of long-standing rotator cuff tendonitis or bursitis. Narrowed glenohumeral joint space is seen with shoulder joint arthritis. A Bankart lesion of the glenoid is indicative of previous traumatic glenohumeral dislocation.

Case A 28-year-old woman presents to the emergency department with a right ankle injury; it occurred approximately 30 minutes ago while sliding into base during a softball game. She notes pain and an inability to bear weight on the affect extremity. The patient denies other injuries. Her past medical history is unremarkable, with no known medical conditions, no allergies, no medications, and no prior surgeries. She denies the use of alcohol, tobacco, and drugs. She has not been sexually active for 1 year, and she reports a normal menstrual period within the last 2 weeks. On physical exam, the patient appears in some discomfort; she is lying on the hospital bed. Vitals are normal. The right ankle appears visually deformed, with dislocation of normal foot anatomy. Edema is prominent. The patient is acutely tender over the lower leg, especially at the lateral and medial malleoli. There is no tenderness with palpation of the calf and proximal limb. Distal to the injured ankle, pulses, coloration, temperature, sensation, and capillary refill are normal. The patient has normal range of motion in the right knee, and she can wiggle her toes. She is unable to move the ankle joint. Due to patient discomfort, stability tests of the joint are not performed. The skin is intact over the injured area. The left lower limb and the remainder of her physical exam are normal. Imaging is obtained and shown in the image below. Question What represents the most appropriate pharmacologic treatment to administer to this patient at this point?

Correct answer: Hydromorphone Explanation This patient is presenting with pain due to a dislocated ankle fracture, affecting both the lateral malleolus and the medial malleolus. (The posterior malleolus could also be fractured, but the given image does not allow visualization of that structure.) The primary treatment for this patient's fracture is a surgical fixation. Medications, most often non-steroidal anti-inflammatory drugs and narcotics, are appropriate for pain control. No single agent is superior for acute fracture injuries. Given that the timing of surgery is unclear at this point, it would be prudent to choose a narcotic analgesic as opposed to NSAID. Therefore, hydromorphone is the most appropriate choice of those listed. inhibits osteoclast activity, is used for osteoporosis treatment and prevention. It plays no role in acute fracture treatment. This patient's history, age, and mechanism of injury are not suspicious for an osteoporotic fracture. Cefazolin, a 1st-generation cephalosporin, would be an appropriate antibiotic for infection prophylaxis, but it is not appropriate to administer at this point. Furosemide, a loop diuretic, is often used for edema. As noted above, this patient's edema is secondary to the fracture and soft tissue injury. The edema should resolve with reduction of the dislocated ankle and normal healing. Diuretics should not be given to reduce the edema. Instead, non-pharmacologic measures, such as elevation, ice, and immobilization are recommended. Tetanus toxoid is important to remember in any open and potentially dirty skin wounds presenting to the emergency department. However, this patient's skin was intact on exam (closed fracture), and a tetanus booster is not necessary for this injury.

Case You are evaluating a 34-year-old man who was brought in to the emergency department after being the driver in a single car motor vehicle accident. Preliminary radiologic studies show a comminuted fracture of the left tibia. Currently the patient is describing a markedly increasing amount of pain felt in the injured extremity. He describes the pain as being a 10/10 that is becoming worse with each passing minute. With anguish on his face he describes it as a deep, achy, burning pain. You quickly examine the left leg and note pallor, a tense "wood-like" feeling of the extremity, diminished sensation, and muscle weakness. Question Given the most likely diagnosis, what would be the most appropriate pharmaceutical intervention for the patient's pain at this time?

Correct answer: IV opioids Explanation The patient should be treated with IV opioids. This patient in the above scenario is suffering from acute compartment syndrome. Compartment syndrome should always be considered an urgent and extremely emergent situation due to the consequences that can ensue. Acute compartment syndrome (ACS) most often develops after the patient has suffered some type of significant trauma and especially involving a long bone fracture. Fractures account for more than 75% of cases of ACS. More often, ACS is seen in patients under the age of 35 years of age and also has a higher incidence in young men, especially after they experience fracture of the tibial diaphysis or distal radius; risks of developing ACS is even more increased if the patient suffers from a comminuted fracture. Our patient described above fits these scenarios that show a higher risk of developing ACS. Many times, the signs and symptoms of ACS occur in a stepwise fashion; an important clue is a rapid progression of symptoms and signs in a short amount of time. Symptoms include pain out of proportion to the apparent injury, persistent deep ache or burning pain, or even paresthesias. Signs that would be evident include pain with passive stretch of muscles in the affected compartment, a tense compartment with a firm 'wood-like' feeling, pallor, diminished sensation, muscle weakness, and a late finding of paralysis. Adequate decompressive fasciotomy is the procedure that must be performed as soon as possible. Just prior to surgery the affected leg in the patient above should be immobilized with the ankle in slight plantar flexion—this helps decrease the deep compartment pressure and does not cause increased pressure in the anterior compartment. Due to the severity of pain a patient with acute compartment syndrome will have, the most appropriate answer for pain management would be IV opioids. Although a patient controlled analgesia (PCA) opioid route may be considered a viable option, this should not be used until intervention has occurred due to the fact that it could mask the severity of the pain; therefore, complications may occur. Oral acetaminophen, oral NSAIDs, or topical NSAIDs could be options, but due to the intense and severe pain this condition creates, it is unlikely they will be effective in managing the patient's pain. References

Case A 36-year-old woman presents with a 1-week history of flu symptoms for which she has taken an over-the-counter nasal decongestant. After evaluation and treatment, she asks about a color change in her hands. For the past several months, her hands have been intermittently turning white and then blue; the fingers become red and then tingle and burn. She is concerned about these episodes and feels they primarily occur during periods of stress. The patient has a family history of diabetes (mother) and heart disease (father). The patient's hands currently appear normal, but she is not having symptoms. Question What is the most likely underlying cause for this condition?

Correct answer: Idiopathic Explanation The correct response is idiopathic. Raynaud's disease, syndrome, or phenomenon, is named for the French physician who first described the process in 1862; it results from intermittent spasm of the arterioles. This leads to a disruption in blood flow and resultant color change, either blanching (white) or cyanosis (blue). The condition usually involves the vessels of the fingers and toes, but it can also involve the lips, nose, ears, and nipples. Primary Raynaud's, or Raynaud's disease, is idiopathic; it is the most common type and tends to be less severe than secondary Raynaud's. Raynaud's phenomenon, which occurs secondary to a specific disease or condition, is also called Raynaud's syndrome. Risk factors for primary Raynaud's are female gender, age less than 30, family history, and living in a cold climate. Risk factors for secondary Raynaud's include age (generally older than 30), existence of a medical condition (e.g., autoimmune disease, atherosclerosis, pulmonary disease, or thyroid disease), history of injury to the affected body part, exposure to chemicals, cigarette smoking, and living in a cold climate. Raynaud's is often triggered by cold or stress. Secondary conditions that may lead to this process include thyroid disease, autoimmune disease (e.g., rheumatoid arthritis, polymyositis, dermatomyositis, Sjögren's syndrome), atherosclerosis, polycythemia, and thyroid disease. Certain medications can also cause Raynaud's phenomenon including birth control pills, beta-blockers, migraine, and cancer medications. Some over-the-counter medicines and diet aids can cause it, as well. The diagnosis is made clinically, although there is a test involving nail fold capillaroscopy that may be positive in the setting of connective tissue disease. Most cases do not require treatment. Common medications used if treatment is indicated include calcium channel blockers, alpha-blockers, and nitroglycerin. While all the choices of causal conditions for this patient's Raynaud's are correct, the most common cause is idiopathic (primary Raynaud's). A workup for underlying causes may be indicated.

ase A 34-year-old man presents with a 10-year history of progressive weakness in his arms and legs. His problems include difficulty brushing his hair and raising his arms above his head. He also is unable to rise from a chair without using his arms to push himself up from a seated position. Your exam notes decreased muscle bulk and objective weakness of the deltoid, biceps, triceps, hip flexors, and hamstrings. Hand strength and handgrip release are normal. He reports that his father developed similar symptoms as a young man, and the father now requires a wheelchair. Question What diagnosis best explains the patient's symptoms?

Correct answer: Limb-girdle muscular dystrophy Explanation The history and physical exam are most compatible with a limb-girdle muscular dystrophy that affects the proximal muscle of the shoulder and pelvic girdles. The condition is not usually present at birth but develops in young adults. The symptoms are gradual and progress over many years; some adults will end up requiring assistive devices. Multiple-inheritance patterns have been described, including the autosomal dominant inheritance that best fits this case. Duchenne muscular dystrophy has an onset in young boys, progresses to death (usually in the late teenage years), and has an X-linked inheritance pattern where male-to-male transmission is not possible. Pompe disease is a lysosomal storage disease due to deficient enzyme activity of alpha-glucosidase (also called acid maltase). Like most enzyme defect disorders, it is inherited in an autosomal recessive fashion, which makes it less likely in this case because 2 generations are affected. Pompe may present with similar clinical features to this case, including the presentation of respiratory complaints such as sleep apnea due to respiratory muscle weakness. Myotonic dystrophy is a variable form of muscular dystrophy that can present in adults with symptoms of delayed/impaired muscle relaxation (myotonia). Commonly, this affects the hands; difficulty with opening jars, releasing doorknobs, and handshakes will be reported. Other features of myotonic dystrophy include cataracts, premature balding, and diabetes mellitus. Hemochromatosis is a disorder of iron overload that does not typically cause overt or prominent muscular symptoms. Instead, reports of arthritis and fatigue are common. Hepatic dysfunction is common in later stages of the disease.

ase A 43-year-old woman presents with stiffness and pain in her fingers. It takes her about an hour in the morning to be able to use her hands. The symptoms started approximately 3 months ago, and they have gradually worsened. Recently, she also realized that some of her finger joints are swollen. She has a 2-year history of knee pain when climbing stairs. The patient's medical history includes hypertension, hyperlipidemia, and coronary artery disease. Her current medication is aspirin (81 mg daily), celecoxib (600 mg daily), glucosamine (500 mg tid), niacin (3 g/day), simvastatin (10 mg qh), vitamin E, and a multivitamin supplement. Vital signs are temperature 98.5° F, blood pressure 145/85 mm Hg, pulse rate 80/min, and respiratory rate 20/min. Physical examination reveals a well-developed, well-nourished woman in no apparent distress. Physical examination is unremarkable except for swelling around the metacarpophalangeal and proximal interphalangeal joints of the 2nd to 4th fingers bilaterally. There are no skin alterations. Laboratory values are as follows: white blood cell count 26,900/mm3; hemoglobin 14.9 g/dL; hematocrit 44.4%; platelet count 152,000/mm3; and CRP 29 mg/L. X-rays taken of both hands and knees show juxta-articular osteopenia, bone erosions, and loss of articular cartilage. Question What is the most logical next step?

Correct answer: IgM RF (rheumatoid factor) Explanation The most likely diagnosis is rheumatoid arthritis. The best test to establish the diagnosis is IgM for rheumatoid factor. A positive rheumatoid factor (RF) alone does not necessarily establish a diagnosis of rheumatoid arthritis. However, in combination with the symptoms, its presence is significant for the prognosis. Patients with high titers tend to have a more severe and progressive disease. 5% of the healthy population has a positive rheumatoid factor. It can be present in chronic liver disease, infectious mononucleosis, Sjögren's syndrome, SLE, tuberculosis, pulmonary fibrosis, leprosy, syphilis, visceral leishmaniasis, malaria, schistosomiasis, and subacute bacterial endocarditis. It also can be positive transiently in people after vaccination or transfusion. Ultrasound and MRI are diagnostic measurements if X-rays, which should be taken first, are unremarkable. Both methods can show incipient development of pannus as well as erosions around the bone-cartilage area, which is important for early diagnosis. In the early stages, X-rays only reveal evidence of tissue swelling and joint effusion. Within weeks of onset, juxta-articular bone demineralization can be seen. HLA-B27 is a histocompatibility antigen. It is found more often in Caucasians, and it is linked to diseases such as Reiter's syndrome and ankylosing spondylitis. Patients with rheumatoid arthritis are commonly HLA-DR4 positive. An electrophoresis would be indicated if scleroderma were suspected, since half the patients with that disease show hypogammaglobulinemia.

Case A 47-year-old woman with a history of obesity presents to the emergency department due to pain in her right ankle after a fall 2 days ago. Upon physical exam, she has pain and tenderness at the tip of the medial malleolus and has the inability to bear weight for at least 4 steps. The patient does not present with pruritus or any rashes; however, the ankle does appear swollen. Additionally, the patient's foot does appear to be neurologically intact. Furthermore, she does not present with a fever or erythema of the ankle. According to the Ottawa Ankle Rule, she qualifies for X-rays. Question What first-line treatment can be used to help treat this condition?

Correct answer: Immobilize in temporary cast/splint Explanation Immobilize in temporary cast/splint is the correct treatment, as this patient has a stable fracture that requires temporary immobility, rest, ice, and elevation. If the patient is in pain, NSAIDs can be taken as well. NSAID is a treatment for the patient's fracture; however, it is not the first-line treatment and therefore an incorrect answer to this problem. Opioid would be used as adjunctive therapy, not as a first line of treatment An antifungal would not help treat a patient with a fracture, and the patient does not present with any pruritus or rashes. The patient does present with an infection of the foot but does not present with a fever or erythema of the ankle; therefore, an antibiotic would not be the proper treatment for this patient.

Case A 68-year-old man presents with progressive onset of lower back pain, unsteady gait, and numbness and occasional weakness in his lower extremities. His symptoms seem worse after a day at work, where he stands for hours at a time. He finds that resting, especially sitting, helps alleviate the symptoms. He has not had any other treatment. Diffuse narrowing of the spinal canal is seen on recent MRI, and the patient now wishes to discuss management of this issue. Question What first-line management should be discussed with this patient?

Correct answer: Implement regular exercise Explanation The correct answer is to implement a regular exercise program. Patients with diffuse spinal canal narrowing have what is known as spinal stenosis and are not restricted in their activity, but they often limit their own activity due to intolerance and pain. As long as there is no evidence of fracture or gross instability, then activity and exercise should be encouraged. This will continue to keep muscles and joints healthy, and will continue to aid the patient in performing activities of daily living. In addition, spinal stenosis tends to be a progressive disease, so maintaining a healthy body is important to slow the progression. As spinal stenosis is a function of nerve compression due to spinal canal narrowing, it is also important to keep the patient's weight under control in order to reduce the added stress that extra weight puts on joints, bones, etc. Lumbar epidural steroid injections is not the correct answer. Steroids, either oral or injectable, should be reserved for severe cases where more conservative measures have not been successful. In addition, there is very little evidence that long-term efficacy of epidural steroid injections exists. Anti-inflammatory medications is not the correct answer. Medications, such as nonsteroidal anti-inflammatories and acetaminophen, can be used for pain in patients with spinal stenosis. However, these medications have side effects when used long-term and could be avoided for a time period or altogether if a patient maintains a good activity level and keeps their weight under control. The gastrointestinal side effects of anti-inflammatories made them a poor long-term management choice. Spinal manipulation is not the correct answer. Spinal manipulation plays no role in the treatment of spinal stenosis because it cannot widen the spinal canal. Decompressive laminectomy is not the correct answer. This answer describes the type of surgical treatment that is reserved for patients who fail more conservative treatment and have pain secondary to spinal stenosis that impedes their lifestyle. This would not be considered first-line treatment.

Case A 17-year-old male college freshman presents with chronic fatigue, back pain, and stiffness. The pain has been present for the past several months, but it appears to be worsening. The back symptoms are worse at night and first thing in the morning; they improve somewhat during the day. He is having difficulty staying productive at school because he is always tired. Question Given the patient's presentation, what laboratory finding is most probable?

Correct answer: Increased erythrocyte sedimentation rate Explanation The patient is presenting with signs and symptoms indicative of inflammatory arthritis, most likely ankylosing spondylitis (AS). Characteristic symptoms include back pain that is worst in the morning and improves with exercise. Back stiffness and extreme fatigue are also reported. AS is more common in male patients. As with other inflammatory arthritides, patients usually present at a younger age (18-35 years) relative to patients with mechanical causes of back pain or arthritis. AS is a clinical diagnosis, but many patients will have elevated erythrocyte sedimentation rates and c-reactive protein rates, both of which are nonspecific markers of inflammation. Elevated b-natriuretic protein is associated with congestive heart failure. An increased white blood cell count is usually a sign of acute infection; it may be seen in infective arthritides, but it is not a characteristic finding in cases of inflammatory spondyloarthropathy. Elevated uric acid levels suggest the diagnosis of gouty arthritis, but some patients with this diagnosis will have normal uric acid levels.

Case A 27-year-old Caucasian man returns to the emergency department with unbearable left lower leg pain; he does so approximately 6 hours after initial discharge. While playing lacrosse, the patient sustained a closed, mid-shaft tibial fracture. After casting and an anti-inflammatory, his pain was noted to be mild (2 out of 10 on 1 - 10 scale) at time of discharge. He reports his pain is increasing dramatically (it is now rated at 9 out of 10) and is unresponsive to his prescribed narcotic, acetaminophen, icing, and elevating his leg. He also describes a feeling of tingling and numbness throughout the lower left extremity. The patient arrived on crutches and appears in obvious pain. He is afebrile; he has a pulse of 105. The cast is intact and the remainder of the left leg and foot is examined. The patient is tender to palpation of the left foot and ankle, with swollen, firm tissue. The skin appears shiny. Pulses are normal, but capillary refill and sensation is decreased on the left foot compared to the right. With passive movement of the patient's left toes, he cries out in pain. There is no ecchymosis or visible skin lesions on the left foot or ankle. Question What explains the physiologic basis for this patient's severe pain?

Correct answer: Increased pressure within a confined tissue space, leading to venous obstruction Explanation This patient is exhibiting a presentation for compartment syndrome, in which the basic process is increased pressure within a confined tissue space, leading to venous obstruction. As the pressure rises, muscle, and nerve necrosis may occur. Compartment syndrome is an emergent condition, which may arise after significant trauma, such as a fracture or burn. It must be quickly identified and treated to prevent irreversible tissue damage. Chronic peripheral vascular disease would be unlikely to cause this patient's severe, acute pain. He is also much younger than the typical patient with chronic peripheral vascular disease. Demyelination of the neuron sheath is a problem with pain, paresthesias, and muscle weakness in various neurological disorders, such as multiple sclerosis. The acute injury this patient suffered should not cause any demyelination. Immobility of this patient's limb could ultimately lead to a deep venous thrombosis (DVT). However, the patient's pain began worsening shortly after his injury, and it is associated with the swelling of the tissue on his leg. This patient is relatively low risk for a DVT because he is young, active, and has no comorbidities. Increased dopamine release in the brain leading to activation of the reward centers is the mechanism for this patient's narcotic pain medicines. However, this process is the not the cause of his pain. If the patient had a simple fracture without compartment syndrome, his pain would likely be responsive to analgesics (e.g., narcotics).

Case A 40-year-old man presents with ongoing back pain; there is increased stiffness in the morning, and the pain has been going on for a few months. The patient reports that it can take him up to 30 minutes after waking up for the discomfort to improve. Physical examination and diagnostic testing confirms the diagnosis of ankylosing spondylitis. Question In addition to regular exercise, what medication will best control the patient's symptoms?

Correct answer: Indocin (Indomethacin) Explanation NSAIDs, particularly indomethacin, are effective in controlling the symptoms of ankylosing spondylitis; NSAIDs are effective due to their anti-inflammatory properties as well as the relative safety of their long-term use. Tylenol is not an NSAID; it would not be effective in this case. Prednisone has anti-inflammatory properties, but it is not an ideal choice for chronic treatment of this condition; there are serious adverse effects associated with chronic use. Aspirin is a drug that has mostly antiplatelet properties, as well as some anti-inflammatory properties, which does not make it an ideal choice for treating this condition. Oxycodone is strictly a pain reliever; it has no anti-inflammatory properties, which are needed to treat this condition.

Case A 22-year-old man presents with a several-month history of lower back pain. The pain is worst in the morning and is associated with stiffness. His symptoms improve with activity. He has no known injury to the area, and attempts to alleviate the pain with a heat pack have been unsuccessful. His ESR and CRP are both elevated, and testing for HLA-B27 is positive. An X-ray is shown in the image. Question What is the first-line treatment for this patient?

Correct answer: Indomethacin Explanation The above patient has ankylosing spondylitis and should be treated initially with indomethacin or other nonsteroidal anti-inflammatory drugs (NSAIDs). Ankylosing spondylitis is an autoimmune disorder that leads to progressive stiffening and eventual fusion of the spine. It is most common in males in their 2nd and 3rd decade of life. Patients complain of pain and stiffness, which are typically worst in the morning and improve with activity. Patients typically have elevation in their ESR and CRP. HLA-B27 is positive in 90% of Caucasian patients and 50% of African-American patients. The above patient's presentation, laboratory, data and X-ray all support the diagnosis of ankylosing spondylitis. Rest and continued supportive treatment with heating therapy is incorrect. Patients with ankylosing spondylitis are encouraged to remain active to help maintain their flexibility and posture. Acetaminophen is incorrect. While acetaminophen may alleviate some of the pain associated with ankylosing spondylitis, it will not target the inflammation that contributes to the disease. Etanercept, a tumor necrosis factor (TNF) inhibitor, may be used in the treatment of ankylosing spondylitis. However, it is considered a second line agent for those whose disease is not responsive to NSAIDs. Methotrexate is sometimes used in the treatment of ankylosing spondylitis, but it would not be the preferred initial agent of choice. Methotrexate is more commonly used in the treatment of rheumatoid arthritis. This patient is unlikely to have rheumatoid arthritis given the location of his symptoms, his radiographic findings, and his positive HLA B27.

Question A 34-year-old man presents with joint pain in his right foot. It began the other night when the pain woke him. On physical exam, you note redness and swelling of the first metatarsophalangeal (MTP) joint. What would be an appropriate treatment for this patient?

Correct answer: Indomethacin Explanation The clinical picture is suggestive of acute gout. Signs and symptoms include acute onset, which is typically nocturnal and monoarticular. The MTP joint of the great toe is often the most susceptible joint. The involved joint(s) are swollen, tender, warm, and are red in color. NSAIDs (Indomethacin) are the treatment of choice for acute gout. Methotrexate is not indicated for the treatment of acute gout attacks. Probenecid and allopurinol (uricosuric drugs) are used in the treatment of chronic gout or in patients with recurrent attacks. Theophylline is used in the treatment of chronic obstructive airway diseases.

Case A 54-year-old man presents with acute onset of excruciating pain in his right toe. The patient states the pain began shortly after dinner and has progressively worsened since then to the point where he is now unable to bear weight on the affected side. Aside from occasional backaches, he has never experienced pain like this before. He reports overall good health and aside from a multivitamin, he uses no medications or supplements. On exam, the patient's right foot is swollen, and the joint of the great toe is tense and inflamed. His temperature is 38° C, blood pressure is 155/85 mm Hg, and pulse is 103 beat per minute. Question What test would confirm the most likely diagnosis?

Correct answer: Joint aspiration Explanation The patient's clinical presentation is most consistent with the diagnosis of gout. Gouty arthritis is a complication resulting from an elevated serum urate level. Elevated urate is most commonly caused by renal under excretion caused by drugs such as diuretics or salicylates, renal insufficiency, or less commonly inherited renal tubular defects in urate handling. About 10% of patients—so-called "overproducers"—generate excess urate. Excess urate production may result from enzymatic defects in the purine degradation pathway, as in the case of Lesch-Nyhan syndrome. Alternatively, excess urate may be caused by increased cellular turnover resulting from hemolysis, malignancy, or chemotherapy. Regardless of the cause, hyperuricemia leads to deposition of urate in joint spaces, with subsequent crystallization that causes inflammation and other clinical signs and symptoms as presented in this patient. Gouty arthritis most commonly affects a single joint of the lower extremity. Classically, it affects the metatarsophalangeal joint of the great toe. The patient experiences acute onset of joint swelling, pain, and erythema. Mildly elevated temperature and leukocytosis may also be present. Since the symptoms of gout can mimic other causes of acute joint pain, such as that caused by infection or autoimmune conditions such as rheumatoid arthritis, it important to establish a definitive diagnosis. This is best accomplished by obtaining a sample of synovial fluid from the affected joint space. In the case of gout, examination of the specimen using polarized light microscopy will reveal negatively birefringent, needle-shaped urate crystals. The shape of the crystals distinguishes gout from other crystal-related joint diseases. Pseudogout, which is caused by calcium pyrophosphate dehydrate deposition, produces crystals that are rhomboid-shaped with weakly positive birefringence. Arthritis caused by hydroxyapatite deposition appears as brown globules under the microscope. To rule out septic arthritis, synovial fluid specimens should be sent for Gram stain and culture as well. Although most patients with gout have elevated serum uric acid levels, this does not establish the diagnosis; therefore, it is not the test of choice for diagnostic confirmation. While a mildly elevated white blood cell count also frequently accompanies the presentation of gout, it is seen with many other causes of acute arthritis; therefore, it is not helpful for definitive diagnosis. An X-ray of the affected joint is likely to demonstrate inflammation, but this also is a non-specific finding. If an infectious arthritis is suspected, blood cultures may reveal the offending organism; however, culture of aspirated joint fluid will provide a more definitive result. References

Question A 37-year-old man presents with pain in his left shoulder after a motor vehicle accident. The ER doctor suspects that the left shoulder pain is secondary to splenic rupture. What is this sign called?

Correct answer: Kehr's sign Explanation Kehr's sign is pain in the left shoulder secondary to splenic rupture. Cullen's sign is ecchymosis around the umbilicus that can sometimes be seen with acute pancreatitis. Grey Turner's sign is ecchymosis of the flanks that can sometimes be seen with acute pancreatitis. Lisker's sign is tibial bone tenderness that can sometimes be elicited with deep vein thrombosis. Blumberg's sign is abdominal rebound tenderness. This is an indication of peritoneal irritation.

Case A 32-year-old man presents with sudden onset of severe chest pain. He describes the sensation as a tearing pain that radiates to his back. He is tall and thin with very long arms and fingers. He has a medical history of lens subluxation and some other eye problems. Based on the patient's history and physical, a dissecting aortic aneurysm secondary to an underlying genetic condition is suspected. Question What bony abnormality is most commonly associated with this patient's condition?

Correct answer: Kyphoscoliosis Explanation This patient has Marfan syndrome, an inherited autosomal condition. People with Marfan syndrome can be tall, lanky, and thin with long arms and fingers. Ocular involvement is seen with Marfan syndrome, including lens subluxation and myopia. A dissecting aortic aneurysm is a complication of Marfan syndrome. A dissecting aortic aneurysm can present with the sudden onset of severe chest pain, as in the case here. The pain of a dissecting aneurysm is sometimes described as "tearing" and sometimes radiates to the back. Scoliosis and kyphoscoliosis are seen with Marfan syndrome. Kyphoscoliosis consists of both kyphosis and scoliosis. Scoliosis is an abnormal lateral curvature of the spine. Kyphosis is a spinal deformity due to extensive flexion. It is an exaggeration of the thoracic curvature. This produces a "hunchback." Lordosis is an extension deformity. It is an exaggerated lumbar curvature. Lordosis is also called swayback. Osteoporosis is a decrease in the bone mineral density. Lordoscoliosis is a combination of lordosis and scoliosis. It is lateral curvature of the spine and exaggerated lumbar curvature.

Question A 13-year-old girl presents for her school physical. On examination, you notice the posterior curvature of her thoracic spine to be very prominent and bulging backward. What type of deformity of the spine does she have?

Correct answer: Kyphosis Explanation Kyphosis is a posterior convex angulation of the thoracic spine as evaluated on a side view. Excessive and exaggerated angulation results in cosmetic problems, back pain, and cardio-respiratory problems. Kyphosis may be the post-traumatic or non-traumatic type. Scheuermann's kyphosis, postural, congenital, and metabolic bone disease are a few of the non-traumatic causes of kyphosis. Scoliosis is the lateral curvature of the spinal column. This can be functional or structural when it compensates for other abnormalities. Scoliokyphosis is the combined lateral and posterior curvature of the spine. Lordosis is the anterior angulation of the spine in the sagittal plane. It is the accentuation of the normal lumbar curve. It may be a compensation for other deformities such as kyphosis. Gibbus is a sharp angular deformity of the spine that could be due to a collapsed vertebra. The deformity may be protruding outward and may be caused by metastatic cancer or tuberculosis of the spine.

Case An 83-year-old malnourished woman presents for examination. She stands with her shoulders rounded and has an exaggerated thoracic convexity. Her spine is not laterally deviated. Question Which of the following most accurately defines the patient's spine condition?

Correct answer: Kyphosis Explanation Kyphosis is the forward curvature of the thoracic spine as viewed from the side. It creates a rounded and exaggerated thoracic convexity. This is common in aging patients and in women. Scoliosis is the lateral curvature of the spinal column. This can be functional or structural when it compensates for other abnormalities. Scoliokyphosis is the combined lateral and posterior curvature of the spine. Lordosis is the curvature of the lumbar spine. It is the accentuation of the normal lumbar curve. It may be as a compensation for other deformities, e.g. kyphosis. Gibbus is an angular deformity of a collapsed vertebra. The deformity may be protruding outward and may be caused by metastatic cancer or tuberculosis of the spine.

Question An elderly woman sustained a fracture of the proximal humerus 4 days ago while she was out of town visiting her daughter. She was seen in an Urgent Care clinic there and was told to follow up with her doctor when she got home. They placed her arm in a shoulder immobilizer and told her to leave it on. She was and is neurovascularly intact distal to her fracture. She comes to your office now for further care. What physical exam finding should be expected after 3 or 4 days?

Correct answer: Large ecchymotic area Explanation With a proximal humerus fracture in an elderly woman (which is common), gross ecchymosis will be present between 2 and 4 days post-injury. It may even travel into her upper chest and breast tissue. Since she was placed in a shoulder immobilizer, further deformity is unlikely unless she had further injury, such as a fall onto the arm. Although edema is associated with this fracture, it will stay in the arm near the fracture site, especially since she was splinted. Also since she has been splinted, the pain should be less since movement is restricted. Compartment syndrome is possible in the arm compartment but not likely in the forearm compartment. References:

Case Ico-delete Highlights A 30-year-old man presents with pain on the outer aspect of the right elbow for the past few days. The pain was mild and initially associated with vigorous activity; however, for the past 2 days, he has been experiencing pain during daily activities (e.g., lifting objects with his palm down, brushing his teeth, and shaking hands). He started doing carpentry as a hobby 2 months ago. He has no other symptoms and is otherwise healthy. On examination, there is point tenderness at a specific point on his right lateral elbow. An X-ray of the elbow does not reveal any abnormal finding. Question What is the most likely diagnosis?

Correct answer: Lateral epicondylitis Explanation Tennis elbow (TE), which is also called lateral epicondylitis, is a painful condition of the elbow caused by repetitive use of the wrist extensors. It is an overuse injury of the extensor tendons of the forearm at their attachment at the lateral epicondyle. The condition is associated with activities such as racquet games, carpentry, and knitting. Symptoms include pain at the lateral elbow that worsens with activity (especially squeezing movements), lifting objects with the palm down, turning a doorknob, and flexing the wrist towards the forearm. Diagnosis may be made by the tennis elbow test. X-ray is usually normal. Tenosynovitis is an inflammation of the synovial sheath, most commonly occurring at the wrist, hands, or feet. There is pain and swelling at the joint. Golfer's elbow, or medial epicondylitis, is a condition of worsening pain at the medial aspect of the elbow during forearm pronation and sudden wrist flexion. This condition is also associated with repetitive movements such as racquet games, carpentry, or typing. Diagnosis is made by eliciting pain on wrist flexion and forearm pronation against resistance. X-ray is usually normal. Radial tunnel syndrome refers to pain in the dorsal forearm due to compression of the deep branch of the radial nerve. Olecranon bursitis presents with painless swelling at the olecranon tip after repeated friction to the elbow. There is no tenderness or decrease in range of motion on examination.

Question What developmental problem is indicated in the attached abdomen radiograph? Refer to the image.

Correct answer: Lateral scoliosis Explanation Mild lateral scoliosis is seen in this radiograph of the abdomen. The lateral deviation is lacking the rotary effect that can accompany this developmental malformation. Rotary scoliosis is seen when the transverse process of the spine is rotated off the central axis. Spina bifida involves a failure of the two laminae to unite, leaving a bony defect in the vertebral canal. On X-ray, this appears as a characteristic gap between the laminae, usually in the lumbar or sacral regions. When the defect involves only bone, the condition is known as spina bifida occulta. When the defect also causes an outpouching the meninges, it is referred to as spina bifida with meningocele. Chiari malformation is a downward displacement of brain contents into the spinal column. Depending on the severity of the malformation, there may be no symptoms or significant neurological defects and hydrocephalus.

Case An 8-year-old afebrile Caucasian boy presents with significant right hip and knee pain. The patient walks with a limp, and his mother denies any history of trauma. On physical examination, you note atrophy of the right quadriceps muscles and decreased range of motion (ROM) of the right hip, particularly with internal rotation and abduction. Leg length inequality is also observed on the right compared with the left. Frog-leg radiographs of the hip reveal cessation of growth at the capital femoral epiphysis and a smaller femoral head epiphysis with widening of the articular space on the right side. There is also a linear radiolucency within the right femoral head epiphysis. White blood cell counts, C-reactive protein, and erythrocyte sedimentation rates are all normal. Question What is the most likely diagnosis?

Correct answer: Legg-Calvé-Perthes disease (LCPD) Explanation Legg-Calvé-Perthes disease (LCPD) is an idiopathic osteonecrosis of the capital femoral epiphysis of the femoral head; it can lead to severe degenerative arthritis if not treated promptly and aggressively. Patients with LCPD are usually male Caucasians ages 3-10 (with a median age of 7). Patients may present with a limp, hip pain, or anterior knee pain without a history of trauma. Physical findings of LCPD include decreased hip motion (especially with internal rotation and abduction), muscle spasm, painful gait, leg length inequality, short stature, and thigh muscle atrophy. Plain film radiographs in affected patients reveal cessation of growth at the capital femoral epiphysis and a smaller femoral head epiphysis with widening of the articular space on the right side. If a linear radiolucency is also seen within the right femoral head epiphysis, this is suggestive of a subchondral fracture. Treatment of LCPD includes minimal weight-bearing and protection of the joint; this is accomplished by keeping the femur abducted and internally rotated so that the femoral head is held inside the rounded portion of the acetabulum. Abduction and rotation of the femur are accomplished either by the use of orthotic devices (bracing) or surgery (osteotomy). Slipped capital femoral epiphysis (SCFE) is a condition that occurs when the femoral head is displaced posteriorly and inferiorly in relation to the femoral neck, placing the patient at risk for avascular necrosis unless there is orthopedic internal fixation. Patients with SCFE are usually 10-16 years old. They are more likely to be obese, male, and African American. The clinical and radiographic findings in the patient are more consistent with LCPD than SCFE. Developmental dysplasia of the hip (DDH) is a condition that affects infants and is associated with ligamentous laxity. The classic examination finding of children with DDH is revealed with the Ortolani maneuver, whereby a palpable "clunk" is felt when the hip is reduced in and out of the acetabulum on physical examination. Treatment of DDH consists of a Pavlik harness, especially if instability is present. Duchene muscular dystrophy is an X-linked condition that leads to a defective dystrophin protein, a protein that is integral to the structural stability of the muscle myofiber. Without dystrophin, the muscles of affected patients are susceptible to mechanical injury; they undergo repeated cycles of necrosis and regeneration with progressive muscular weakening. The clinical history and radiographic findings in the case above are more consistent with Legg-Calvé-Perthes disease. Osteomyelitis of the right hip is a possibility in this patient, but osseous infections normally produce characteristic laboratory values (elevated white blood cell counts, C-reactive protein, and erythrocyte sedimentation rate) that are typically accompanied by fever and pain of the affected extremity. 3-phase technetium radionuclide bone scanning and MRI examinations can aid in the diagnosis of osteomyelitis. It is unlikely in this patient due to his normal WBC and normal ESR.

Case An 89-year-old Caucasian man presents with pain in his left shin. He bumped his leg on a coffee table about 3 weeks ago; he developed some mild discomfort, bruising, and a small gash in the skin. His injury seemed to be healing well, but his condition has worsened over the last few days. Now he is moaning with pain, and he says it hurts to walk on the leg. He describes the pain as "horrible", and he rates it an 8 on a 1 - 10 pain scale. His daughter reports the pain keeps him up at night; he is unresponsive to ibuprofen and narcotic pain pills. He denies fever and chills. Test results are shown in the chart. Complete blood count Elevated white blood cell count Urinalysis Normal Uric acid Normal D-dimer Normal Comprehensive metabolic panel Normal Erythrocyte sedimentation rate Elevated C-reactive protein Elevated X-ray of leg Non-specific soft tissue swelling, no fracture Leg ultrasound Normal MRI of leg Tibial edema and necrosis Question What physical exam finding is most consistent with this patient's history and test results?

Correct answer: Localized erythema Explanation This patient presents with a likely osteomyelitis. Osteomyelitis is infection of the bone tissue. In this patient's case, some bacterial spread from a relatively minor injury at the skin surface is likely responsible. Osteomyelitis can affect all age groups, but it more commonly affects the elderly and those with poor peripheral circulation. Long bones and vertebrae are most commonly affected. We would expect this patient's pain to be out of proportion with exam findings, which may be minimal. With acute osteomyelitis in adults, exam findings are often limited to localized erythema and edema, with tenderness, but with exaggerated pain complaints. Acanthosis nigricans is a physical exam finding of darkening of the skin folds, especially in the neck or axillary or inguinal regions. It is associated with diabetes. It would not be found with osteomyelitis. Hyperreflexia (also known as autonomic dysreflexia) is a life-threatening condition, due to overactivity of the autonomic nervous system. It is associated with spinal cord injuries, but not with osteomyelitis. Livedo reticularis is a skin finding in which there is a fine, lacy purplish discoloration or mottling. It is most commonly found in the lower extremities, and it can be related to poor circulation and autoimmune disorders. It does not fit with this patient's history and test results. A positive Homans' sign (pain with forceful dorsiflexion of the patient's foot) has been traditionally used to test for deep venous thrombosis (DVT). However, it has not been shown to be reliable for DVT evaluation. This patient's history and tests do not indicate DVT, and the Homans sign would not be expected to be positive for osteomyelitis.

Case A 67-year-old man presents with the complaint of a "tender bulge in his right dominant arm" following an attempt at lifting his couch 2 days ago. He admits to having mild chronic shoulder pain and taking ibuprofen (Motrin) for relief. While attempting to lift the couch, he felt a snap in his shoulder and noticed a bulging in his anterior arm. Question What anatomic structure is most likely injured?

Correct answer: Long head of the biceps brachii tendon rupture Explanation The long head of the biceps brachii passes through the bicipital groove of the humeral head and attaches to the superior rim of the glenoid. It functions to suppress the humeral head on the glenoid surface, but it is exposed to friction rubbing over time. Therefore, it is subject to rupture, even with what seems like minimal trauma. The distal biceps rupture may occur, but it is less common, more painful, and requires much more force to rupture. The short head of the biceps brachii attaches to the coracoid process of the scapula and is less likely to rupture since there is minimal irritation to this tendon from use. The supraspinatus is a rotator cuff muscle and would not cause a bulge when it ruptures. The triceps brachii muscle is on the posterior arm and would not cause bulge in the anterior arm if it ruptured.

Case A 16-year-old boy presents following the striking of a wooden door with a closed fist an hour ago when he was angry at his mother. He is neurovascularly intact, and the skin is closed. There is an obvious deformity with a loss of small finger metacarpal knuckle. Radiographs reveal an oblique mid-shaft fracture of the fifth metacarpal with a palmar angulation of 45°. Question What physical exam technique must be performed to check for rotation of the fracture?

Correct answer: Look for finger malposition when the fingers are flexed into the palm. Explanation With a fractured fifth metacarpal, there is an increased incidence of malrotation of the distal digit due to the potential loss of metacarpal height and lack of ligamentous support of the metacarpal head by the intermetacarpal ligaments. Therefore, rotation must be checked closely to ensure proper position of the distal fracture segment in relation to the proximal fracture fragment for functional healing to take place. This is best accomplished by asking the patient to make a partial fist (gently) and watching for finger malposition (normally all the digits of a closed fist point towards a single spot on the scaphoid) or "cross-over." Be sure to compare the injured hand to the uninjured one as some "normal cross-over" can be seen with the fifth metacarpal. Looking at the fingernails in the extended position may not show rotation as easily as in the flexed position. Although there may be "folds of skin" over the metacarpal head, this wouldn't show you the malrotation if there is any. A fracture of the fifth metacarpal will not stop motion of the PIP or DIP joints (although it may cause pain at the fracture site). Edema is a given accompaniment with a fracture, but it won't give you an indication of a fracture rotation.

Question A 19-year-old man presents with pain and deformity of his right dominant shoulder after a sudden jerking movement to the same from a wrestling competitor approximately 1 hour ago. He states he felt a clunking sensation when it happened. He was unable to continue wrestling and has pain with movement of the right shoulder. On a physical exam, what findings would you expect?

Correct answer: Loss of normal surface contour of the shoulder Explanation With the dislocation of the glenohumeral joint, there will be a loss of normal contour of the shoulder's surface anatomy. Tenderness of the AC joint will be found more with a shoulder separation (AC Separation). There will be a great decrease in active and passive range of motion secondary to pain. Frozen shoulder has an insidious onset.

Case A 75-year-old man with a history of hypertension, hyperlipidemia, intravenous drug use, chronic lumbar stenosis, and diabetes presents to his pain management office. 2 weeks ago, he underwent a lumbar laminectomy for stenosis. He states that he has had fever for the past 6 days. He denies any headache, chest pain, shortness of breath, cough, abdominal pain, nausea, diarrhea, vomiting, or extremity pain. He was referred for an MRI of his spine; it reveals the following. Question What is the most likely additional clinical manifestation of this patient at this time?

Correct answer: Lower back pain Explanation This patient is experiencing acute osteomyelitis the recent lumbar laminectomy. The great majority of cases are monomicrobial in etiology. Of episodes of acute vertebral osteomyelitis, 40 - 50% are caused by Staphylococcus aureus. The signs and symptoms of vertebral osteomyelitis are nonspecific. Only about 50% of patients develop fever >38°C (100.4°F), perhaps because analgesic drugs are frequently used by these patients. Lower back pain is the leading initial symptom (>85% of cases). The location of the pain corresponds to the site of infection: the cervical spine in ~10% of cases, the thoracic spine in 30%, and the lumbar spine in 60%. On physical exam, local pain, swelling, erythema, and edema, scars, local disturbance of wound healing, and the cardinal signs of inflammation may all be observed. If periosteal tissues are involved, point tenderness may be present. Urinary symptoms, such as dysuria, hematuria, pyuria, or odor changes, are not typical of vertebral osteomyelitis. Neurologic deficits, such as radiculopathy, weakness, or sensory loss, are observed in about 1/3 of cases of vertebral osteomyelitis. 25% of patients with low-virulence organisms, such as coagulase-negative staphylococci or P. acnes, will have a sinus tract. Bowel and bladder dysfunction are complications caused mostly by spinal epidural abscess.

Case An 18-year-old man presents with progressive, severe knee pain. On physical examination, you note focal soft tissue swelling and erythema over the distal metaphysis of the left femur. Plain film radiographs of the left knee are obtained. Refer to the image. Question To what anatomic location does the neoplasm demonstrated on the radiograph most commonly metastasize?

Correct answer: Lungs Explanation Classic osteosarcoma is the most common malignant bone tumor in the pediatric population, and it is thought to arise from primitive mesenchymal, bone-forming cells. Classically, osteosarcoma is located in a metaphyseal, intra-medullary location, with its histologic hallmark being the production of a malignant osteoid. Plain film radiographs most commonly demonstrate an aggressive, bone-forming lesion located about the knee, most often affecting the femoral or tibial metaphysis. The three most common sites of origin for osteosarcoma are the femur (40%), tibia (20%), and proximal humerus (10%). The mainstay of osteosarcoma therapy is surgical removal of the malignant lesion, with a special emphasis on limb-sparing, limb-preserving procedures that preserve function and locomotion. Metastasis to the lungs is the most common; metastases to other sites are extremely rare.

Question A 33-year-old female presents for routine follow-up with an abnormal Computed Tomography (CT). Her past medical history includes a leg amputation four years ago for osteogenic sarcoma treated with neoadjunctive chemotherapy. She is married and a non-smoker. Her physical exam includes a BP-111/67 mm Hg, pulse-70/min, and a healed laparotomy incision for a colon resection from perforated diverticulitis. A chest CT demonstrates a new solitary, irregular, non-calcified 3 cm nodule. A CT guided biopsy demonstrates metastatic osteogenic sarcoma. Which is a negative prognostic indicator?

Correct answer: Lymph node involvement Explanation Pulmonary metastases may present with symptoms similar to primary lung cancer. These include chest pain, cough, hemoptysis, and dyspnea. Usually metastatic lesions are detected with chest X-ray or CT. CT can characterize suspicious findings on chest X-ray and detail the number of metastasis. CT detects approximately 80% of pulmonary metastases detected at surgical exploration. Positron Emission Tomography (PET) scanning is currently being investigated as a modality to access for pulmonary metastases. In patients with a previous history of sarcoma, a new pulmonary nodule has a 92% probability of being a metastatic nodule. Patients who are candidates for resection with pulmonary metastasis include those who have the a controllable or controlled primary tumor, no extrapulmonary tumor, no better method of treatment available, and adequate medical status. Tumor pathology that benefits from resection under these circumstances includes sarcoma, germ cell tumors, colorectal tumors, breast tumors, hypernephroma, head and neck tumors, melanoma, and hepatocellular carcinoma. Approximate five year survival for colorectal is 37%, breast 37%, kidney 41%, sarcoma 31%, melanoma 21%, and germ cell tumors 68%. Surgical approaches include unilateral or bilateral thoracotomy, sternotomy, and video-assisted thoracic surgery. Prognostic factors include tumor doubling time, disease-free interval, number of metastasis, lymph node metastases, and complete resection. Primary pulmonary thymoma is rare. Treatment includes surgical resection and possibly post-operative radiation. References:

Question A 73-year-old man presents with the inability to actively raise his left non-dominant arm to retrieve plates from the kitchen cabinet. This began a month ago after his shoulder pain improved. He had a history of pain in that shoulder for over 6 months that kept him from sleeping on the left side and the pain would wake him often. There was no specific injury he can recall although he felt a pop a month ago while taking out the trash. What is the diagnostic study of choice if surgery is indicated?

Correct answer: MRI Explanation This elderly man probably has a torn rotator cuff. If surgical treatment is being considered, MRI is the imaging study of choice because it can provide additional information on the status of the muscle and the size of the rotator cuff tear. Electromyelogram is used to assess the nerves. Ultrasound doesn't give as clear a picture of the tear due to the bony interference. Plain films can assess only the boney picture.

Case A 60-year-old man accompanied by his daughter presents with a history of low back and buttock pain; it is 6/10 in severity. The pain starts when he stands or walks, but he can walk faster and with less intensity of pain when he is leaning over the shopping cart. On examination, the pain increases on extension of the spine. Question In order to evaluate the etiology of the patient's test, what test would be most informative and cause least risk to the patient?

Correct answer: MRI (Magnetic resonance imaging) Explanation The likely diagnosis is Lumbar Spinal Stenosis (LSS). Magnetic resonance imaging (MRI) is the correct answer, as it is the most appropriate, noninvasive test for imaging degenerative lumbar spinal stenosis1. It helps in obtaining a more detailed evaluation of spinal structures, including hypertrophy of the facet joints and/or ligament flavum with a corresponding decrease in spinal canal diameter2. CT myelography is a useful study in patients who have a contraindication to MRI, for whom MRI findings are inconclusive, or in patients for whom there is a poor correlation between symptoms and MRI findings1. It requires the injection of contrast medium, which carries the risk of anaphylactic or allergic reaction. Therefore, it is associated with more risk than conventional MRI or CT scan. CT scan is a useful noninvasive study in patients who have a contraindication to MRI, for whom MRI findings are inconclusive, or for whom there is a poor correlation between symptoms and MRI findings, as well as in whom CT myelogram is deemed inappropriate1. Electrodiagnostic studies are best used when there is concern about additional neurologic compromise, such as peripheral polyneuropathy, rather than establishing diagnosis1. X-ray examination has great limitations in diagnosis of LSS. Therefore, it is not used for detailed evaluation of this condition2.

Question A 47-year-old grossly obese woman presents with left non-dominant shoulder pain and limited motion. The pain began about a month ago, and her shoulder has progressively lost motion during that time; now she can't reach overhead with that arm. There is no history of trauma; it doesn't wake her at night; and she can sleep on the left side. Her past medical history is significant for Diabetes Type I and hypothyroidism. She is on regular insulin and NPH insulin as well as levothyroxine (Synthroid). What diagnostic study would best help confirm your suspected diagnosis?

Correct answer: MRI arthrogram Explanation This is adhesive capsulitis (frozen shoulder). MRI arthrogram can substantiate a frozen shoulder by demonstrating a contracted capsule and loss of the inferior pouch of the capsule. Although plain radiographs are used to rule out pathology such as osteophytes, loose bodies, calcific tendonitis, or tumors, it won't show soft tissue, which is what the etiology of capsulitis is. CT is good for bony pathology mostly and less effective with soft tissues. EMG is used for nerve study. Ultrasound is good if there is fluid in the joint, but there is not in this case. Reference:

Case A 50-year-old obese woman presents with severe left knee pain. She states the pain began about 8 months ago but has gotten significantly worse in the last 3 months. The patient denies any trauma or event that initiated the pain. She notes stiffness in the knee first thing in the morning which only lasts around 5-10 minutes. The knee pain worsens with activity and is relieved with rest. The patient's medication list includes lisinopril 10 mg once daily for high blood pressure. She has a documented medication allergy to acetaminophen, which gives her hives. Physical examination reveals a female with a BMI of 40, limited range of motion of the left knee, and crepitus. Question Considering the most likely diagnosis for this patient, what pharmaceutical regimen would be recommended for her at this time?

Correct answer: Meloxicam Explanation This patient most likely has degenerative joint disease, also known as osteoarthritis (OA). OA is the most common form of knee arthritis. Common characteristics of OA include onset after 40 years old (usually patients older than 55 years old) along with obesity, and typically these patients are genetically predisposed. Other components seen in patients with OA initially have an insidious onset of pain that will rapidly progress. Characteristics of this pain will that is recognized as being exacerbated by activity and relieved at rest. Morning stiffness is common in patients with OA, although it will resolve within a 30 minute time period. Common symptoms experienced by patients with OA include buckling or giving way of the affected joint due to the bony areas impinging upon each other. Physical examination findings include limited range of motion of the affected joint and crepitus; the patient above exhibits both findings. Patients may also have tenderness to palpation of the joint, joint effusion, or even palpable osteophytes. Initially pharmaceutical management will be to initiate acetaminophen, up to 4 grams a day. Since our patient has an acetaminophen allergy, this would not be an appropriate choice. NSAID therapy or a COX-2 inhibitor would be an appropriate next option, making meloxicam the correct answer in this scenario. Allopurinol would be appropriate in a patient scenario that has the diagnosis of gout. This would not apply to this patient. Narcotics such as oxycodone are an inappropriate choice at this time before trying anti-inflammatories. Both alendronate and risedronate are in the class of bisphosphonates and therefore are used in patients diagnosed with osteoporosis.

Case A 25-year-old man presents with back pain and stiffness. He states he has had longstanding issues with back pain. He denies any trauma to his back. He has noticed associated increasing stiffness and general fatigue. He feels that these issues have gradually worsened over the last several months and are more persistent recently. He notes that the pain is much worse first thing in the morning, rating it a 6-7/10. Radiation occasionally occurs into the buttock areas and the patient feels the symptoms actually lessen with activity. Physical examination shows marked forward stooping of the thoracic and cervical spine with the lower spine showing the presence of a substantial reduction in lateral flexion. Question What would be the first-line pharmaceutical treatment to consider in this patient?

Correct answer: NSAIDs Explanation The correct response is NSAIDs. The patient history and physical examination described above is most likely a case of ankylosing spondylitis (AS). This is a chronic inflammatory disease that consists of many signs of symptoms, specifically significant back pain and progressive spinal stiffness. Many times, patients with AS will also have transient or persistent peripheral arthritis, as well as other manifestations such as anterior uveitis. Typically this is seen in male patients ages 20-30, although some may start having issues as early as their late teens. The goals of management of AS is to maximize the long-term health-related quality of life: relief of symptoms, maintenance of function, prevention of complications from spinal disease, and the minimization of extra-spinal and extra-articular manifestations and comorbidities. The first-line treatment for AS is nonsteroidal anti-inflammatory drugs (NSAIDs). Usually, empiric trials of different NSAIDs are warranted to gain satisfactory and beneficial relief of symptoms. TNF inhibitors and etanercept are both viable secondary options for patients who have failed relief from NSAID trials, but they are not considered first-line treatment. Sulfasalazine is sometimes used to help relieve symptoms in peripheral arthritic manifestations, but it is not useful for spinal or sacroiliac disease in AS. Corticosteroids have a very minimal impact on treating AS, and they could even worsen the osteopenia that is sometimes seen in this pathology.

Case You are evaluating a 34-year-old African American man for a 5-week history of increasing right groin pain. He denies any injury or history of similar pain. The pain is worse with movement and has progressed to the point that the patient has severe pain with weight bearing. He denies fever, chills, urinary symptoms, or any other complaint at this time. He has a past medical history of sickle cell disease and hypothyroidism. Physical exam reveals tenderness upon palpation of the groin with increased pain on both active and passive range of motion of the hip. Homan's sign is negative. Distal pulses and sensation are intact and normal. Question What diagnostics study should be ordered at this time?

Correct answer: MRI of the right hip with and without contrast Explanation This patient's presentation is concerning for avascular necrosis of the right hip. The most appropriate diagnostic test to order at this time is an MRI of the hip with and without contrast. Avascular necrosis, also known as osteonecrosis, is a focal infarct of the bone due to an interruption of blood supply.1,2 Avascular necrosis (AVN) most commonly occurs in the hips, jaw, and scaphoid bone of the hand.2 Risk factors for AVN include trauma, coagulopathy, alcoholism, chronic corticosteroid use, sickle cell disease, and auto-immune diseases such as RA and SLE.1,2 The differential of DVT does need to be considered in this patient; however, with the current clinical picture, it is unlikely a DVT is the primary cause of the patient's symptoms. While it is plausible this patient may have a DVT as well, ultrasound does not provide any information on the bone integrity and cannot diagnosis AVN. A CBC, CXR, and UA may be ordered as components of this patient's work up, but individually, these tests will not provide enough information to make a diagnosis.

ase A 65-year-old man with uncontrolled diabetes presents for evaluation of a foot injury. He ran into his dresser 3 days ago, and an ulceration has developed at the site of the injury. He admits to minimal localized pain, and he denies any associated fever or chills. On examination, the ulceration measures 2.5 cm in diameter, and a sterile probe can be easily advanced to the underlying bone. Question Based on your suspected diagnosis, what diagnostic test should be ordered because it has the highest level of sensitivity?

Correct answer: Magnetic resonance imaging Explanation MRI has been identified as the most sensitive imaging modality for the diagnosis of osteomyelitis, and it is particularly helpful in identifying soft tissue involvement. Plain radiographs are readily available, but they can be associated with a false negative result early in the course of the disease. CT and nuclear medicine bone scanning are both less sensitive than MRI. Nuclear medicine images are most helpful in identifying a location or multiple sites of infection. Venous ultrasound is not indicated for the diagnosis of osteomyelitis; it is used in the diagnosis of deep vein thrombosis.

Case A 27-year-old man presents with knee pain that started while he was playing basketball the previous afternoon. The patient states that as he attempted to pass the ball to a teammate, he twisted his left knee with his left foot still planted on the floor. He describes a popping sensation at the time of the injury, but he could still bear weight on his leg. He does not recall any immediate swelling. The morning of presentation, the patient noted swelling of the knee and pain that was significantly worse than it was the day before. He has taken 600 mg ibuprofen for the pain but has experienced only mild relief. Upon examination of the left knee, there is a limitation in range of motion, the presence of medial joint line tenderness, and an effusion. McMurray's test is positive. Question What is the optimal imaging modality or procedure for confirming the suspected diagnosis?

Correct answer: Magnetic resonance imaging (MRI) Explanation This patient most likely has a medial meniscal injury of the left knee. Meniscal tears are the most common knee injuries encountered in primary care. They result from a twisting action exerted on the knee joint while the foot is still in a weight-bearing position. Magnetic resonance imaging (MRI) is still the optimal test for confirming the diagnosis of a meniscal injury. All intra-articular structures (menisci, ligaments, and articulating surfaces) can be visualized in great detail. MRI is also very sensitive to abnormalities of bone marrow that may not be readily apparent on either radiographs or CT. Patients with meniscal injuries often report a tearing or popping sensation at the time of injury, followed by severe pain. Effusions associated with these injuries accumulate over hours; this is in contrast to ligamentous injuries in which hemorrhage causes immediate swelling. The knee swelling and pain associated with meniscal tears are typically worse the day after the injury. With medial meniscus injuries, tenderness localized to the medial joint line may be noted on exam and the range of motion of the affected knee may be limited. McMurray's test can be performed to aid in the diagnosis. Radiographs are usually the initial imaging studies performed when a patient complains of knee pain (acute or chronic), but plain films best show general wear and tear of joint spaces (arthritic changes) and bony deformities rather than intra-articular soft tissue damage/injury. With any acute injury of the knee, plain radiographs can be used to exclude fractures or other conditions. Anteroposterior (AP) and lateral radiographs of the knee best visualize the tibiofemoral and tibiofibular joints. The tangential ("sunrise") view allows the assessment of the patellofemoral joint and provides an unobstructed view of the patella. Weight-bearing radiographs are helpful in assessing the extent of arthritis in the knee joint and in planning various joint resurfacing procedures. Diagnostic ultrasonography is useful in imaging the soft tissues about the knee, muscle injury, bone healing, and foreign bodies. Recent technologic improvements have made this imaging modality increasingly more accurate while providing significant benefits over traditional modalities (e.g., magnetic resonance imaging), particularly regarding cost, patient satisfaction, and ease of use. Increasing use of diagnostic ultrasonography for musculoskeletal conditions is likely as its acceptance grows. The primary indication for computed tomography (CT) of the knee is to assess the alignment and degree of displacement of fracture fragments, particularly at the articular surfaces. The other major indication for a knee CT is to assess the integrity of the bone around a prosthesis (e.g., total knee arthroplasty). Arthroscopy of the knee is an invasive procedure and is rarely needed to make an accurate diagnosis, but arthroscopic procedures are commonly used to treat disorders/injuries that have already been confirmed by other imaging (e.g., meniscal injuries, damaged cartilage, foreign bodies, reconstruction of torn ligaments). Occasionally, arthroscopy of the knee is performed for general diagnostic purposes when a patient has chronic knee pain that cannot be explained by any other modality and is not improving with conservative intervention. References Doperak J, Anderson K. Acute Musculoskeletal Complaints. In: South-Paul JE, Matheny SC, Lewis EL, eds. Current Diagnosis & Treatment: Family Medicine. 4th ed

Question A 38-year-old man presents with pain and the inability to extend his middle finger DIP joint following a sudden jamming-type injury when attempting to catch a football 3 hours ago. His radiographs are normal. Based on the patient history, what is the most likely diagnosis?

Correct answer: Mallet finger Explanation A mallet finger deformity is due to the rupture of the extensor tendon at the base of the dorsal distal phalanx of any digit of the hand. Patients will report pain dorsally at the DIP joint with the inability to actively extend the DIP joint. Swan neck deformity may be the result of a mallet finger, but only over time. Since this injury is acute, swan neck deformity cannot be the answer. Boutonnière deformity is a result of rupture of the central portion of the extensor tendon at its insertion onto the middle phalanx. It results in flexion of the PIP joint and hyperextension of the DIP joint. DIP dislocation would be evident on radiographs. Trigger finger is the result of thickening of the A1 pulley of the synovial sheath or flexor tendon itself at the entrance to the synovial sheath. This causes the tendon to "catch" when the finger is flexed and then to "snap" like pulling a "trigger" when trying to extend the finger.

Question A football player plants his foot and is hit by an opponent on the lateral aspect of his knee. What structure is most likely affected?

Correct answer: Medial collateral ligament Explanation A force applied to the knee will tear the ligament on the opposite side. Striking the lateral aspect of the knee will damage the medial collateral ligament. Neither the meniscus nor the cruciate ligament is likely to be injured.

Case A 32-year-old man presents with pain on the inner aspect of the right elbow that began a few days ago. The pain was initially mild; however, for the past 2 days, the man says he has been experiencing it during daily activities such as lifting objects, brushing his teeth, shaking hands, etc. He recently became a rock-climbing instructor, and he climbs 3-5 days a week. On examination, there is point tenderness at a specific point on his medial elbow. Pain increases on wrist flexion and forearm pronation against resistance. An X-ray of the elbow is normal. Question What is the most likely diagnosis?

Correct answer: Medial epicondylitis Explanation The correct answer is golfer's elbow, also called medial epicondylitis. Golfer's elbow is a condition of worsening pain at the medial aspect of the elbow during forearm pronation and sudden wrist flexion. The condition is associated with repetitive movements such as rock climbing, racquet games, carpentry, typing, etc. Diagnosis is by eliciting pain on wrist flexion and forearm pronation against resistance. X-ray is usually normal. Tennis elbow, or lateral epicondylitis, is a painful condition of the elbow caused by repetitive use of the wrist extensors. It is a degenerative tendinosis of the extensor carpi radialis brevi muscle. The condition is associated with racquet games, carpentry, knitting, etc. Symptoms include pain at the lateral elbow, worsening with activity, especially squeezing movements, lifting objects with palm down, turning a doorknob, and flexing the wrist towards the forearm. Diagnosis may be made by the tennis elbow test. X-ray is usually normal. Tenosynovitis is the inflammation of the synovial sheath, most commonly occurring at the wrist, hands, or feet. There is pain and swelling at the joint. Radial tunnel syndrome refers to pain in the dorsal forearm due to compression of the deep branch of the radial nerve. Olecranon bursitis may or may not present with painless swelling at the olecranon tip after repeated friction to the elbow. There is no tenderness or decrease in range of motion on examination. References

Question A 25-year-old woman presents with pain and tingling sensation in her right hand. Her symptoms are especially intense at night. By holding her hand flexed for about a minute, you can provoke the symptoms. The paresthesia is extended to the palmar area of the thumb, index and middle finger, as well as half of the ring finger; thumb adduction and apposition are weakened on the right. What nerve is affected?

Correct answer: Median Explanation The described symptoms are typical for carpal tunnel syndrome, a compression of the median nerve in the volar aspect of the wrist between the flexor tendons and the superficial transverse ligament. The syndrome is relatively common, affects more women than men, can be uni- or bilateral, and is frequently associated with occupations that require a lot of wrist flexion. Compression of the ulnar nerve is often caused by trauma. Repeatedly leaning on the elbow is an example of trauma that could cause it. The paresthesia affects the palmar area of the 5th and half of the 4th finger as well as the dorsal area of the 5th, 4th, and half of the 3rd finger. There can also be weakness of the thumb adductor, 5th finger abductor, and interossei muscles. Compression of the radial nerve (having an arm hang over the back of a chair for a long period of time is an example of something that could cause it) leads to paresthesia over the area of the first interosseus muscle and weakness of wrist and finger extensors. The musculocutaneous nerve supplies the upper arm flexors and the skin of the radial side of the forearm. Compression of the peroneal nerve in the area of the fibular neck is common in bedridden patients or those with badly administered leg casts. Dorsiflexion and eversion of the foot are weakened, and a sensory deficit can be found over the anterolateral area of the lower leg, the dorsum of the foot, or between the 1st and 2nd metatarsal.

Question Following injury to a nerve at the wrist, the thumb is laterally rotated and adducted. The hand has a flattened appearance and is "ape-like." What nerve is damaged?

Correct answer: Median nerve Explanation In the palm, the median nerve supplies the 3 muscles [(i) abductor pollicis brevis, which abducts the thumb at carpometacarpal joint and metacarpophalangeal joint; (ii) flexor pollicis brevis, which enables the thumb to form one claw in the pincer-like action used in the picking up of objects; and (iii) opponens pollicis, which enable the thumb to adduct at the carpometacarpal and at the metacarpophalangeal joint] of the thenar eminence. All 3 are supplied by the median nerve. Injuries to the nerve make the thumb laterally rotated and adducted and makes the hand have a flattened appearance. The ulnar nerve supplies the 3 muscles of the hypothenar eminence, the palmaris brevis, the 3rd and 4th lumbrical muscles, all the interossei muscles, and the adductor pollicis muscle. Anterior interosseous nerve supplies to the flexor pollicis longus, the pronator quadratus, and the lateral half of the flexor digitorum profundus, as well as to the wrist, carpus, and distal radioulnar joints. Deep radial nerve supplies the supinator muscle and the posterior compartment of the forearm. Musculocutaneous nerve supplies the biceps, coracobrachialis, brachialis (= muscular branches), skin of the front and lateral aspects of the forearm down as far as the root of the thumb (= cutaneous branches), and the elbow joint (= articular branches).

Case A 33-year-old woman presents with tingling and numbness in the palms, thumbs, and index fingers bilaterally. Her symptoms are worse during the night. Recently, she noticed that she has difficulty grasping small objects. She works in a poultry farm and is involved in assembly packing. On examination, Tinel's sign and Phalen's test are positive. Question Ico-delete Highlights What nerve is compressed?

Correct answer: Median nerve Explanation The correct diagnosis is carpal tunnel syndrome (CTS), in which the median nerve is compressed. CTS is a compressive neuropathy of the median nerve in the carpal tunnel at the wrist. The carpal tunnel is bound by carpal bones on the 3 sides and is located at the base of the palm. The median nerve runs within it. Several conditions that cause compression of the nerve, such as acromegaly, hypothyroidism, diabetes, gout, pseudogout, repetitive work such as assembly packing, and prolonged use of vibratory tools, can cause CTS. Treatment consists of anti-inflammatory drugs, exercises, splinting, and carpal tunnel release surgery. Ulnar nerve compression causes numbness and tingling in the one and a half fingers on the ulnar aspect of the hand. Radial nerve compression causes pain in the upper extensor part of the forearm and weakness of extension of the wrist, fingers, and thumb. In posterior interosseous nerve compression, there is usually no sensory nerve deficit; motor paralysis of the extensors can occur. Musculocutaneous nerve compression is rare and causes symptoms in the forearm, not the hand.

Question A 65-year-old woman presents after tripping on a rug in her home and falling on her outstretched arm. She states that her "hand bent backwards" and she heard a "snap". Physical exam shows an obvious deformity over the distal radius, with radial shortening. What anatomical structure may also be injured by this classic Colles' fracture?

Correct answer: Median nerve Explanation The correct response is median nerve. Due to the mechanism of the injury (i.e., forced wrist dorsiflexion), the distal fragment is angulated dorsally, and the proximal fragment is volarly displaced. Bony fragments or compression within the carpal tunnel may injure the median nerve. The radial nerve and artery course in close approximation to each other, as do the ulnar nerve and artery, but they are not typically at risk in an uncomplicated classic Colles' fracture.

Case A 42-year-old woman works full-time as a data entry clerk and often puts in many hours of overtime. She is experiencing numbness and tingling in her right thumb, index finger, middle finger, and half of her ring finger. The numbness and tingling initially comes and goes, but it is constantly present after a few months. Question What nerve is involved?

Correct answer: Median nerve Explanation This patient has symptoms consistent with carpal tunnel syndrome. In carpal tunnel syndrome, the median nerve is compressed in the wrist. The distribution of her symptoms indicates distribution of the median nerve. The median nerve innervates the skin on the lateral 2/3 of the hand and fingers on the palmar side and the dorsum of digits 2, 3, and some of 4. The median nerve innervates most of the muscles of the anterior forearm and some of the muscles of the hand. The ulnar nerve innervates the skin of the medial third of the hand. The ulnar nerve innervates some of the flexors in the anterior forearm and most of the intrinsic muscles of the hand. The radial nerve innervates the skin and posterior surface of the upper extremity. The radial nerve innervates the muscles of the back of arm, forearm, and hand. The axillary nerve innervates some to the skin in the shoulder region and the back of the arm. The axillary nerve innervates the deltoid and teres minor. The musculocutaneous nerve innervates some of the skin on the lateral forearm. The muscles innervated by the musculocutaneous nerve are the brachialis, biceps brachii, and coracobrachialis. References

Case Ico-delete Highlights A 33-year-old woman presents for routine follow-up with an abnormal Computed Tomography (CT). Her past medical history includes a leg amputation 4 years ago for osteogenic sarcoma treated with neoadjunctive chemotherapy. She is married and a non-smoker. Her physical exam includes a BP 111/67 mmHg, pulse 70/min, and a healed laparotomy incision for a colon resection from perforated diverticulitis. A chest CT demonstrates a new solitary, irregular, non-calcified 3 cm nodule. Question Ico-delete Highlights What is the most likely diagnosis of this nodule?

Correct answer: Metastatic osteogenic sarcoma Explanation This patient most likely presents with metastatic osteogenic sarcoma. Pulmonary metastases may present with symptoms similar to primary lung cancer. These include chest pain, cough, hemoptysis, and dyspnea. Usually, metastatic lesions are detected with chest X-ray or CT. CT can characterize suspicious findings on chest X-ray and detail the number of metastases. CT detects approximately 80% of pulmonary metastases detected at surgical exploration. Positron emission tomography (PET) scanning is currently being investigated as a modality to access for pulmonary metastases. In patients with a previous history of sarcoma, a new pulmonary nodule has >90% chance of being a metastasis. The lungs are the most common site of metastasis for osteosarcoma. Primary lung cancer would be much less likely than a metastatic lesion in a patient of this age with no other risk factors for lung cancer. Lymphoproliferative diseases of the lung are rare. It is also much more likely that the patient has a metastatic lesion than a second primary cancer. Pneumonia may present with focal consolidation, but a calcified nodule is not suggestive of pneumonia. Primary pulmonary thymoma is rare. Treatment includes surgical resection and possibly post-operative radiation. References

Case A 65-year-old woman presents with fatigue, loss of energy, lack of appetite, low-grade fever, muscle and joint aches, and stiffness. She has a glucose-6-phosphate dehydrogenase (G6PD) deficiency and a history of coronary artery disease. On examination, you find swollen, painful, and tender small joints in a symmetrical pattern. According to the history, physical, and X-rays, as well as the presence of rheumatoid factor and antibodies to cyclic citrullinated peptides, you conclude that she has a flare-up of rheumatoid arthritis. Question What long-term therapy should be recommended in order to modify the course of the disease?

Correct answer: Methotrexate Explanation Methotrexate is usually the initial synthetic disease-modifying antirheumatic drugs (DMARD) of choice for patients with rheumatoid arthritis. It is generally well-tolerated, and often produces a beneficial effect in 2 - 6 weeks. NSAIDs will help the pain, but they will not modify the course of rheumatoid arthritis. They should only be used in conjunction with DMARDs. Sulfasalazine is a second-line agent for rheumatoid arthritis. It is not indicated in this patient because it causes hemolysis in patients with a glucose-6-phosphate dehydrogenase (G6PD) deficiency. Low-dose corticosteroids will produce a prompt anti-inflammatory effect in rheumatoid arthritis and slow the rate of bony destruction; however, their multiple side effects limit their long-term use. They are given only to reduce disease activity until the slower acting DMARDs take effect, or as adjunctive therapy for an active disease that persists despite treatment with DMARDs. Selective COX-2 inhibitors are effective for treating the pain in rheumatoid arthritis, but they will not modify the course of disease; they can increase the risk of cardiovascular events, particularly in patients not taking aspirin.

Case A 43-year-old woman presents with stiffness and pain in her fingers. It takes her about an hour in the morning to be able to use her hands. The symptoms started approximately 3 months ago, and they have gradually worsened. Recently, she also realized that some of her finger joints are swollen. She has a 2-year history of knee pain when climbing stairs. The patient's medical history includes hypertension, hyperlipidemia, and coronary artery disease. Her current medication is aspirin (81 mg daily), celecoxib (600 mg daily), glucosamine (500 mg tid), niacin (3 g/day), simvastatin (10 mg qh), vitamin E, and a multivitamin supplement. Vital signs are temperature 98.5° F, blood pressure 145/85 mm Hg, pulse rate 80/min, and respiratory rate 20/min. Physical examination reveals a well-developed, well-nourished woman in no apparent distress. Physical examination is unremarkable except for swelling around the metacarpophalangeal and proximal interphalangeal joints of the 2nd to 4th fingers bilaterally. There are no skin alterations. Laboratory values are as follows: white blood cell count 26,900/mm3; hemoglobin 14.9 g/dL; hematocrit 44.4%; platelet count 152,000/mm3; and CRP 29 mg/L. X-rays taken of both hands and knees show juxta-articular osteopenia, bone erosions, and loss of articular cartilage. Question What is the most appropriate treatment option?

Correct answer: Methotrexate Explanation The correct response is methotrexate. Modern management of rheumatoid arthritis (RA) is based on the knowledge of the joint-destructive course of the disease. Early diagnosis and onset of therapy is of utmost importance. Destruction of joints can be reduced, and economic costs (incurred through morbidity and increased mortality) can be lowered. The goal of treatment is to prevent inflammatory damage to the joints, keep the joints functioning, and increase quality of life. There are 2 groups of basic therapeutics for the treatment of RA: BASIC THERAPEUTIC DRUGS DOSAGE Conventionals Methotrexate 7.5 - 25 mg/wk po, sc, im Sulfasalazine 2 - 3 g/day po Leflunomide 20 mg/day po Biologicals Infliximab 3 - 10 mg/kg IV Etanercept 25 mg/twice a week sc Adalimumab 40 mg/every 2 weeks sc Anakinra 100 mg/day sc Biologicals (i.e., infliximab, etanercept, adalimumab, and anakinra) should only be used after failing to treat RA with conventional basic therapeutics or in combination with them. The criterion standard is methotrexate; it prevents thymidine synthesis by blocking the enzyme dihydrofolate reductase. This leads to increased apoptosis of activated T-cells and decreased production of IL-1. Cyclosporine and hydrochloroquine are other conventionals; they are used in mild cases or in combination with methotrexate. Infliximab is a monoclonal anti-TNF-antibody and can be combined with methotrexate. 2/3 of patients who did not respond to conventional therapy respond to infliximab (with improvement of symptoms). Infliximab is usually administered IV at 0, 2, and 6 weeks, and every 8 weeks thereafter. The dosage can be increased from 3 mg/kg to 10 mg/kg. Side effects include headaches, respiratory infections, urinary tract infections, and sporadic allergic reactions. Before starting therapy with TNF-blockers, tuberculosis must be ruled out; therapy can lead to tuberculosis. Etanercept is a p75 TNF-α-receptor blocker; it is comparable to infliximab. In addition to the possible side effects of infliximab, reactions at the injection site have been reported. Anakinra is an interleukin-1-receptor blocker; it is injected subcutaneously every day. Increased infection rates and reactions at the injection site have been reported. Glucocorticoids should only be administered in combination with conventional basic therapeutics until the anti-inflammatory effect sets in. Dosage should be 7.5 - 10 mg/day, and can also be administered into the joint directly.

Case A 35-year-old man presents with right shoulder pain that is becoming progressively worse. He expresses concern that, although he visits the gym 3 times a week, over the past month he has not been able to increase the amount of weight he lifts secondary to the shoulder pain. He has not tried anything to alleviate the pain. The pain is at its worst at night while he is trying to sleep. He also reports pain while in the shower washing his hair or using the shoulder press machine at the gym. He denies any history of recent trauma or sports-related injury; however, upon questioning, he reports that about 1 month ago he and his wife painted their entire house in a weekend. Upon physical exam of the shoulders, no swelling, atrophy, redness, or bruising is noted. Point tenderness is noted over the right lateral deltoid muscle. Active ROM of the right shoulder at 80 degrees of abduction elicits pain. Patient has a negative drop arm test, negative apprehensive test, and a positive Neer impingement test of the right shoulder. Question What is the suggested first line of treatment?

Correct answer: Modification of activity, NSAIDs, and physical therapy Explanation The first-choice treatment is modification of activities that produce pain, use of nonsteroidal anti-inflammatory drugs (medications may help relieve symptoms), and physical therapy. The goal is pain control and inflammation reduction. Movement is needed to maintain flexibility and range of motion. Rest arm in a sling is incorrect because movement needs to occur in order to prevent frozen shoulder (adhesive capsulitis). Movement is needed to maintain flexibility and range of motion. Subacromial corticosteroid injections are considered to be the third line of treatment, not the first. Surgical treatment is considered for refractory shoulder cuff pain or tears if conservative management failed for more than 6 to 12 months (therefore, not the first-line choice for treatment). Surgical treatment also depends on the patient's age, loss of function, weakness, and pain. Opioid analgesics might be used if the NSAIDs do not alleviate the pain; however, NSAIDs should be tried first.

Case A 70-year-old man with a history of type II diabetes presents with a 2-day history of a red, hot, painful, and swollen left toe. He cannot recall any recent injury or illness, but states that he has been eating a lot of protein-rich foods and consuming alcohol against medical advice. His only complaint is the painful foot. He denies fever, chills, headache, or problems with his right foot. An X-ray of the foot reveals no evidence of fracture, and analysis of the synovial fluid shows crystals that are negatively birefringent and needle-shaped. The patient begins treatment and is able to put pressure on the foot once again. Question What is the most likely initial treatment for the patient?

Correct answer: NSAIDs Explanation This patient has gout, which is a form of inflammatory arthritis. It is the body's response to the deposition of uric acid crystals in the joints. Gout presents as acute monoarticular arthritis in 90% of patients. In early gout, usually only 1 or 2 joints are involved. Usually, they are the smaller, lower-extremity joints. Podagra, or inflammation of the first metatarsophalangeal joint, is the initial joint manifestation involved in about half of all cases. However, podagra may also be observed in patients with pseudogout, reactive arthritis, gonococcal arthritis, psoriatic arthritis, and sarcoidosis. The attacks usually begin abruptly and can reach maximum intensity in 6-12 hours. The joints are red, hot, and extremely tender. Untreated, the characteristics of gout change over time. The attacks become more polyarticular. Although more joints may become involved, inflammation in a given joint may become less intense. Attacks occur more frequently and last longer. Eventually, patients may develop a chronic polyarticular arthritis, which can be symmetrical and resemble rheumatoid arthritis. Tophi, which are collections of uric acid crystals in the soft tissues, occur frequently in untreated patients. They can be found in multiple locations, including the fingers, toes, and olecranon bursae; they can also be found along the olecranon, where they may appear to be rheumatoid nodules. Tophi tend to develop after 10 years in untreated patients who develop chronic gouty arthritis. Acute flares of gout can occur in situations that lead to increased levels of serum uric acid, such as the use of alcohol, overindulgence of certain foods rich in protein, trauma, hemorrhage, or the use of medications that elevate levels of uric acid. NSAIDS are the treatment of choice in most patients without underlying health problems. They have an anti-inflammatory effect that works by inhibiting cyclo-oxygenase, which acts to produce leukotrienes from arachidonic acid. Indomethacin is the drug of choice and is usually given at a dose of 25 - 50 mg, 4 times a day, unless the patient is elderly. However, other NSAIDs, such as ibuprofen, naproxen, sulindac, and ketoprofen, may be used. Aspirin is not recommended because it can elevate uric acid levels. Some of the side effects of NSAID therapy include gastropathy, nephropathy, and liver dysfunction. It can also cause fluid overload in patients with congestive heart failure. Therefore, their use should be limited in these patients. Colchicine is an antimitotic drug that is often used in patients who cannot take NSAIDs or corticosteroids. However, due to its side effects (such as nausea, vomiting, and diarrhea and following intravenous administration- bone marrow suppression, renal failure, and death), it is not used as a first-line treatment. The normal dosage is 0.5-0.6mg orally every hour until relief or side effects occur or until a maximum dosage of 6 mg is reached. Allopurinol is used in the prevention of future attacks. It is not used to treat acute flare-ups. Patients with frequent gout attacks may begin prophylactic treatment to prevent further episodes. Allopurinol is used in such cases. Allopurinol blocks xanthine oxidase, thereby reducing the production of uric acid. It should be used in patients who overproduce uric acid. It is the most effective agent to lower serum uric acid levels. The dosage is 200 - 300 mg daily. The side effects include rash, gastrointestinal symptoms, headache, urticaria, interstitial nephritis, and rarely hypersensitivity syndrome. It was originally developed as a chemotherapeutic agent. However, alcohol can interfere with its effectiveness. Sulfinpyrazone, a uricosuric agent, is an alternative agent to aid in the prevention of attacks. However, it is not used as frequently due to the fact that it can cause bone marrow suppression. Antibiotics are used when the symptoms are caused by an infectious agent. This is not the case in this patient.

Case A 50-year-old man was playing baseball with his company team last weekend and is now experiencing severe pain in his left shoulder. He states that he has been the team pitcher for several years now. He has developed pain progressively in his left shoulder over the last few months. He denies any numbness or tingling in his arm, hand, or fingers of his left upper extremity. Question Which exam would you expect to yield pain?

Correct answer: Neer test Explanation Neer test is the only exam listed that evaluates for a rotator cuff injury. The Neer test is performed by having the patient internally rotate the shoulder while the practitioner flexes the arm forward. Pain will be elicited as the supraspinatus muscle presses against the acromion. Tinel sign evaluates for any irritated nerve, such as in carpal tunnel syndrome. Phalen test evaluates for carpal tunnel syndrome. Bragard stretch test evaluates for nerve root irritation at the level of L4, L5, and S1. Apley grind test evaluates for a torn meniscus.

Case A 54-year-old man presents with acute onset of excruciating pain in his right toe. The patient states the pain began shortly after dinner, and it has progressively worsened since then to the point where he is now unable to bear weight on the affected side. In addition, he explains that aside from occasional backaches, he has never experienced pain like this before. He reports overall good health and aside from a multivitamin, he uses no medications or supplements. On exam, the patient's right foot is swollen, and the joint of the great toe is tense and inflamed. His temperature is 38°C, blood pressure is 155/85 mm Hg, and pulse is 103 beats per minute. Labs reveal an elevated serum uric acid level. Question What is examination of synovial fluid from the affected joint space most likely to reveal?

Correct answer: Negatively birefringent, needle-shaped crystals under polarized light microscopy Explanation The correct response is negatively birefringent, needle-shaped crystals under polarized light microscopy. The patient's clinical presentation is most consistent with the diagnosis of gout. Gouty arthritis is a complication resulting from an elevated serum urate level. Elevated urate is most commonly caused by renal underexcretion caused by drugs (such as diuretics or salicylates), renal insufficiency, or (less commonly) inherited renal tubular defects in urate handling. About 10% of patients—so-called "overproducers"—generate excess urate. Excess urate production may result from enzymatic defects in the purine degradation pathway, as in the case of Lesch-Nyhan syndrome. Alternatively, excess urate may be caused by increased cellular turnover resulting from hemolysis, malignancy, or chemotherapy. Regardless of the cause, hyperuricemia leads to deposition of urate in joint spaces, with subsequent crystallization that causes inflammation and other clinical signs and symptoms as presented in this patient. Gouty arthritis most commonly affects a single joint of the lower extremity, classically the metatarsophalangeal joint of the great toe. The patient experiences acute onset of joint swelling, pain, and erythema. Mildly elevated temperature and leukocytosis may also be present. Since the symptoms of gout can mimic other causes of acute joint pain, such as those caused by infection or autoimmune conditions (e.g., rheumatoid arthritis), it important to establish a definitive diagnosis. This is best accomplished by obtaining a sample of synovial fluid from the affected joint space. In the case of gout, examination of the specimen using polarized light microscopy will reveal negatively birefringent, needle-shaped urate crystals. The shape of the crystals distinguishes gout from other crystal-related joint diseases. Pseudogout, which is caused by calcium pyrophosphate dihydrate deposition, produces crystals that are rhomboid-shaped, with weakly positive birefringence. Arthritis caused by hydroxyapatite deposition appears as brown globules under the microscope. To rule out septic arthritis, synovial fluid specimens should be sent for Gram stain and culture as well. Although most patients with gout have elevated serum uric acid levels, this does not establish the diagnosis and therefore is not the test of choice for diagnostic confirmation. While a mildly elevated white blood cell count also often accompanies the presentation of gout, it is seen with many other causes of acute arthritis, and therefore is not helpful for definitive diagnosis. An X-ray of the affected joint is likely to demonstrate inflammation, but this also is a non-specific finding. If an infectious arthritis is suspected, blood cultures may reveal the offending organism; however, culture of aspirated joint fluid will provide a more definitive result.

Case A 32-year-old man presents with pain in his back, buttocks, and posterior thighs for 2 days after lifting a heavy load at work. He denies pain when sleeping unless he rolls over in bed. He also has pain with just standing or sitting still in a chair. He denies any radicular symptoms or bladder or bowel dysfunction. Question What diagnostic tests does this patient require?

Correct answer: No diagnostic tests are required at this time Explanation No diagnostic tests are required with an acute lower back pain patient initially. Plain films usually are not helpful to diagnose acute back pain/strain. If the patient has pain at night, at rest, or has a history of significant trauma, however, then radiographs are indicated. MRI is best used for patients with findings suggestive of nerve impingement from a herniated disk. CT, though able to visualize herniated disk material, is best used for boney pathology. EMG is indicated if there is a long-standing nerve impingement.

Question A 43-year-old woman presents with pain, swelling, and limited motion of her right dominant elbow for the past half hour after slipping on a wet spot on the floor at work while carrying a tray of food. She landed directly on her bent elbow. She is unable to flex her elbow due to pain. There is edema and ecchymosis over the elbow itself. Which of the following physical exam findings is most possible with this injury?

Correct answer: Numbness of the little finger palmar surface Explanation This patient likely has a fracture of the olecranon. With this fracture, injury to the ulnar nerve is most possible. The ulnar nerve supplies sensation of the little finger palmar surface. Numbness of the dorsal thumb and back of the hand would be an injury to the radial nerve. Vascular compromise is rare with this injury, and thus capillary refill and radial pulse should be normal. References:

Case A 63-year-old man presents with pain in both knee joints. It began almost a year ago, was mild, and present only during activity, but has worsened now and is present even during rest. There is no joint stiffness. He is a diabetic, and has been on insulin treatment for the past 10 years. His BMI is 30. Vitals are within normal limits. Examination reveals pale conjunctivae. On examination of the knee joints, there is no tenderness, warmth, or erythema. Pain in the left knee joint is reproduced on walking. You suspect osteoarthritis and order lab investigations and X-ray. Meanwhile, you prescribe exercises and ibuprofen. Question What is a risk factor for osteoarthritis in this patient's case?

Correct answer: Obesity Explanation Obesity is a risk factor for osteoarthritis (OA). OA is a degenerative joint disease commonly seen in advanced age. It is characterized by deep achy joint pain involving the hip, knee, ankle, and foot joints with an insidious onset. The pain is usually present during activity initially and gradually worsens. The gait may be antalgic if weight-bearing joints are involved. The underlying pathology is the breakdown of the articular cartilage in the joint synovial tissue. It is believed to be due to excessive wear and tear, although nonspecific inflammatory changes may also occur. Some of the risk factors for OA include: Advanced age Obesity (due to increased weight-bearing stress on the joints). The knee joints are most susceptible. Menopause Trauma Excessive use Infection Certain hemoglobinopathies, previous rheumatoid arthritis, acromegaly, etc. Before the age of 55, OA affects women and men equally; however, after 55, it has been found to be more common in women. Anemia, insulin, and diabetes are not known to be associated with OA.

Case An 89-year-old Caucasian man is brought in by his daughter due to pain in his left shin. He bumped his leg on a coffee table about 3 weeks ago; he developed some mild discomfort, bruising, and a small gash in the skin. It had seemed as if his condition was healing well, but his condition has worsened over the past few days. Now he is moaning due to pain, and he says it hurts to walk on the leg. He describes the pain as "horrible" and an 8 on a 1 - 10 pain scale. The daughter reports the pain keeps him up at night and is unresponsive to ibuprofen and narcotic pain pills. He denies fever and chills. On physical exam, the patient is in obvious pain and is assisted to the exam table with limited weight bearing on his left leg. The lower extremities are examined; significant findings include healing and a scabbed lesion of approximately 3 cm in length across mid-tibia, with surrounding erythema and edema. Tenderness is elicited along the shin, extending well past the area of erythema. Homan's sign is negative. Distal pulses, temperature, coloration, knee range of motion, and lower extremity reflexes are symmetric and normal. Right lower extremity is normal. Question What is the most likely diagnosis?

Correct answer: Osteomyelitis Explanation This patient presents with a likely osteomyelitis. Osteomyelitis is infection of the bone tissue. In this patient's case, some bacterial spread from a relatively minor injury at the skin surface is likely responsible. Osteomyelitis can affect all age groups, but it can especially affect the elderly and in those with poor peripheral circulation. Long bones and vertebrae are most commonly affected. This patient's pain is out of proportion with exam findings. Often, with acute osteomyelitis in adults, exam findings are limited to local erythema and edema (with tenderness), but with exaggerated pain. Claudication is a condition in which the patient experiences pain in the calf region with prolonged walking or similar exercise. Pain should resolve upon rest. It indicates peripheral vascular disease. This patient's pain does not fit the pattern related to activity and rest. Gout is a painful condition affecting the joints, and it appears with erythema and warmth. It would not affect the tibia; it would not develop after a minor trauma. Pes anserine bursitis is a painful condition affecting the area on the medial proximal tibia, just inferior to the knee. Patients typically experience knee pain, and the physical exam would have pinpointed findings around the tibial plateau, not the mid-tibia (as is the case in this patient). Bursitis is much more likely to respond to pain treatments than osteomyelitis. Stress fractures commonly affect the tibia and can present with localized tenderness and edema on the tibia, such as in this patient's case. However, stress fractures occur in response to repetitive and excessive stress on the bone, such as with high-intensity training. This patient's history is not suspicious for stress fracture.

Case A 54-year-old man presents with a 2-month history of left, non-dominant shoulder pain. There is no history of trauma, but the pain began about a week after shoveling wet, heavy snow from his 100-foot driveway. At first, the pain seemed to come and go from day to day, but has gotten progressively worse and more constant. Pain is worse with overhead use, and he cannot sleep on his left side; he wakes up if he rolls over onto his left shoulder. On exam, he is tender to palpation over the left humeral greater tuberosity. He has pain with Neer and Hawkins tests, but has a negative drop arm test. You prescribe physical therapy and non-steroidal anti-inflammatories, and you also advise taking a break from using the arm for heavy work.. Question At what point would you recommend a steroid injection?

Correct answer: Only after failure of a home therapy program of at least 6 weeks Explanation The correct response is only after failure of a home therapy program of at least 6 weeks. When a home therapy program performed 3 - 4 times a day for 6 weeks fails to improve the symptoms of impingement syndrome, a subacromial injection of steroid could be considered. Posterior capsule stretching exercises should be performed at home prior to any steroid injection. Some patients require the supervision of a physical therapist to assure adequate stretching, but they still require at least 6 weeks of therapy prior to an injection. Steroid injections should not be repeated if the 1st one does not improve the symptoms for at least 4 - 6 weeks. Repeated steroid injections can weaken the tendons of the rotator cuff and potentially cause a rupture of the tendon.

Case A 28-year-old man presents first thing in the morning due to a left ankle sprain. The patient describes walking in the woods the day before while wearing flip flops and inadvertently stepping into a hidden hole. The patient immediately noticed extreme pain with weight bearing on the left ankle, making it extremely difficult for him to walk out of the woods. Ambulation currently is extremely difficult at this time. He describes it as an 8-9/10 on a 1-10 numerical pain scale. The patient admits to having ankle sprains before in his life, but this one has been much more severe and debilitating. He has noted significant swelling and exquisite tenderness to touch. He has treated his sprain with elevation, ice for 20 minutes at a time, and acetaminophen 1000 mg every 6 hours. Physical examination reveals a moderate degree of ecchymosis of the left ankle, with substantial mechanical instability and moderate restriction of range of motion. Question In addition to RICE guidelines and immobilization of the ankle, what pharmaceutical intervention should be initiated to further help reduce swelling and pain in this patient?

Correct answer: Oral NSAIDs Explanation Oral NSAIDS should be the pharmaceutical intervention of choice in this case. The scenario above is describing a Grade II ankle sprain. This injury typically involves an incomplete tear of a ligament. Patients will experience moderate pain, swelling, tenderness, and ecchymosis. There will be mild to moderate joint instability during exam and some restriction of the range of motion as well as loss of function. Ambulation and weight bearing are painful. Treatment should follow the MICE mnemonic: modified activities, ice, compression, and elevation. Since this is a Grade II ankle sprain there should be protected weight bearing with the use of crutches and an ankle stabilizer brace. Early motion is essential in this type of sprain and patients should be strongly encouraged to perform home exercise or see a physical therapist. Controlling pain and reducing the swelling is a common concern for most patients. Commonly prescribed medications are oral form analgesics such as NSAIDs (nonsteroidal anti-inflammatory drugs) to reduce this pain and inflammation.

Case A 13-year-old boy presents with a 5-month history of intermittent right knee pain. He notes that his pain is poorly localized over the anterior knee, but it is provoked by activities such as running, jumping, squatting, and climbing or descending stairs. As a result, he has had to curtail recreational games at recess and home. He denies any history of trauma, falls, injury, fever, chills, swelling, skin changes, hip pain, or ankle pain. His physical exam reveals a boy with an appropriate build. His right knee is notable for point tenderness over an enlarged and prominent tibial tubercle. There is associated focal swelling. An in-office X-ray is performed, demonstrating the following image. Question What is the most appropriate pharmacotherapeutic agent at this time?

Correct answer: Oral ibuprofen Explanation This patient is presenting with signs and symptoms consistent with Osgood-Schlatter disease. Therapy for Osgood-Schlatter disease (OSD) is conservative. Initial treatment includes the application of ice for 20 minutes every 2 - 4 hours. The only medications that need to be prescribed are nonsteroidal anti-inflammatory drugs (NSAIDs), such as oral ibuprofen, for pain relief and reduction of local inflammation (any NSAID may be used). Along with nonsteroidal anti-inflammatory drugs (NSAIDs), analgesics (e.g., acetaminophen) may be given for pain relief and reduction of local inflammation. Steroid injections should not be used, as they may weaken the patellar ligament as well as thin and depigment skin. Oral corticosteroids (e.g., prednisone) are not necessary. Gabapentin is a gamma aminobutyric acid (GABA) analogue; structurally related to neurotransmitter GABA. It is indicated as adjunctive therapy for partial seizures with or without secondary generalization as well as for post-herpetic neuralgia and in amyotrophic lateral sclerosis. Off-label uses include treatments for insomnia, tremors, diabetic neuropathy, hot flashes, restless leg syndrome, and cocaine withdrawal. Lorazepam is indicated as treatment for anxiety, short term treatment of insomnia, for perioperative sedation, status epilepticus, and off-label for chemotherapy-induced nausea and chronic insomnia.

Case An 11-year-old boy comes in to see you due to knee pain. He notes that he started having right knee pain about 3 months ago and has increasingly become more apparent. The patient plays basketball in the winter and baseball in the spring; he has done this for the last 2 years. When asked to point to where the pain is, the patient points to an enlarged area just inferior to the knee cap. Aggravating factors include participating in the sports mentioned, especially when he is doing excessive amounts of running and jumping. Alleviating factors include resting the knee. Physical examination reveals a prominence and soft tissue swelling over the right tibial tubercle as well as tenderness to palpation of the right patellar tendon. The remainder of the musculoskeletal exam is unremarkable. Question Ico-delete Highlights Given the most likely diagnosis, what would be the most appropriate pharmaceutical regimen that you can recommend at this time?

Correct answer: Oral nonsteroidal anti-inflammatory (NSAIDs) Explanation The condition that is being described in the above patient scenario is known as Osgood-Schlatter disease (OS). This is a condition that results from a tractions phenomenon; it results from repetitive quadriceps contraction through the patellar tendon at its insertion upon the skeletally immature tibial tubercle. By this description, the occurrence is found to be in adolescent patients, typically between 10-15 years old. Onset is typically gradual; the complaint of pain in the area of the tibial tubercle and/or patellar tendon region is common and especially noticeable after repetitive activities (especially running and jumping.) OS is seen to be the most frequent cause of knee pain in the pediatric population aged 10-15 years. OS rarely causes permanent impairment or disability, but rather loss of time from physical activity or athletics. Typical presentation will be a knee pain that is worsened with activity and improves at rest. Physical findings will include a prominence and soft-tissue swelling over the tibial tubercle as well as tenderness of the patellar tendon. Attempted flexion against resistance may initiate pain. Patients may also resist flexion of the affected knee because of inflammation and pain from the pull of the patellar tendon. Conservative care is commonly efficient and usually consists of rest and avoidance of the offending activity. NSAIDs could potentially be used to help alleviate the signs and symptoms but they have not been shown to shorten the course of OS. Topical NSAIDs would not be an appropriate option. Steroid injections as well as oral steroids should not be used as treatment of OS. Antibiotics are also not a viable option for treatment of OS.

Case A 35-year-old man with no significant past medical history has been experiencing progressive, moderately severe right knee pain for the past 3 months. Pain was initially felt only at night; however, it is now constant throughout the course of the day for the last several weeks. It is especially severe upon ambulation and during knee ranges of motion, causing him to limp. He denies fever, chills, weight changes, history of gout, sexually transmitted diseases, hip or back pain, recent instrumentation, trauma, or injuries. His physical exam reveals an antalgic gait with limp, limited ranges of motion of the right knee, and a 3 cm diameter firm, tender mass at the distal femur. There is no erythema, crepitus, alignment deformity, or effusion noted. Question What is the next appropriate step in the management of this patient?

Correct answer: Order an LDH, ALP, and a plain radiograph of the knee Explanation This patient's manifestations are most suggestive of osteosarcoma. It is typically seen in patients in their second or third decade, occurs more frequently in men than in women, and is found in the metaphyseal areas of long bones, with 50% of lesions seen about the knee joint. The distal femur is the most common site, followed by the proximal tibia, and then the proximal humerus. The only blood tests with prognostic significance are lactic dehydrogenase (LDH) and alkaline phosphatase (ALP). Patients with an elevated ALP at diagnosis are more likely to have pulmonary metastases. In patients without metastases, those with an elevated LDH are less likely to do well than are those with a normal LDH. Other laboratory tests include a complete blood cell (CBC), including a differential and platelet level, electrolytes, and liver and renal function tests. The practitioner should also obtain plain films of the suspected lesions in 2 views. Elevation of the periosteum may appear as the characteristic Codman triangle; in approximately 60% of cases, extension of the tumor through the periosteum may result in a so-called sunburst appearance.

Case A 14 -year old girl presents due to right leg pain. She is a competitive gymnast and works out in the gym 6 days a week. Upon physical exam, there is knee pain, swelling, and tenderness at the tibial tubercle. Additionally, the patient's muscles are very tight in front and behind her thigh. All laboratory findings are within normal limits. Question What is the most likely diagnosis?

Correct answer: Osgood-Schlatter Disease Explanation The clinical picture is suggestive of Osgood-Schlatter disease, because the patient is an adolescent; the disorder usually affects children who are going through a growth spurt and who are in sports with a lot of jumping. Additionally, the patient presents with the symptoms of pain, swelling, and tenderness around the tibial tubercle. Iliotibial Band Syndrome is not correct because the IT band runs on the lateral part of the thigh and knee. Although a patient can experience knee pain, it is not located over the tibial tubercle as described in the stem of the problem. Kneecap dislocation is not correct because the patient describes a gradual pain, rather than a sharp acute pain. Additionally, a patient with a dislocated knee cap can present with a bent knee and not be able to straighten the leg, which is not described in this patient. Patellar tendinitis is not correct, because the patient presents with pain over the tibial tubercle not the patellar region. Sinding-Larsen-Johansson is not correct because the patient describes pain around the tibial tubercle and not the patellar tendon, which is the structure associated with Sinding-Larsen-Johansson.

Case A 76-year-old man has a history of pain and stiffness in his knees bilaterally as well as his right hip. The pain is worse after an activity or first thing in the morning. Upon physical exam, the patient has limited range of motion and swelling in the painful joints. A synovial fluid analysis was conducted and it was negative for urate crystals. X-rays of the knees and bones do not show sclerosis of the bone. Additionally, a Gram stain was conducted and was negative for any bacteria. Question What is the most likely diagnosis?

Correct answer: Osteoarthritis Explanation Osteoarthritis is the correct answer because the patient presents with knee and hip pain after long use or activity, which is descriptive of osteoarthritis. Additionally, upon physical exam, the patient presents with swelling in his knees and hips, which is due to cartilage breaking down. As a result, this causes pain and swelling and decreased movement of the joint. Gout is not the correct answer because urate crystals could not be found in the synovial fluid analysis. Fibromyalgia is not the correct answer, as a patient with fibromyalgia would present with bilateral pain, fatigue, depression, and sleep disturbances. Additionally, the patient would not present with swelling of the joints; furthermore, pain after an activity is not present in fibromyalgia. Avascular necrosis is not the correct answer, as the patient's X-rays do not show sclerosis of the bone; therefore, avascular necrosis is ruled out. Septic arthritis is incorrect because a negative Gram stain confirms that septic arthritis could not be the answer.

Case A 73-year-old woman presents with pain and stiffness in both her wrists and right knee. She reports that the pain has been increasing slowly. The symptoms in the lower right leg have been present for 6 months, resulting in the woman walking with a limp. The patient reports that the pain is worse when using her right knee (the predominant reason for her limping) and the pain goes away when the knee is rested. The wrist pain is a new presentation. Upon physical exam, there is no joint swelling or erythema of the wrists or knees. Some crepitus is noted with range of motion of both knees (right more than left). Blood work, including ESR, is unremarkable. Question What is the most likely diagnosis?

Correct answer: Osteoarthritis Explanation The cardinal sign of osteoarthritis is pain that occurs only when the joint is in use. The pain is aching and poorly localized. Rest gives relief. Stiffness presents upon rising and after periods of rest. Some loss of motion involving extension and flexion can occur. Crepitus signifies bony contact (late stage). Rheumatoid arthritis is a chronic inflammatory condition in which the body's immune system attacks cartilage, bone, and sometimes internal organs. It is a symmetrical inflammatory arthritis involving small and large joints. Criteria include morning stiffness, symmetrical symptoms, serum rheumatoid factor, and radiologic changes. Symptoms such as fatigue, weight loss, and fever may accompany early rheumatoid arthritis. Gouty arthritis may include a painful joint and tophi deposits; these tophi deposits lead to joint enlargement and limitation of motion. Acute exacerbations of gouty episodes correspond with swelling, pain, and warmth of the joint. The skin over the affected area is often red, shiny, and tense. Symptoms usually begin as a sudden, severe, and unexpected arthritic attack affecting a single joint in one of the lower limbs. Some people experience a low-grade fever, loss of appetite, and a general feeling of not being well. Hyperuricemia is common in long-standing gout. Reactive arthritis is precipitated by an infection (often via the gastrointestinal tract or urogenital tract) in which the causative factor cannot be isolated from the joint. It has a similar pattern of involvement, occurrence of enthesopathy and sacroiliitis, and high association with HLA-B27, as is seen in the spondyloarthropathies. Symptoms include asymmetrical oligoarthritis of the lower limb, sausage digits (dactylitis), enthesopathies, and spondylitis. Pseudogout (calcium pyrophosphate arthropathy) results when calcium pyrophosphate crystals collect in joints. There may be attacks of joint swelling and pain in the knee, wrist, ankle, and other joints. It primarily affects the elderly. Joint fluid reveals calcium pyrophosphate crystals, and joint X-rays show calcification of cartilage.

Case A 50-year-old obese woman presents with severe left knee pain. She states the pain began about 8 months ago but has gotten significantly worse in the last 3 months. The patient denies any trauma or event that initiated the pain. She notes stiffness in the knee first thing in the morning; it only lasts around 5 - 10 minutes. The knee pain worsens with activity and is relieved with rest. Physical examination findings reveal a woman with a BMI of 40; there is limited range of motion of the left knee and crepitus. Plain film radiographs are ordered, and the interpretation reveals severe non-uniform joint space loss, osteophytes (present on both the left and right), and the presence of a subchondral cyst evident on left. Question Based on the history, physical examination, and results of the radiographic studies, what is the most likely diagnosis?

Correct answer: Osteoarthritis Explanation This patient is most likely suffering from degenerative joint disease, which is also known as osteoarthritis (OA). Common characteristics of OA include onset after 40 years old along with obesity being recognized as a frequent risk factor for development of OA. Other components seen in patients with OA include fairly rapid progressive onset of pain; it is exacerbated by activity and relieved at rest. Morning stiffness is common in patients with OA, although it will resolve within a 30 minute time period. Physical examination findings include limited range of motion of that affected joint and crepitus, to which the patient above has both. Patients may also have tenderness to palpation of the joint, joint effusion, or even palpable osteophytes. Radiographic hallmarks of OA include non-uniform joint space loss, osteophyte formation, cyst formation and subchondral sclerosis. Osteoporosis and osteopenia may be present in this patient population group, although they are much more common in older women that have experienced menopause. Occurrence of osteopenia or osteoporosis is higher in Caucasian women that possess other risk factors, such as tobacco or alcohol abuse, excess/deficiency in hormones, malignancy, and even some genetic disorders. Both disease states are typically asymptomatic until an actual fracture occurs. Loss of height may also be seen. Radiographic findings seen in both pathologies will include evidence of decreased bone density or pathologic fractures. Rheumatoid arthritis (RA) has a more insidious onset and characteristic morning stiffness that symmetrically affects small joints of the hands and feet. It usually presents in a polyarticular manner, and OA presents as monoarticular. RA involves the metacarpophalangeal joints as well as the wrists while OA affects the distal interphalangeal joints and potentially even the first metatarsophalangeal joints. Radiographic changes are extremely specific for RA. Patients will have evidence of the following, depending the progression of the disease: soft tissue welling and juxta-articular demineralization, uniform joint space narrowing, erosions, or even subluxation. Acute gouty arthritis has an extremely sudden onset, and it very commonly occurs at night. Precipitating factors include excessive alcohol intake, changes in medications, or fasting episodes for surgical procedures. The most commonly affected joint is the metatarsalphalangeal joint of the great toe, sometimes referred to as 'podagra'; attacks can occur in feet, ankles or knees as well. The affected joints are progressively swollen and exquisitely tender as well the overlying skin being tense, warm and dusky red. The patient will commonly also have a fever of 39°C or higher. Radiographic changes of gouty arthritis early on may show no changes; later on, 'rat bite' changes of the bone occur. These are punched-out erosions, with an overhanging rim of cortical bone.

Case A 50-year-old obese woman presents with severe left knee pain. She states the pain began about 8 months ago, but it has gotten significantly worse in the last 3 months. The patient denies any trauma or event that initiated the pain. She notes stiffness in the knee first thing in the morning; it only lasts around 5 - 10 minutes. The knee pain worsens with activity and is relieved with rest. Physical examination findings reveal a woman with a BMI of 40, limited range of motion of the left knee, and crepitus. Question What is the most likely diagnosis?

Correct answer: Osteoarthritis Explanation This patient is most likely suffering from degenerative joint disease, which is also known as osteoarthritis (OA). Common characteristics of OA include onset after the age of 40; obesity is recognized as a common risk factor for the development of OA. Other components seen in patients with OA include fairly rapid progressive onset of pain that is exacerbated by activity and relieved at rest. Morning stiffness is common in patients with OA, although it resolves within a 30 minute time period. Physical examination findings include limited range of motion of the affected joint and crepitus; the patient in this case has both. Patients may also have tenderness to palpation of the joint, joint effusion, or even palpable osteophytes. Osteoporosis and osteopenia may be present in this patient's population group, although they are much more common in older women that have experienced menopause. Occurrence of osteopenia or osteoporosis is higher in Caucasian women with other risk factors, such as tobacco or alcohol abuse, excess/deficiency in hormones, malignancy, and even some genetic disorders. Both disease states are typically asymptomatic until an actual fracture occurs. Loss of height may also be seen. Rheumatoid arthritis (RA) has a more insidious onset and presents with characteristic morning stiffness that symmetrically affects small joints of the hands and feet. It usually presents in a polyarticular manner, whereas OA presents as monoarticular. RA involves the metacarpophalangeal joints as well as the wrists, while OA affects the distal interphalangeal joints and potentially even the first metatarsophalangeal joints. Acute gouty arthritis has an extremely sudden onset and very commonly occurs at night. Precipitating factors include excessive alcohol intake, changes in medications, or fasting episodes for surgical procedures. The most commonly affected joint is the metatarsalphalangeal joint of the great toe, which is sometimes referred to as 'podagra'; attacks can occur in the feet, ankles, or knees as well. The affected joints are progressively swollen and exquisitely tender; the overlying skin is tense, warm, and dusky red. The patient will commonly also have a fever of 39°C or higher. References

Case A 63-year-old woman presents with a 2-year history of knee pain; it has been getting worse. She reports the pain is bilateral, but it is worse on her left side. She describes the pain as deep and "achy"; it is located in the front and back of her knees. Her pain increases after long days on her feet, and it is somewhat relieved with over-the-counter pain relievers and rest. She denies other joint problems, other than occasionally "over doing it". The patient denies trauma to her knees. She has no known medical conditions, and she takes no regular medications. She has never had surgery. She works in food service for the local school district. On physical exam, her BMI is 42. The other vitals are normal. Both knees exhibit crepitus and some decreased range of motion in flexion and extension. No erythema, edema, or warmth is noted in the joints. Lachman, McMurray's, Apley grind, and anterior and posterior drawer tests are negative. The remainder of her exam is normal. An X-ray is obtained, and it shows narrowed joint spaces and some subchondral sclerosis. Question What is the most likely diagnosis?

Correct answer: Osteoarthritis Explanation This patient most likely has osteoarthritis (OA) of her knees. Osteoarthritis is often described as the "wear-and-tear" (degenerative) arthritis, with a chronic aching pain and slow, progressive course. Bursitis (or inflammation of the bursa) can affect the knees. It can be acute or chronic; it can be caused by trauma, infection, or even osteoarthritis. Swelling is associated with bursitis. This patient's exam and description of the pain are much more consistent with OA. Gout is a metabolic condition that causes acute joint pain. Typically, a single joint, most commonly one of the metatarsophalangeal joints, is affected. Gout can affect multiple joints, but the distribution is often asymmetric. Affected joints are swollen, erythematous, and exquisitely tender. A meniscal tear can cause knee pain, but it is typically acute pain that follows a twisting or hyperextension event in athletes. The history may also include problems with clicking or locking in the knee joint. The McMurray's and Apley grind tests are often positive with meniscal tears. Rheumatoid arthritis (RA) is a chronic, inflammatory polyarthritis. RA tends to occur in smaller joints, often symmetrically, although knee joints can be involved. Patients with RA will often report morning stiffness in the joints, with improved mobility and less pain with movement. In later stages, nodules and deformity of the joint can be observed. A torn anterior cruciate ligament (ACL) will typically occur following an injury, with soccer and skiing injuries as the most common. The patient may initially note swelling, pain, and a feeling of instability or "giving out". The Lachman and anterior drawer test are helpful in evaluation of a torn ACL.

Case Ico-delete Highlights A 6-year-old boy has gradually increasing sharp pain to his left anterior hip that seems worse at night. He is moderately overweight, but he remains active and plays baseball. There is no history of trauma. There is no redness, swelling, or fever; there is no involvement of the other hip. Ibuprofen does seem to help. There is also no family history of any bone or joint disease. His weight is at the 95th percentile with height at the 50th percentile. On exam, there is mild tenderness to palpation over his left anterior hip, but the rest of the exam is normal. Laboratory studies show a normal complete blood count, hemoglobin, sedimentation rate, platelets, and C reactive protein. Plain X-rays show a small round lytic lesion surrounded by mild reactive bone formation. Question Based on these findings, what is the most likely diagnosis?

Correct answer: Osteoid osteoma Explanation Osteoid osteoma is a common benign bone tumor consisting of an ovoid or spherical nidus of osteoid-rich tissue with interconnected bone trabeculae on highly vascularized connective tissue. It ranges from 0.5 to 2 cm in size. It does not progress or have malignant potential. In fact, some lesions may regress spontaneously. Multinucleated giant cells, osteoclasts, and axons are frequently observed. There may be bone sclerosis around the central nidus. Most are cortical or periosteal, but 20% arise within the marrow. Most patients are 5 to 20 years old. Most commonly, the proximal femur and tibia are involved, although any bone may be affected. Pain is usually the presenting symptom and is frequently relieved with non-steroidal anti-inflammatory drugs (NSAIDs). Tenderness upon palpation may be elicited on exam; patients may also have disuse atrophy, painful scoliosis, or limb-length discrepancy. X-rays are the initial examination of choice to reveal the central nidus as a small spherical or oval lytic lesion surrounded by soft-tissue edema and possible reactive bone formation. Computed Tomography (CT) is also used for precise localization and as guidance for biopsy and ablation. The primary goal of treatment is pain control with NSAIDs; however, if pain is not controlled or complications such as neuropathy, synovitis, growth disturbance, or scoliosis develop, the lesion should be removed. Several techniques are available for ablation, including percutaneous radiofrequency, ethanol, laser, or thermocoagulation therapy under CT guidance. Surgical options include en bloc excision, curettage, or CT guided removal. Although small lytic lesions can be caused by infections or other benign neoplasms of the bone and leukemia, the history, physical findings, small size of the lesion, and location in the diaphysis make osteoid osteoma the most likely diagnosis. Ewing sarcoma is a highly malignant primary bone tumor derived from bone marrow. Males are affected more frequently than females, and it most commonly presents in children and adolescents aged 4-15 years. It is especially infrequent in African Americans and Asians. Metaphyseal or diaphyseal areas of long and flat bones are both affected. The earliest symptom is intermittent pain that becomes intense and may radiate to the limbs, particularly with tumors in the vertebral or pelvic region. X-rays will show a long, permeative lytic lesion in the metadiaphysis and diaphysis of the bone with a prominent soft tissue mass extending from the bone, or it will show a lesion with poorly defined margins that is destroying the bone. Occult metastatic disease is usually present, requiring multi-drug chemotherapy as well as local disease control with surgery and/or radiation. Length of treatment varies depending on location and staging, but most patients undergo chemotherapy for 6-12 months and radiation for 5-8 weeks. Legg-Calve-Perthes disease, a childhood hip disorder, results from infarction with idiopathic avascular necrosis of the bony epiphysis of the femoral head. It occurs bilaterally in 10-20% of children, and they usually present at 4-8 years of age. Children present with limp, pain, and reduced hip range of motion. Possible causes include trauma and developmental, inflammatory, and coagulation abnormalities. Risk factors include male gender, low socioeconomic group, presence of inguinal hernia or genitourinary tract anomalies, and low birth weight. Early radiological signs most commonly show a small femoral epiphysis, sclerosis of the femoral head with sequestration and collapse with fragmentation, slight widening of the joint space due to thickening of the cartilage, failure of epiphyseal growth, and the presence of joint fluid or joint laxity. Magnetic Resonance Imaging is more sensitive in detecting early disease. Treatment aims at maintaining containment of the femoral head within the acetabulum with abduction splints, casts, or surgically with an osteotomy of the proximal femur. Osteomyelitis is an infective process that may involve all osseous components including the metaphyses of long bones, particularly the tibia or femur. Hematogenous spread most commonly occurs in infants and young children. Neonates have group B streptococci and Escherichia coli as the most commonly involved organisms, with 50% also having a septic joint. Beyond the neonatal period, the most common organisms involved are Staphylococcus aureus, group A streptococci, and Haemophilus influenzae type b. Infections in this age group may gain access from traumatic injuries or surgical procedures. Joint involvement is rare beyond the neonatal period. Untreated or inadequately treated infections can become chronic, with staphylococcus isolated most frequently. Clinical presentation of hematogenous osteomyelitis can be variable, ranging from acute pain with decreased movement of the affected limb and adjacent joint to possible swelling and redness. Systemic signs can include fever, malaise, and irritability. Following trauma onset may be insidious; despite localized pain, edema, and redness, the frequent absence of systemic signs can misleadingly suggest a superficial infection. Laboratory studies may show an elevated or normal leukocyte count where sedimentation rate and C-reactive protein are typically elevated. A blood culture is positive in 50% of cases. Diagnosis is based on the history and physical exam findings supported by a positive technetium-99 methylene diphosphonate bone scan. X-ray changes show swelling of tissue around the infection and periosteal reaction or new bone formation, but these may not be evident until 1 to 2 weeks after onset of symptoms. Eventually, bone destruction with lytic lesions will be visible. Successful treatment requires an appropriate antibiotic at sufficiently high dose and duration. For staphylococcal infections, the most common cause of osteomyelitis, a penicillinase-resistant semisynthetic penicillin is the drug of choice. Primary lymphoma of bone (PLB) is a rare malignant neoplastic disorder. Most cases result from non-Hodgkin lymphoma. PLB tumors produce osteoclast-stimulating factors that cause lytic bone destruction. It is rare in children younger than 10 years of age and fairly evenly distributed in the 2nd through 8th decades. The usual presenting symptom is prolonged pain with possible swelling. Diagnostic criteria include a primary focus in a single bone, histologic confirmation, and no evidence of distant soft tissue or lymph node involvement. The most common location is the thoracic spine, but diffuse spinal involvement is also quite frequent. Treatment involves radiation therapy and chemotherapy. Surgery is indicated only when a large mass causes acute cord compression.

Case Ico-delete Highlights A 33-year-old Caucasian man with a history of severe Type I Diabetes has come in to see you due to a red and swollen pinky toe on his left foot. Further details from the patient include that he was running to answer his phone in his home and stepped on an exposed carpet tack. This occurred about 2 days ago. He washed it out and covered it as soon as he could. Since then, he has noted a significant increase in redness and severe restriction of movement of this toe; these characteristics have worsened tremendously in the last 12 hours. In addition, he admits to "feeling feverish" for the past day and experienced significant night sweats the previous night. He denies formally measuring his temperature. He has tried to take acetaminophen 500 mg, 2 tablets every 8 hours without any noticeable relief. Pertinent physical examination findings include an oral temperature of 102.8° F, significant erythema and edema extending the entire 5th toe of the left foot that is accompanied with restricted range of motion due to tenderness and swelling. There is a small amount of purulent drainage at the site of the wound. Question Given the history and physical examination, what would be the most likely diagnosis for this patient at this time?

Correct answer: Osteomyelitis Explanation It is highly likely that the patient above has an acute case of osteomyelitis. In simplest terms, this is an infection of the bone involved. Various etiologies could lead to this condition that may include direct trauma, an infection from somewhere else in the body, a chronic ulcer or wound, or even bacteremia. It has been noted that specific patient populations are more prone to develop osteomyelitis: these include diabetics, hemodialysis patients, sickle cell disease patients, IV drug abusers, or even elderly patients. Symptoms that should make a provider suspicious of osteomyelitis includes pain and/or tenderness of the affected area/bone, swelling and warmth of the area, significant fever, general discomfort, drainage of purulent material from the site, as well as sweating and chills. Degenerative joint disease, also known as osteoarthritis (OA), is an incorrect choice. Components seen in patients with OA include progressive onset of pain that is exacerbated by activity and relieved at rest. OA signs and symptoms are usually not found in an acute time period, but rather over months to years of progressing pain of the affected joint. Morning stiffness is common in patients with OA, although it will resolve within a 30 minute time period. Physical examination findings include limited range of motion of the affected joint and crepitus, both present in this patient. Patients may also have tenderness on palpation of the joint, joint effusion, or palpable osteophytes. Psoriatic arthritis is an incorrect choice for the patient described above. Psoriatic arthritis has distinct signs and symptoms including but not limited to: psoriasis preceding the arthritis, "sausage" appearance of finger and toes, symmetric involvement, and nail pitting. Acute gouty arthritis has an extremely sudden onset and very commonly occurs at night. Precipitating factors include excessive alcohol intake, changes in medications, or fasting episodes for surgical procedures. A specific injury does not initiate the signs and symptoms of gouty arthritis. The most commonly affected joint is the metatarsalphalangeal joint of the great toe, sometimes referred to as "podagra"; attacks can occur in feet, ankles, or knees. The affected joints are progressively swollen and exquisitely tender, while the overlying skin becomes tense, warm, and dusky red. The patient will commonly also have a fever of 39°C or higher. Although some factors are similar to the patient above, his case is most consistent with the diagnosis of SA. Osteopenia is a much more common diagnosis in older women that have experienced menopause; occurrence is higher in Caucasian women that possess other risk factors, such as tobacco or alcohol abuse, excess/deficiency in hormones, malignancy, and even some genetic disorders. Both disease states are typically asymptomatic until an actual fracture occurs. Loss of height may also be seen.

Case A 76-year-old Caucasian woman presents to the emergency department after a fall. Upon further questioning, she states she has a history of rheumatoid arthritis that has been treated with steroids. Her doctor tells her that she has broken her hip. X-ray shows demineralization. Her doctor is concerned and runs some additional tests. A bone scan is done and shows reduced bone density. Her BMI is 21 kg/m2. Her laboratory results are included in the chart. TEST RESULTS REFERENCE RANGE Calcium 9.3 mg/dL 8.4-10.2 mg/dL Phosphorus 3.5 mg/dL 3.0-4.5 mg/dL Alkaline phosphatase 45 UL 30-115 U/L ESR 10 mm/hr 0-20 mm/h Urinary hydroxyproline 42 mg/day 25-75 mg/day Question What is the most likely diagnosis?

Correct answer: Osteoporosis Explanation This woman has a history suggestive of osteoporosis. She developed a pathologic hip fracture secondary to the osteoporosis. Both the glucocorticoids she is on and the rheumatoid arthritis could have also contributed to her low bone mass. With osteoporosis, there will be normal serum levels of calcium, phosphorous, and alkaline phosphatase. The chemical composition of the bone is normal but the density is reduced. Osteoporosis is asymptomatic until a fracture occurs. Paget's disease is also called osteitis deformans. Serum calcium would be normal, but alkaline phosphatase and urinary hydroxyproline would be elevated and X-rays would show dense and expanded bones. The lab results with hyperparathyroidism would show an elevation of serum calcium. Alkaline phosphatase would be normal to elevated. Lab results are normal with osteoarthritis, but there would not be a reduced bone density with osteoarthritis.

Case A 34-year-old man presents for evaluation of 10/10 left calf pain after being run over by a car. Physical exam reveals tense calf swelling, increased pain with passive muscle stretching, and tenderness on calf palpation. Question What other findings are expected in this patient?

Correct answer: Pain out of proportion to exam Explanation This patient presents with acute compartment syndrome after a crush injury. Compartment syndrome occurs when excessive muscle swelling increases the pressure within the fascial sheath compartment, compressing vessels and nerves. Compression of the artery leads to ischemia. Compartment syndrome is a medical emergency and can result in the loss of the limb if the pressure is not relieved immediately. Compartment syndrome is a clinical diagnosis based on the 6 Ps (pain out of proportion, pressure on palpation, paresthesia, paralysis, pallor, and pulselessness). In addition to the 6 Ps on PE, diagnosis is confirmed by obtaining the compartment pressure, in which anything above 10 mm Hg is abnormal. At 20 mm Hg, ischemia can occur, and anything above over 30 mm Hg requires an emergent fasciotomy. Streaking erythema is a sign of lymphangitis, which is inflammation or infection that has spread to the lymphatic system. A palpable calf cord can be appreciated in patients who present with a DVT or varicose veins. Lymphadenopathy is swelling of a lymph node secondary to infection, autoimmune disease, or malignancy.

Question A 48-year-old woman presents with pain on the lateral aspect of her right dominant wrist. It began 3 days ago after playing video games with her son. She does not remember any specific injury, but she had to use a pencil to figure her taxes. She has pain now trying to hold a pencil to write. What physical findings would you most likely find?

Correct answer: Pain with the Finkelstein test Explanation Flexion of the thumb into the palm along with ulnar deviation of the wrist (Finkelstein test) will increase the pain of the lateral wrist due to stretching of the tendons of the 1stdorsal compartment (the extensor pollicus brevis and the abductor pollicus longus), which are swollen and tender. Tenderness over the dorsocentral wrist is more likely due to a ganglion cyst or ligament sprain/tear. Edema is over the 1st dorsal compartment and NOT the thumb flexors. The ulnar nerve is medial and not associated with this disorder. Erythema of the palmar thumb is more likely due to flexor tendon sheath infection of a felon.

Case A 56-year-old Caucasian woman presents to the emergency department for evaluation of chest pain. Vital signs upon presentation are normal and the oxygen saturation level is 96% on room air. Initial lab work performed in the emergency department shows a normal troponin and normal D-dimer. EKG shows normal sinus rhythm without STT wave changes. The patient is admitted to your service for observation of chest pain. Two additional serial troponins are normal, and a follow-up EKG remains unchanged. Chest X-ray is unremarkable. Cardiac monitoring has revealed normal sinus rhythm with rare PVCs. You now decide to reassess the patient prior to making further decisions regarding additional testing or hospital discharge. The patient reports several months of substernal chest pain that is aggravated by exertion or movement. The pain can last several hours at a time and is described as sharp and moderate in intensity. It is not aggravated by deep breathing. She denies associated shortness of breath, nausea, vomiting, diaphoresis, and lightheadedness. She has never had a cardiac stress test. Past medical history: Cholecystectomy 5 years ago. No prior history of cardiac disease, hypertension, dyslipidemia, or diabetes. Social history: Patient denies recent travel or surgery. Family history: Her father had an MI at age 72. Cardiac exam shows a nondisplaced point of maximal impulse (PMI) with a normal S1 and S2 and a regular rate and rhythm. Lungs are clear to auscultation and percussion. Question What physical examination procedure would most likely reveal her diagnosis?

Correct answer: Palpation for sternal and costochondral joint tenderness. Explanation The correct answer is palpation for sternal and costochondral joint tenderness. Costochondritis is inflammation of the ribs and cartilaginous material of the chest wall and is a common cause of chest pain. The chest pain is often aggravated by exertion, movement, or respiration. To establish a diagnosis of costochondritis, palpation of the chest wall should reproduce the patient's presenting symptoms of chest pain. Costochondritis may coexist with other illnesses, so potentially life-threatening causes of chest pain still should be ruled out, especially in moderate- or high-risk age groups. Assessment for calf tenderness to palpation and dorsiflexion is incorrect. This test has traditionally been used to evaluate for deep venous thrombosis but lacks sensitivity and specificity. Lower extremity swelling is the most common physical exam finding. Patients with DVT are at increased risk for pulmonary embolism. Risk factors for deep venous thrombosis and pulmonary embolism include recent surgery, immobilization, cancer, or prior DVT. Pulmonary embolism is more likely in the presence of tachycardia, hemoptysis, prior history of PE, or an abnormal D-dimer test. In the absence of any of these manifestations, the likelihood of deep venous thrombosis of pulmonary embolism is decreased. Assessment of aortic width and pulsation is incorrect, but it is indicated in evaluating for abdominal aortic aneurysm. This patient's symptoms are unlikely to be due to abdominal aortic aneurysm because AAA rarely causes chest pain, but it is occasionally associated with abdominal discomfort. Most often, an abdominal aortic aneurysm is asymptomatic unless there is an associated dissection or rupture. Evaluation of brachial and femoral pulses is incorrect. Asymmetrical brachial or femoral pulses may be noted in the presence of aortic dissection; however, aortic dissection (especially thoracic) is typically associated with sudden onset of severe chest pain with a tearing or ripping quality. Patients with aortic dissection are typically described as writhing and restless. This patient's chest pain is unlike that associated with aortic dissection. Evaluation for pulsus paradoxus is incorrect. Pulsus paradoxus (the disappearance or diminution of the pulse during inspiration) may be identified at the bedside and aid in the early diagnosis of cardiac tamponade. The normal fall in blood pressure during inspiration is less than 10 mm Hg. Clinical manifestations of cardiac tamponade may include tachycardia, dyspnea, muffled heart sounds, jugular venous distention, hypotension, or shock. The chest X-ray may show an enlarged cardiac silhouette. This patient's clinical features do not suggest cardiac tamponade.

Case A 16-year-old girl presents with severe right knee pain; it started during a softball game 3 days ago after the patient swung to hit the ball. She reports hearing a popping sound before the pain began. There is no significant past medical history. Examination of the right knee reveals a positive 'apprehension sign'; Lachman, McMurray, valgus, and varus tests are negative. X-rays show a lateral tilt of the right patellar and subluxation. Question What is the most likely diagnosis?

Correct answer: Patellar dislocation Explanation Patellar dislocation is commonly seen with a history of twisting of the torso when the foot is planted. The positive apprehension sign, in conjunction with the X-ray findings, are diagnostic for this disorder only. Anterior cruciate ligament tear is incorrect; there would be increased anterior laxity with Lachman test, which was negative. Medial collateral ligament tear is incorrect; there would be pain and laxity with valgus testing, which was negative. Medial meniscal tear is incorrect; the McMurray test was negative, and it would have elicited pain at the medial joint line. Patellofemoral pain syndrome is incorrect; it does not include instability or dislocation of the patellar, which is evident on X-ray.

Case A 34-year-old woman presents Monday morning with knee pain. She states that she was playing with her two children in their backyard over the weekend and fell onto solid ground, landing directly on her knees. She noted immediate and significant right knee pain. She notes significant swelling; it accompanies the knee pain, which she rates as an 8/10 on a numerical pain scale. Physical examination reveals significant obvious joint effusion and exquisite focal tenderness to palpation over the patellar area of the right knee; the left knee has no obvious abnormalities. Results of the anterior drawer, McMurray, and varus/valgus stress testing are within normal limits. Question Given the history and physical examination findings, what is the most likely diagnosis?

Correct answer: Patellar fracture Explanation The patient above has most likely experienced a right patellar fracture. Many fractures of the patella are the result of direct forces, such as striking the dashboard in a motor vehicle accident, or in the patient's scenario, falling onto a flexed knee. Patellar fractures should be high on the differential diagnosis list if a patient presents with acutely swollen knee and patella pain following trauma to that area. There will be presence of joint effusion and focal tenderness to palpation of the patella. Another feature is that the patient will be unable to extend the knee against gravity, which requires surgical repair if present. Our patient had all these signs and symptoms, minus the inability to extend the knee. A bone cyst is described as a fluid-filled hole that develops inside a bone. These most commonly occur in children and young adults and very rarely cause significant symptoms. The cause of bone cysts is unclear, but they usually will not cause serious health issues. Tibial fractures are caused by very similar mechanisms of injury and present with similar signs and symptoms. If a tibial fracture has occurred, there is more often than not a ligamentous or meniscal injury in conjunction with the tibial fracture. With these issues present, the patient would not even be able to bear the various tests to assess these structures, and the patient would have positive findings when completing the anterior drawer and/or Lachman test (assesses ACL), the McMurray test (assessing meniscal injury), or the varus and valgus stress tests, which assess the medial and lateral collateral ligaments. Femoral fractures are not commonly caused by the type of injury described in the patient scenario. The femur is the longest, strongest, and heaviest bone in the body, and along the same lines is the principle load-bearing bone in the lower extremity. If a femoral shaft fracture does occur, it is almost always the result of high energy forces, such as in motor vehicle collisions, being struck by a motor vehicle, motorcycle accidents, falls from height of ≥3 meters, or gunshot wounds. Presentation is typically substantial in its signs and symptoms: pain, swelling, deformity, or even shortening of the thigh. Ligament tear is low in the differential diagnosis as well. The patient never mentioned a "pop" at the time of injury, which is extremely common and indicative of a ligament injury. With this injury, patients will have acute swelling and a feeling that the knee will give out and is unstable. Almost all will develop knee effusion from hemarthrosis. The anterior drawer is seen as one of the three tests that are most sensitive and specific when diagnosing ACL tears during the physical examination. In the case of the patient above, there were no abnormal results.

Case A 32-year-old woman with no significant past medical history presents with a 3-month history of right anterior knee pain described as a dull and aching pain that is "right under the kneecap." Provocative activities include bending movements, descending stairs, and performing squatting maneuvers. Pain is relieved during rest. She notes that she loves the outdoors and her hobbies include running and hiking; her symptoms began following a run. She denies a history of falls, prior surgeries or instrumentation, fever, chills, malaise, myalgias, changes in weight, joint swelling, skin changes or rashes, or other joint pains. Her physical exam is normal with the exception of a tender undersurface of the patella, with crepitus upon passive range of motion of the right knee. There is abnormal patellar tracking upon right knee flexion and apprehension of the patient upon passive manipulation of the patella. Additionally, there is a positive patellar grind test. McMurray's, Lachman, the anterior and posterior drawer, Apley's compression and distraction tests, and varus/valgus tests are all negative. There is no joint line tenderness, effusion, or restriction of range of motion of the right knee. Question What is the most likely diagnosis?

Correct answer: Patellofemoral pain syndrome Explanation This patient exhibits signs and symptoms of patellofemoral pain syndrome, also known as chondromalacia patellae, miserable malalignment syndrome, and runner's knee. This syndrome describes any pain involving the patellofemoral joint. The pain affects any or all of the anterior knee structures, including the medial and lateral aspects of the patella, as well as the quadriceps and patellar tendon insertions. The patella engages the femoral trochlear groove with approximately 30° of knee flexion. Forces on the patellofemoral joint increase up to 3 times body weight as the knee flexes to 90° (e.g., climbing stairs), and 5 times body weight when going into full knee flexion (e.g., squatting). Abnormal patellar tracking during flexion can lead to abnormal articular cartilage wear and pain. When the patient has ligamentous hyperlaxity, the patella can sublux out of the groove, usually laterally. Patellofemoral pain is also associated with muscle strength and flexibility imbalances, as well as altered hip and ankle biomechanics. Patients usually complain of pain in the anterior knee with bending movements and less commonly in full extension. Pain from this condition is localized under the kneecap but can sometimes refer to the posterior knee or over the medial or lateral inferior patella. Symptoms may begin after a trauma or after repetitive physical activity, such as running and jumping. When maltracking, palpable and sometimes audible crepitus can occur. Intra-articular swelling usually does not occur unless there are articular cartilage defects or if osteoarthritis changes develop. Patellar mobility can be assessed by medially and laterally deviating the patella (deviation by one-quarter of the diameter of the kneecap is consider normal; greater than one-half the diameter suggests excessive mobility). The apprehension sign suggests instability of the patellofemoral joint and is positive when the patient becomes apprehensive when the patella is deviated laterally. The patellar grind test is performed by grasping the knee superior to the patella and pushing it downward with the patient supine and the knee extended, thus pushing the patella inferiorly. The patient is asked to contract the quadriceps muscle to oppose this downward translation, with reproduction of pain or grinding being the positive sign for chondromalacia of the patella. Iliotibial band syndrome typically presents with a lateral "snapping" with flexion and extension of the knee. Patients with ACL tears hear an audible "pop" and usually fall down following the injury. They also have acute swelling, difficulty with bearing weight, and instability. Positive Lachman and anterior drawer tests are observed. Medial collateral ligamentous injuries usually occur as a result of a valgus stress to the partially flexed knee. It can also occur with a blow to the lateral leg. The MCL is commonly injured with acute ACL injuries. The patient may have a limited range of motion due to pain, especially during the first 2 weeks following the injury. A positive valgus stress test is expected. Injuries to a meniscus can lead to pain, clicking, and locking sensation. Most meniscus injuries occur with acute injuries (usually in younger patients) or repeated microtrauma, such as squatting or twisting in older patients. Clinical findings include an antalgic gait, difficulty with squatting, catching, or locking of the meniscal fragment, effusion, and joint line tenderness. Patients can usually point out the area of maximal tenderness along the joint line. The McMurray test is usually present.

Question A 19-year-old boy presents with pain and deformity of his right dominant shoulder after a sudden jerking movement to the same from a wrestling competitor approximately 1 hour ago. He states he felt a clunking sensation when it happened. He was unable to continue wrestling and has pain with movement of the right shoulder. After returning from radiology, you review the films and diagnose a glenohumeral dislocation. After reducing the dislocation, what part of the physical exam MUST be done again?

Correct answer: Perform a neurologic check of the affected upper extremity Explanation Axillary nerve palsy may develop with reduction of a glenohumeral dislocation. It is imperative that both pre- and post-reduction neurological exams be performed. Never do an apprehension test following a reduction or you may have to repeat the reduction. The arm should be placed in a sling after confirming the reduction with repeat radiographs (AP and transscapular views of the shoulder). Vigorous shoulder motion should be delayed for 3 months in anyone under 30 years old, but may begin at 6 weeks for those over 30 years old. Motion check of the elbow is important to do, but is unlikely to change from pre-reduction to post-reduction of the shoulder. Inspection of the skin for open wounds should have been performed at presentation and is unlikely to change with a reduction maneuver. References: Andrews JR. Shoulder Instability. In: Griffin LY, ed. Essentials of Musculoskeletal Care, 3rded. Rosemont, IL: American Academy of Orthopaedic Surgeons; 2005:214. Nobuhara K. The Shoulder: Its Function and Clinical Aspects. World Scientific.2003.Pg 319,320. Reichman E. Emergency Medicine Procedures.McGraw-Hill Professional.2004.Pg 608.

Case An 18-year-old man with no significant past medical history presents with pain and swelling over the upper right knee for 5 months. The pain was initially insidious, dull, and achy. Over the past several weeks however, it has gradually become progressively more severe and unremitting, often waking the patient up from sleep during the night. He also notes increased swelling, warmth, and erythema. He denies a history of injuries, accidents, trauma, surgeries, or sexual encounters. His physical exam reveals a noticeable limp, reduced right knee range of motion, and localized tenderness and swelling to the distal anterior femur. Question What is the most appropriate next step in the management of this patient?

Correct answer: Perform a right knee radiograph Explanation This patient's presentation is suspicious for the malignant bone tumor known as osteosarcoma. As with all patients, evaluation of those with sarcoma should begin with a careful history, physical examination, and routine laboratory tests, followed by imaging tailored to the given complaint. The imaging of any bone tumor should begin with a plain film of the involved area. Such an X-ray image is often helpful in the diagnosis of bone sarcomas; for example, osteosarcoma often has a 'sunburst' appearance of calcification on X-ray imaging, which is virtually diagnostic Once the diagnosis has been established, a chest X-ray, CT scan of the chest, and bone scan should be performed to evaluate for metastatic disease. As with soft tissue sarcomas, the most common site of metastasis from bone sarcoma is the lungs; therefore, a chest X-ray and CT chest are warranted. Physical therapy, analgesics with observation, and anti-gout medications should not be used until diagnostic testing is employed to identify the contributory disorder.

Question A 42-year-old woman presents with pain and swelling of her right elbow following a fall on her outstretched hand with her elbow locked in full extension. She had immediate pain and limited range of motion (ROM) of her elbow. Upon exam, her elbow is swollen. X-rays reveal a type-II radial head fracture. What diagnostic procedure can be done to determine if a mechanical block is present, limiting pronation and/or supination of the elbow?

Correct answer: Perform full active and passive ROM Explanation Assessing the patient's active and passive ROM is essential to determining if a mechanical block exists. Passive ROM will require the injection of an anesthetic, as the patient will resist until given the injection into the joint. A CT will show bony fracture definition but not soft tissue interposed. An MRI shows tendons and ligaments around the shoulder and knee. An EMG and a nerve conduction study would only be used to identify nerve injury.

Case A 72-year-old woman, with a past medical history of COPD, deep venous thrombosis, breast cancer in remission, hyperthyroidism, and premature ovarian failure, presents with sudden-onset severe lower back pain of 2 hours duration. She states that the pain began upon stepping out of her bed onto the floor. She denies any falls, prior injuries, genitourinary complaints, or lower extremity numbness, tingling, paresthesias, or weakness. She takes a daily prednisone tablet for COPD. Her social history is notable for a sedentary lifestyle and a 40-pack-per-year smoking history. Her physical exam reveals a tall, thin-framed woman with noticeable lid lag. Her thoracic vertebral body is exquisitely tender at T12, and she has limited mobility of all thoracolumbar planes of motion. Lateral and AP lower back X-ray findings are shown in the following image. Question What is the most appropriate intervention for this patient at this time?

Correct answer: Refer the patient for a dual-energy X-ray absorptiometry test Explanation The correct response is to refer the patient for a dual-energy X-ray absorptiometry test. This patient's symptoms, coupled with the radiographic images, suggest a diagnosis of an osteoporotic vertebral compression fracture of the T12 vertebral body. Risk factors for primary osteoporosis include early menopause or premenopausal estrogen deficiency states, prolonged periods of inadequate calcium intake, a sedentary lifestyle, and tobacco and alcohol abuse. Secondary osteoporosis results from chronic conditions that contribute significantly to accelerated bone loss (e.g., endogenous and exogenous thyroxine excess, hyperparathyroidism, cancer, gastrointestinal diseases, medications, renal failure, and connective tissue diseases). The most widely used techniques for assessing bone mineral density (BMD) are dual-energy X-ray absorptiometry (DXA) and quantitative computerized tomography (CT). These methods have errors in precision of 0.5 - 2%. Quantitative CT is more sensitive, but results in substantially greater radiation exposure than DXA; for this reason, DXA is the diagnostic measure of choice. Vertebral fracture assessment is now available through imaging completed with the DXA assessment and provides a more accurate assessment of the patient's bone density. Increasing prednisone would not be beneficial in this patient as corticosteroids, along with aluminum, anticonvulsants, chemotherapeutics, cyclosporine A, tacrolimus, lithium, methotrexate, proton pump inhibitors, selective serotonin reuptake inhibitors, and thiazolidinediones, are all medications known to contribute to osteoporosis. Specific biochemical markers (i.e., human osteocalcin, bone alkaline phosphatase, immunoassays for pyridinoline crosslinks, and type 1 collagen-related peptides in urine) that reflect the overall rate of bone formation and bone resorption are now available. These markers are primarily of research interest and are not recommended as part of the basic workup for osteoporosis. They suffer from substantial biological variability and diurnal variation and do not differentiate causes of altered bone metabolism. The patient's history of breast cancer and history of a thromboembolic disorder preclude any estrogen replacement therapy for osteoporosis, as these are absolute contraindications for its use.

Case A young mother brings in her 5-week-old daughter for a routine checkup. Pregnancy and delivery had been uneventful. The infant is doing fine, but the mother tells you that she has problems spreading the girl's left leg when she is changing her diaper. You suspect a dislocated hip. Question What should be looked for in order to determine whether or not the infant has a dislocated left hip?

Correct answer: Physical exam Explanation Congenital dislocation of the hip is more common in female infants, infants with a positive family history, and breech in utero positioning. In infants, physical exam with a series of provocative tests is the best way to make the diagnosis. Since the major sign of hip dislocation or subluxation is the Ortolani's test, the inability to completely abduct the thigh to the surface of the examining table when the hip and knee are flexed will be present on the left side. This is not typically a painful test and should not produce discomfort in the patient. The Trendelenburg's sign is positive in persons with congenital hip dislocation and pelvis abnormality. Gluteal skins folds are also often asymmetrical on the side with the dislocation. Hip X-rays are very difficult to interpret, especially at an early age, so they are not the method of choice for determining whether or not an infant has a dislocated hip. Hip ultrasounds are very accurate in early diagnosis. There would also be an increase in skin fold on the affected side. Similarly, CT scan and MRI are not typically used to make the diagnosis.

Case A 72-year-old man presents with pain, rated 5/10, in his right upper arm. He heard a pop and then felt sudden pain when it occurred. On exam, there is ecchymosis of the upper and middle arm; there is also a visible, palpable bulge proximal to the elbow. Although a relatively active and healthy patient for his age, he does have a longstanding history of impingement syndrome in the same arm. Question What treatment is indicated for this patient?

Correct answer: Physical therapy Explanation Range of motion and strengthening exercises are the best treatment for a ruptured proximal biceps tendon in an elderly patient as there is minimal loss of function. Surgical repair of the torn tendon is indicated in young, athletic patients. Cortisone injections would not be helpful, and debridement is not necessary. Pain is usually mild; therefore, narcotics are not needed.

Question A 17-year-old male high school football player (punter) presents after being tackled and slammed onto his right dominant shoulder forcefully 2 hours ago during a game. He had immediate pain but was able to continue punting. He has full active and passive range of motion, but some pain (6/10) with abduction. There is no obvious deformity and the skin over the shoulder is intact and not tented. What initial radiographic procedure should be ordered?

Correct answer: Plain film radiograph Explanation Assessment of acute shoulder injuries may be difficult. Standard (plain film) radiography is used to consider presence or absence of injury and differentiate acromioclavicular versus other shoulder injuries in the setting of acute injury. A minimum of two views is necessary to evaluate the injury. Views may consist of Anterior-Posterior, lateral, axillary, or scapular Y views.

Case A 75-year-old woman with a past medical history of hypertension, hyperlipidemia, and obesity presents with insidious but progressive bilateral knee pain for the past several months. She states that her pain is worsened with movement and relieved by rest. She reports "cracking" sounds to her knees upon ambulation but has minimal stiffness lasting 10 minutes. Her physical exam is remarkable for bilateral knee crepitus, joint line tenderness, and an antalgic gait; there is no swelling, erythema, or warmth noted. Question What is an expected diagnostic test result in this patient?

Correct answer: Plain radiograph subchondral sclerosis Explanation This patient is presenting with signs and symptoms consistent with osteoarthritis. Knee radiography is typically expected to reveal joint space narrowing (especially of the medial compartment), thickened, dense subchondral sclerosis, osteophytes, and bony spurring. Other findings include subchondral cyst formation and lipping of marginal bone. Osteoarthritis does not cause elevation of the erythrocyte sedimentation rate (ESR) or other laboratory signs of inflammation. The earliest radiographic findings of rheumatoid arthritis occur in the hands or feet, and consist of soft tissue swelling and juxta-articular demineralization. Later, diagnostic changes of uniform joint space narrowing and erosions develop. Compensated polarized light microscopy identifies and distinguishes monosodium urate (gout, negatively birefringent) and calcium pyrophosphate (pseudogout, positive birefringent) crystals. The synovial fluid white cell count discriminates between noninflammatory (< 2000 white cells/mcL [2.0 × 109/L]), inflammatory (2000-75,000 white cells/mcL [2.0 × 109/L-75.0 × 109/L]), and purulent (> 100,000 white cells/mcL [> 100 × 109/L]) joint effusions. The synovial fluid aspirate of osteoarthritis is noninflammatory.

Case A 28-year-old woman presents to the emergency department with a right ankle injury; it occurred approximately 30 minutes ago, while sliding into base during a softball game. She notes pain and an inability to bear weight on the affect extremity. The patient denies other injuries. Her past medical history is unremarkable, with no known medical conditions, no allergies, no medications, and no prior surgeries. She denies the use of alcohol, tobacco, and drugs. She has not been sexually active for 1 year, and she reports a normal menstrual period within the last 2 weeks. On physical exam, the patient appears in some discomfort; she is lying on the hospital bed. Vitals are normal. The right ankle appears visually deformed, with dislocation of normal foot anatomy. Edema is prominent. The patient is acutely tender over the lower leg, especially at the lateral and medial malleoli. There is no tenderness with palpation of the calf and proximal limb. Distal to the injured ankle, pulses, coloration, temperature, sensation, and capillary refill are normal. The patient has normal range of motion in the right knee, and she can wiggle her toes. She is unable to move the ankle joint. Due to patient discomfort, stability tests of the joint are not performed. The skin is intact over the injured area. The left lower limb and the remainder of her physical exam are normal Question What test should be obtained next?

Correct answer: Plain radiographs (X-ray) Explanation This patient is presenting with a dislocated ankle fracture, most likely affecting both the lateral malleolus (of the fibula) and the medial malleolus (of the tibia). Plain radiographs (X-ray) of the ankle are indicated as the initial tests to obtain, and they will help to differentiate a severe sprain versus fracture. The X-ray will often show bone fractures quite well. Additionally, the cost and availability of plain radiographs make them a wise first choice. Arthroscopy is a method of visualizing the interior of the joint. It can be utilized in the painful ankle for osteochondral fractures, syndesmotic injuries, and synovitis. It would best be utilized during open surgical reduction, however; it is not a first-line test in the emergency room, prior to evaluating bony anatomy. Magnetic resonance imaging (MRI) may be used in ankle evaluation for soft tissues, cartilage, and bone disorders. It may show ligament rupture or dislocation and damage to interosseous membranes. MRI is much more costly, less readily-available, and should be obtained only in cases of ankle injuries in which the diagnosis is not clear from plain radiographs and history, or in cases when the injury is not resolving with adequate treatment. Technetium scintigraphy (bone scan) can be useful in the ankle in determining stress or occult fractures, infections and malignancy. It would not be used for an acute fracture, as is the case in this patient.

Case Ico-delete Highlights A 24-year-old man presents due to increasing pain in his right groin and buttock and difficulty walking. The pain has been present and worsening for about 1 month. He further reports that he had quite significantly injured himself, also near his right hip in a fall, while snowboarding about 3 months ago. He did not seek care immediately and had difficulty ambulating for a week, then saw a chiropractor to "put his lower back back in". The pain and gait problems eventually improved. Now he is concerned because the pain has returned with no new history of trauma. He is otherwise healthy and takes no regular medicines. He denies any chronic health conditions and has an unremarkable family history for musculoskeletal and rheumatologic conditions. On physical exam, the patient was noted to walk into the exam room with a slight limp. Examination of the hip, buttock, and groin region reveals no edema, erythema, or ecchymosis. The exam does not produce one particular point of tenderness with palpation, but pain is elicited with right hip motions, particularly internal rotation. Question What diagnostic study, if found to show the "crescent sign", would confirm this patient's condition?

Correct answer: Plain radiography (x-ray) Explanation This patient's condition is suspicious for avascular necrosis (or the more preferred term, osteonecrosis) of the hip. Avascular necrosis (AVN) is a relatively common complication, following a traumatic hip dislocation, when the vascular supply to the femoral head is damaged and bone death occurs. There are many theories about the exact mechanism, and other causes, such as vascular, congenital, and autoimmune diseases, have been implicated with osteonecrosis. Plain radiography (x-ray) is the preferred initial test, and if the "crescent sign" is seen, AVN is confirmed. The crescent sign is a lucent (dark-appearing) crescent-shaped region of the femoral head, usually just a sliver beneath a more opaque (white-appearing) bone edge, which indicates some sclerosis and bone structure collapse upon itself. Arthroscopy is a surgical procedure for viewing the joint through a small endoscope, introduced via a small incision. Arthroscopy can be done for a variety of hip joint disorders (as well as other joints), but does not have a strong role in osteonecrosis evaluation. The crescent sign is not associated with arthroscopy. Bone biopsy plays a role in the diagnosis and staging the extent of a suspected osteonecrosis. Characteristic findings can include dead bone cells and degenerative changes. A nuclear bone scan can be very helpful in diagnosing and staging osteonecrosis. In all but the mildest cases of osteonecrosis, a bone scan will show increased uptake at the bone site (often the femoral head). Ultrasound does not typically play any role in AVN.

Question A 54-year-old man presents to your clinic with a 2-month history of left non-dominant shoulder pain. There is no history of trauma, but the pain began about a week after shoveling wet heavy snow from his 100-foot driveway. At first the pain seemed to come and go from day to day, but has gotten progressively worse and more constant. Pain is worse with overhead use and he can't sleep on his left side and will wake up if he rolls over onto his left shoulder. You diagnose the patient with Impingement Syndrome. Which of the following physical exam findings led you to your diagnosis?

Correct answer: Positive Neer and Hawkins signs Explanation Neer and Hawkins signs are positive in impingement syndrome. Drop arm test is positive in rotator cuff tear. Tenderness is over the greater tuberosity of the proximal humerus and not the AC joint (unless associated with arthritis of the AC joint). Sulcus sign is seen with glenohumeral joint instability. Atrophy, not edema, is common with this disorder. Reference: Andrews JR. Impingement Syndrome. In: Griffin LY, ed. Essentials of Musculoskeletal Care, 3rd ed. Rosemont, IL: American Academy of Orthopaedic Surgeons; 2005:188.

Case A 52-year-old woman, with a body mass index of 35, presents for evaluation of back pain that has been present for several months. The pain becomes worse at night and is accompanied by stiffness. For the past week, she has been experiencing night sweats and fever. She relates that she had a positive PPD test a year ago and did not follow up as directed. An X-ray of her lumbar spine reveals osteopenia, lytic and sclerotic lesions, and cortical breakdown of vertebral bodies L4 and L5. Question What is the most likely diagnosis?

Correct answer: Pott Disease Explanation The clinical picture is suggestive of tuberculosis of the spine (Pott disease). The fact that this patient had a positive PPD and did not follow her treatment appropriately indicates that she has active tuberculosis of the spine. Clinical findings include fever, sweating, or prostration. Pain may be mild at onset, is usually worse at night, and may be accompanied by stiffness. Imaging of the lower spine may find bone atrophy, narrowing of the cortex, lytic and sclerotic lesions, and enlargement of the medullary canal. Cauda equina syndrome will cause back pain but will have accompanied loss of neurologic function (bowel and bladder) and paresthesias. Radiographic findings with osteoarthritis include narrowing of the joint space, osteophyte formation, sharpened articular margins, and lipping of marginal bone. Clinical findings of compression fracture secondary to obesity or osteoporosis include loss of height and backache of varying degrees of severity, which are not present in this patient.

Question A 66-year-old man presents with monoarticular arthritis. He has stage 3 chronic kidney disease. His affected joint is warm to the touch. You suspect gout. He is allergic to aspirin. What anti-inflammatory should you prescribe?

Correct answer: Prednisone Explanation In this case, you are looking for an anti-inflammatory agent that will not cause a decline in renal function. Prednisone is recommended. Urate-lowering therapy such as allopurinol may also be initiated. Aspirin has anti-inflammatory properties, but it is contraindicated because the patient is allergic to it. Acetaminophen could be used for analgesia, but it is unlikely to decrease inflammation. Metformin is a medication used for the treatment of diabetes. Ibuprofen is an anti-inflammatory and antipyretic agent. Ibuprofen will likely cause a decline in renal function; it and other non-steroidal medications block prostaglandin-mediated vascular relaxation, effectively vasoconstricting and decreasing glomerular filtration rate and renal function. Approximately 30% of patients exposed to non-steroidal medications will have a decline in renal function. Patients with pre-existing kidney disease may be more susceptible to a decline in function, as they may be more dependent on the above-described prostaglandin-medicated vascular tone.

Case A 52-year-old woman presents with left hip pain. There is no known history of trauma to the area. Her past medical history includes Crohn's disease, type II diabetes, and hypertension. She is currently taking metformin, metoprolol, and mesalamine; she recently completed a prednisone taper for her Crohn's disease. She also finished a course of ciprofloxacin, which she took for a urinary tract infection. An X-ray is obtained, and it reveals a collapsed left femoral head. Question What medication likely contributed to her current condition?

Correct answer: Prednisone Explanation Prednisone is correct. The patient has a collapsed left femoral head on radiography, suggesting avascular necrosis. Avascular necrosis is commonly associated with the use of corticosteroids. Metformin, metoprolol, mesalamine, and ciprofloxacin are incorrect responses. These medications are not commonly associated with avascular necrosis.

Case A 40-year-old recreational tennis player presents with pain in his playing-side elbow since a tournament last weekend. He played 5 matches of singles and 2 matches of doubles games. He cannot even hold a pen without pain. He says he needs to get better fast because he has another tournament coming in a week. On exam, he has pain with extension against resistance of his affected side wrist. Question What treatment option should be recommended on this first visit?

Correct answer: Prescribe an NSAID of choice and recommend skipping the next tournament. Explanation Activity modification and NSAIDs are first line treatment for lateral epicondylitis. This may have to include changing to a lighter racket, changing the tension in the strings, or overwrapping the grip to make it slightly larger. Use of commercially available "tennis elbow" straps may be beneficial as well. Once the acute pain phase subsides, gentle stretching and strengthening exercise will be the key to resolution and prevention. Muscle relaxant medications have no role in treatment of this disorder. If symptoms persist despite conservative treatment, injection of a corticosteroid to the point of maximal tenderness at the lateral epicondyle may be helpful as well.

Case A 32-year-old man presents with pain in his back, buttocks, and posterior thighs for 2 days after lifting a heavy load at work. He denies pain while sleeping, unless he rolls over in bed. He also denies pain while standing still or sitting still in a chair. He also denies any radicular symptoms or bladder or bowel dysfunction. Question What is the appropriate treatment for this man?

Correct answer: Prescribe an NSAID, moist heat, and decreased physical activity Explanation For the patient with acute low back pain (LBP) without any neurological deficits, treatment is in two phases: the symptomatic relief phase and the gradual return to activity phase. For the symptomatic phase, pain management can be initiated with aspirin, acetaminophen, or a NSAID of choice. Narcotics, steroids, and muscle relaxants are not recommended for first-line pain management in this type of patient. Phase two begins as the pain begins to improve, and it should include referral to a physical therapist for an exercise program of aerobic activity. Spine surgeon referrals should be for patients with neurological compromise only.

Case An anxious-looking young mother brings a 6-week-old female infant to your office wrapped in many blankets. When your nurse unwraps her, she appears lethargic; she has dusky and mottled skin and very weak pulses. As your staff quickly provides ventilation and compressions, you prepare to establish vascular access. Question What is the best site for immediate vascular access for this infant?

Correct answer: Proximal anteromedial tibia Explanation Insertion of an intraosseous needle directly into the anterior tibia is relatively easy and will ensure rapid vascular access. This infant's clinical presentation of decreased activity, mottled and dusky skin, and weak pulses suggests severe volume depletion. Whatever the cause—in this emergent situation that can be assessed later—rapid fluid resuscitation and possibly pressors are indicated. A peripheral scalp vein would be very difficult to cannulate in this volume-depleted infant. Blood would be shunted away from the periphery to more vital organs. Also, these small caliber veins would not allow a large bore needle for rapid fluid resuscitation. The saphenous vein, though larger than scalp veins, would also be difficult to cannulate due to shunting. The distal radius is a site chosen for intraosseous access in the adolescent. In an infant, it is not yet fully developed. The sternum is a site used for intraosseous access in the adult and is part of a system called First Access for Shock and Trauma (FAST) and works by a hand driven, push-pull mechanism

Case An 8-year-old boy sustained a puncture wound to his right foot 4 days ago. He was playing and stepped on a nail that went through his sneaker. His mother said the wound bled profusely but the nail did not go completely through his foot. They washed the wound at home with soap and water, wrapped it in a bandage, and did not seek further care. This morning, he complained that it was very painful and his mother noted that his foot looked red and swollen. On exam, his temperature is 99°F; pulse is 114 BPM, and his BP is 104/68 mm Hg. The plantar surface of his right foot has a small 2 mm scabbed entry wound that is surrounded by a 5-6 cm area that is erythematous, swollen, and quite tender. There is a scant amount of thin, seropurulent material from the entry wound on examination. You are worried about possible osteomyelitis. Question What bacteria would most likely cause this complication?

Correct answer: Pseudomonas Explanation Pseudomonas aeruginosa causes more than 90% of foot osteomyelitis in plantar puncture wounds. It should be considered as a likely pathogen when there is a penetrating injury through footwear, especially sneakers or from soil and manure contamination. It is believed that the penetrating object may push particles of foam contaminated with bacteria into the wound. In addition, anaerobes, Klebsiella, Bacteroides, Serratia, and Salmonella sp. are less common organisms in this setting that may also cause secondary infections. Streptococci (as well as Staphylococci) are the most common causes of secondary infection of ordinary puncture wounds. Secondary infections will occur in 6%-10% of non-bite puncture wounds in addition to retention of a foreign body. Eikenella should be considered as a possible dog bite pathogen after staphylococci and streptococci. Pasturella is a common dog and cat bite pathogen. P. multocida is responsible for 60%-80% of infections following a cat bite. Though dog bites are 8-10 times more common than cat bites, the rate of infection is much greater from cat bites, approximately 10%-20% for dog bites and 50% from cat bites. Clostridium is a common anaerobic bacterium that may complicate human bites along with Streptococci and Staphylococci. Eikenella corrodens should also be considered and is found primarily in adults but can also be a significant pathogen in children. Due to high risk of secondary infection, prophylactic antibiotics to cover Gram-positive and anaerobic bacteria are generally recommended following deep bite wounds through the dermis or to the hand, along with close follow-up.

Case A 12-year-old girl presents to the emergency room with worsening foot pain. 2 weeks ago, she stepped on a nail while wearing rubber soled tennis shoes. The area was thoroughly cleaned, but she has developed worsening pain, redness, and drainage from the area. X-ray shows periosteal changes at the site of the wound. A wound culture is obtained. Question Which organism is most likely to grow on culture for this specific mechanism of injury?

Correct answer: Pseudomonas aeruginosa Explanation Pseudomonas aeruginosa is correct. Pseudomonas aeruginosa is the most common cause of osteomyelitis when a penetrating object passes through the sole of the shoe. Staphylococcus aureus is incorrect. While Staphylococcus aureus is the most common cause of osteomyelitis overall, Pseudomonas aeruginosa is the most common cause when the infection has been caused by a puncture wound through the sole of a shoe. Escherichia coli, Serratia marcescens, and Salmonella enterica are incorrect. All of these organisms may cause osteomyelitis, but Pseudomonas aeruginosa is the most likely cause in this patient.

Case A 28-year-old man is brought in after being involved in a motor vehicle accident. The patient was the driver in the car and was hit head on during a wrong way crash. After initial survey of ABCs is done and the patient is ruled stable, a secondary evaluation reveals that the patient's left ankle is painful. During inspection, it is found that there is localized pain and swelling over the medial aspect of the left ankle. The patient is able to walk with this ankle, but a mild limp is present. Pulses are palpable. Question Based on the history and physical examination, what is the best clinical intervention at this time?

Correct answer: RICE approach Explanation Eversion ankle sprains will present with localized pain and swelling over the medial aspect of the ankle only. In this type of sprain there is a tear of the deltoid ligaments on the medial side of the ankle. The patient will likely have difficulty weight bearing, but he or she would still attempt to walk, although he or she would most likely limp. Sprains in general are related to injuries involving the stretching or tearing of a ligament and may be approached as a Grade I (Mild), Grade II (Moderate), or Grade III (Severe). Most likely the injury above is a Grade I Mild ankle sprain. Treatment of this type of injury would consist of using the RICE method: rest, ice, compression, and elevation. Following these actions would most likely allow this injury to heal efficiently and effectively. I mmediate physical therapy on its own would not be optimal treatment at this point in time; if the patient appropriately uses the RICE method, improvement may be seen as soon as 24 -48 hours after the initial injury. Observation only would most likely not allow the injury to heal as well. An emergent orthopedic referral is not needed at this time; there are no visible or substantial musculoskeletal injuries and the pulses are present. A cast placement is not necessary unless there is the presence of a fracture or a Grade III type of strain.

Question A 24-year-old woman enjoys participating in an amateur karate class. While taking part in a tournament, she uses her right upper arm to block a high roundhouse kick and is injured. She develops a right wrist drop. What nerve was injured?

Correct answer: Radial nerve Explanation A radial nerve injury can cause a wrist drop. The radial nerve innervates the skin and posterior surface of the upper extremity. The radial nerve innervates the muscles of the back of arm, forearm, and hand. If the muscles of the posterior compartment of the forearm are paralyzed, as occurs with radial nerve damage, there will be wrist drop. The ulnar nerve innervates the skin of the medial third of the hand. The ulnar nerve innervates some of the flexors in the anterior forearm and most of the intrinsic muscles of the hand. The median nerve innervates the skin on the lateral 2/3 of the hand and fingers on the palmar side and the dorsum of digits 2, 3, and some of 4. Most of the muscles of the anterior forearm and some of the muscles of the hand are innervated by the median nerve. The axillary nerve innervates some to the skin in the shoulder region and the back of the arm. The deltoid and teres minor are innervated by the axillary nerve. The musculocutaneous nerve innervates some of the skin on the lateral forearm. The muscles innervated by the musculocutaneous nerve are the brachialis, biceps brachii, and coracobrachialis.

Question A patient complains of pain during medial rotation of the humerus but not during abduction or extension of the shoulder. What muscle is injured?

Correct answer: Subscapularis Explanation The pectoralis major flexes, adducts, medially rotates, and is a minor extensor of the shoulder. The teres minor is a lateral rotator. The latissimus dorsi extends and medially rotates. The deltoid abducts and can assist in flexion, extension, and medial and lateral rotation. Only the subscapularis medially rotates and is not involved in extension and abduction. Therefore, this is the injured muscle.

Case A 10-year-old boy presents with a possible arm fracture. While playing baseball at school, he slid into another student and hit his upper arm on the other student's leg. An X-ray of the patient's right arm reveals that he has a fracture at the distal third of the humerus. You also note that the patient cannot extend his wrist. Question What structure has most likely been injured

Correct answer: Radial nerve Explanation The clinical picture is suggestive of an injury to the radial nerve. The characteristic sign of injury to the radial nerve is wrist-drop: the inability to extend the wrist and the digits of the metacarpophalangeal joints. Injury to the brachial artery would result in contraction of the digits and sometimes the wrist. Since there is collateral circulation, symptoms of ischemia to the distal nerves and muscles may take up to 6 hours to present. Injury to the musculocutaneous nerve would result in weakening of elbow flexion and supination of the forearm. Injury to the median nerve would result in loss of flexion of the PIP joints in digits 1 to 3. Injury to the ulnar nerve can result in sensory loss in the hand, and patients may have difficulty making a fist because they cannot flex their fourth and fifth digits at the DIP joint.

Question Parents bring their 4-year-old daughter in because of knock-knee. She is otherwise healthy, and her height is in the 50th percentile for age. On examination, she has about 10 degrees of valgus. What should you recommend?

Correct answer: Reassurance that this is normal for age Explanation Children are normally bowlegged (genu varum) at birth. By 12 to 18 months of age, the legs become straight. Then children develop knock-knee (genu valgum), which is maximal by about age 3-4 years. This averages 10-15 degrees at this age. The legs then spontaneously "straighten" to 5-10 degrees of valgum, which is the average for adults. This child is within the normal range, and the parents should be reassured. Because virtually all of these children improve spontaneously, they do not need special shoes or braces. Because this child is not short, she does not require additional work-up such as X-rays of the lower extremities or laboratory tests for serum phosphorus, vitamin D levels, calcium, and parathyroid hormone levels. If she was short and had other physical findings, rickets would be in the differential diagnosis and further work-up would be indicated.

Case A 33-year-old man with no significant past medical history presents with a 2-month history of persistent right knee pain. The knee pain is located in the anterior part of the knee, but according to the patient, "behind the knee cap." Pain is worse as he descends stairs, performs squatting maneuvers, and sits for excessive periods of time. He is an avid runner and states that running also increases pain. He denies any trauma, falls, accidents, or prior surgeries. He further denies any fever, chills, insect bites, rashes, effusions, grinding, popping, or clicking sensations in the knee. He denies any hip or ankle pain. The physical exam reveals tenderness to palpation along the medial undersurface of the right patella and a positive patellar apprehension test. The anterior and posterior draw tests, McMurray's test, and Apley's compression and distraction tests are all negative. Question What is the most appropriate clinical intervention for this patient at this time?

Correct answer: Reduce running exercises Explanation This patient's most likely diagnosis is patellofemoral pain syndrome. Clinical interventions include relative rest, a reduction of running pace and mileage, and avoidance of squats, lunges, and running uphill and downhill. Other interventions include quadriceps strengthening and NSAID medications. Surgical interventions are indicated if symptoms persist beyond 6 months, if a rehabilitation program fails, and if other causes of knee pain have been excluded. Weight loss is recommended to decrease the stress on the patellofemoral joint. Knee orthotics may be beneficial.

Case A 28-year-old woman presents to the emergency department with a right ankle injury; it occurred approximately 30 minutes ago while sliding into base during a softball game. She notes pain and an inability to bear weight on the affect extremity. The patient denies other injuries. Her past medical history is unremarkable, with no known medical conditions, no allergies, no medications, and no prior surgeries. She denies the use of alcohol, tobacco, and drugs. She has not been sexually active for 1 year, and she reports a normal menstrual period within the last 2 weeks. On physical exam, the patient appears in some discomfort; she is lying on the hospital bed. Vitals are normal. The right ankle appears visually deformed, with dislocation of normal foot anatomy. Edema is prominent. The patient is acutely tender over the lower leg, especially at the lateral and medial malleoli. There is no tenderness with palpation of the calf and proximal limb. Distal to the injured ankle, pulses, coloration, temperature, sensation, and capillary refill are normal. The patient has normal range of motion in the right knee, and she can wiggle her toes. She is unable to move the ankle joint. Due to patient discomfort, stability tests of the joint are not performed. The skin is intact over the injured area. The left lower limb and the remainder of her physical exam are normal. Imaging is obtained and shown in the image below. Question What intervention represents the best choice for this patient's condition?

Correct answer: Reduction, stabilization, and referral for surgical fixation Explanation This patient is presenting with a dislocated ankle fracture, affecting both the lateral malleolus and the medial malleolus. (The posterior malleolus could also be fractured, but the given image does not allow visualization of that structure.) A closed, acute fracture involving both the lateral and medial malleoli is rated at 100% instability, and surgical treatment is recommended. In this patient's case, her fracture should be treated with reduction (as soon as possible), stabilization, and referral for surgical fixation. Application of splint, referral for delayed reduction, and surgical fixation would risk further edema and reduced distal blood circulation. With marked deformity, the patient typically experiences great pain, and cartilage may be damaged by a sharp bony edge. Provisional reduction should occur immediately, with application of a well-padded splint afterward, until surgical fixation can occur. For an ankle sprain, it may be appropriate to recommend a compression wrap, with early ambulation that is assisted by crutches. However, this patient's joint is unstable and these recommendations would only worsen her condition and risk permanent deformity. An undisplaced fracture of either malleolus does not require surgical treatment, and it may be reasonable to recommend application of short-leg walking cast. However, if the fracture was clearly dislocated on exam, it should be reduced and evaluated for possible surgical repair. For some simple, single-bone, dislocated closed fractures, it may be appropriate to recommend reduction, application of hard cast and non-weight-bearing for 6 - 8 weeks. As noted above, the resulting instability of fractures of both the distal tibia and fibula necessitate surgical fixation.

Case A 73-year-old man presents with the inability to actively raise his left non-dominant arm to retrieve plates from the kitchen cabinet. He further describes the inability to retrieve any objects with his left hand/arm because of the limited range of motion. He is worried because he is the only driver in his household, and he doesn't want to lose his driving privileges. He has a long-standing history of chronic shoulder impingement syndrome. On exam, he is found to have a (+) positive drop arm test. Question What initial treatment is recommended for the most likely diagnosis?

Correct answer: Refer for physical therapy Explanation Rotator cuff tears (RCT) are a common clinical issue in geriatric patients, and there has been a long debate on how to approach restoring shoulder function in these patients. Treatment options usually consist of nonsurgical and surgical options. Many times the majority of patients are initially placed on a trial of conservative therapy which would include physical therapy. Refer this elderly patient with a rotator cuff tear to physical therapy to attempt regaining range of motion by stretching and strengthening the remaining rotator cuff. If the patient is younger or an active athlete and experiences a sudden rotator cuff tear, they should be referred to an orthopedic surgeon for immediate repair. There is little chance this gentleman will need repair. CT of the shoulder would not be an appropriate test to order at this time. It may show the tear, but little clinical information will be gained unless he becomes a candidate for surgery. Injection of a steroid may improve his painful symptoms, but it also increases the risk of worsening the tear by weakening the tendons. Do NOT place this elderly patient in a shoulder immobilizer, as doing so will decrease the chance of obtaining the return of function by making the shoulder joint stiff.

Case A 30-year-old Japanese woman presents with dizziness and weakness. On examination, her blood pressure is 90/60 mm Hg. Her upper extremities are cold. Both of her radial pulses are very weak. On further questioning, her relative says that she has had such several attacks in the past. The angiography reveals narrowing of the brachiocephalic and subclavian arteries. Question What is the most likely diagnosis?

Correct answer: Takayasu's arteritis Explanation Takayasu's arteritis is a granulomatous vasculitis of medium and larger arteries. Takayasu first described it in 1908 as a clinical syndrome characterized by ocular disturbances and marked weakening of the pulses in the upper extremities (Pulseless disease). This is related to the fibrous thickening of the aortic arch with narrowing, or virtual obliteration, of the origins or more distal portions of the great vessels arising in the arch. Microscopically, there is adventitial mononuclear infiltrate with perivascular cuffing of the vasa vasorum (initially). Later, there is intense mononuclear inflammation in the media, in some cases accompanied by granulomatous changes replete with giant cells and patchy necrosis of the media. In addition, when it heals, the inflammation is replaced by marked collagenous fibrosis involving all the layers of the vessel wall accompanied by lymphocytic infiltration. Giant cell arteritis, or temporal arteritis, is the most common of the vasculitides. It is an acute and chronic granulomatous inflammation of the medium and small arteries. It affects mainly the temporal, vertebral, and ophthalmic arteries. The short segments of one or more arteries develop nodular thickening with the reduction of the lumen, which may become thrombosed. Histologically, there is granulomatous inflammation of the inner half of the media, centered on the internal elastic membrane. It is marked by mononuclear infiltrate, multinucleated giant cells of both foreign body and Langhans-type, and fragmentation of the internal elastic lamina. Sometimes the granulomas may be absent, and there is only nonspecific polyarteritis without giant cells. The healed stage of both will reveal only collagenous thickening of the vessel wall. Kawasaki disease is an acute arteritis involving large, medium, and small arteries (often the coronary arteries) and is associated with mucocutaneous lymph node syndrome. The mucocutaneous syndrome is characterized by fever, conjunctival and oral erythema, edema of the hands, feet, erythema of the palms and soles, a skin rash (often with desquamation), and enlargement of the cervical lymph nodes. It is usually self-limited. Approximately 20% develop cardiovascular sequelae, ranging from asymptomatic vasculitis of the coronary arteries, coronary artery ectasia, or aneurysm, progressing in severity to giant coronary artery aneurysm (7 to 8 mm) with rupture or thrombosis, myocardial infarction, or sudden death. Acute fatalities occur in 1% of the cases due to coronary artery thrombosis or rupture of coronary artery aneurysm. Vasculitis resembles polyarteritis nodosa with necrosis and pronounced inflammation, affecting the entire thickness of the vessel wall. Polyarteritis nodosa is systemic vasculitis characterized by necrotizing inflammation of the small or medium vessels, typically involving renal arteries and visceral vessels but sparing the pulmonary circulation. There is neither glomerulonephritis nor vasculitis of the arterioles, capillaries, and venules. It particularly involves the branching points, also known as the points of bifurcation. The involvement is segmental and may involve only a portion of the circumference. It causes segmental erosion with weakening of the arterial wall with aneurysmal dilatation or localized rupture. Initially, there is transmural inflammation of the vessel consisting of neutrophils, eosinophils, and mononuclear infiltrate with fibrinoid necrosis. Later, the inflammation is replaced by collagenous fibrosis. Within the same vessel, various stages of inflammation may be seen. Microscopic polyangiitis is also called hypersensitivity, or leukocytoclastic vasculitis. It generally affects arterioles, capillaries, and venules. The lesions are thought to represent a hypersensitivity reaction that involves the skin, mucous membranes, lungs, brain, heart, gastrointestinal tract, kidneys, and muscle. Necrotizing glomerulonephritis and pulmonary capillaritis are common. The lesions are histologically similar to polyarteritis nodosa, but muscular and large arteries are spared. Histologically, segmental fibrinoid necrosis of the media may be present; however, in some, the change is limited to infiltration with neutrophils. Greater than 80% of the patients have ANCA (anti-neutrophil cytoplasmic antibodies), most often p-ANCA. In many cases, reaction to an antigen such as drugs (penicillin), microorganisms (streptococci), heterologous proteins, or tumor antigens can be traced as the precipitating cause, but there are few or no immune deposits in this type of vasculitis. Simple removal of the offending agent may help most patients with cutaneous vasculitis, but those with systemic disease may develop organ failure unless treated. Refer to the table for additional information.

Case A 27-year-old man is admitted to the hospital following a motor vehicle accident. He sustained lacerations to his arms bilaterally and has fractures of the right tibia and fibula. A cast is placed and the patient is scheduled for surgery the following day. A few hours after the cast is placed, he develops severe pain; the pain is unresponsive to several doses of intravenous morphine. His pain increases when he extends his right leg. Peripheral pulses are weak but present. Question Based on the history and physical, what should be done next?

Correct answer: Remove cast and check compartment pressure. Explanation The patient should have the cast removed and compartment pressure measured. This patient is showing signs of compartment syndrome. Compartment syndrome develops when there is an accumulation of pressure within a muscle compartment. It typically occurs following trauma to a limb. Patients typically present with pain unrelieved by analgesia as well as pain with extension of the involved limb. As the swelling within the compartment worsens, patients develop absence of pulse, paresthesias, pallor, and poikilothermia. The compartment pressure should be checked; if it is elevated, the patient should be taken for fasciotomy. An X-ray of the tibia and fibula would not demonstrate compartment syndrome. An MRI of the right leg would not be useful in this patient. An ultrasound of the lower extremity would be useful in diagnosing a lower extremity DVT. However, the patient's presentation is highly suggestive of compartment syndrome, and it should be ruled out before pursuing any other diagnoses. Monitor and continue giving narcotics is incorrect. If compartment syndrome is left untreated, it can lead to tissue death.

Case A 12-year-old pre-menarcheal girl is referred to you following a screening for scoliosis at school by the school nurse. The nurse used a scoliometer that revealed a reading >5° and felt radiographs were needed at this point. The girl is totally asymptomatic of any back pain or motion or activity limitations. You order a standing AP and lateral long cassette spine and use the Cobb method to measure your curvatures. You measure and find a thoraco-lumbar single curve of 20° with a Risser stage of 3. Question What is recommended for this patient?

Correct answer: Repeat radiographs in 3 months. Explanation Treatment for scoliosis is partly based on the physiologic maturity of the skeleton (Risser classification) and the curve. For progressive curves in a patient with Risser stage of 3, it is recommended that you repeat radiographs in 3 months. 1 year is too long to wait in an actively growing child. Bracing is reserved for curves >30° in a patient with Risser stage of 3. Surgical referral is not needed until idiopathic curves reach 50° or for curves of 40-50° that are likely to progress. This patient should be followed up with every 3 months until skeletally mature. Risser Classification (level of ossification and fusion of the iliac crest apophyses): stage 0: no ossification center at the level of iliac crest apophysis stage 1: apophysis under 25% of the iliac crest stage 2: apophysis over 25-50% of the iliac crest stage 3: apophysis over 50-75% of the iliac crest stage 4: apophysis over >75% of the iliac crest stage 5: complete ossification and fusion of the iliac crest apophysis

Case A 15-year-old boy presents for a routine physical. He reveals a 1-month history of mildly painful swelling of the anterior superior left shin. It has been unaccompanied by fever, erythema, or joint complaints. He is his high school football team's tailback. He has been assisting his father in caulking the hull of their new boat, which entailed considerable kneeling that worsened the pain in the affected area. Examination is unremarkable except for mild, slightly tender swelling of the left anterior shin approximately 5 cm below the knee. Radiographs of the left knee showed mild irregularity of the tibial tubercle. Question What treatment recommendation is appropriate in this case?

Correct answer: Rest, ice, wrapping, elevation, NSAIDs, physical therapy Explanation The diagnosis is Osgood-Schlatter disease (condition), which is an inflammation of the tibial tuberosity. Once the significant conditions are eliminated, treatment is supportive and symptomatic. This condition is not intra-articular; arthrocentesis is of no value. The condition is diagnosed clinically and radiographically. There is no need for a biopsy in the typical case. There is nothing in the presentation to suggest an actual arthritis or arthropathy. Repeat X-rays are of no value in the typical case and would be warranted only in the case of worsening physical findings and symptoms.

Case A 34-year-old woman presents with redness and swelling of some of the joints of her hands. She notes that the symptoms are worse in the morning. On physical examination, redness and swelling of her PIP joints on both hands are noted. Her DIP joints are spared. Radiographic imaging of her hands reveals joint erosion and space narrowing. Her sedimentation rate is elevated and her rheumatoid factor is negative. Question What is the most likely diagnosis?

Correct answer: Rheumatoid arthritis Explanation The clinical picture is suggestive of rheumatoid arthritis (RA). RA characteristically includes small joint symmetrical arthritis with an elevated ESR, indicating an inflammatory process. Roughly 75-85% of patients have a positive rheumatoid factor. Radiographic imaging includes juxta-articular osteoporosis and joint erosion with space narrowing. Osteoarthritis would not have an elevated ESR, and radiological findings are not supportive of this diagnosis. Ankylosing spondylitis would have an elevated ESR and negative rheumatoid factor, but mainly involves the SI joint and lumbar/thoracic spine fusion. Dupuytren's contracture is characterized by the hyperplasia of the palmar fascia with nodule formation and contraction of the palmar fascia. Fibromyalgia is seen frequently in women ages 20-50 years of age, but it presents with widespread musculoskeletal pain with multiple tender points; signs of inflammation

Case A 12-year-old boy is brought in by his mother following a fall on his right outstretched hand from the top half of a slide an hour ago. He denies any loss of consciousness. There is deformity, pain, and decreased motion of the elbow due to pain, as well as pain down the forearm into the wrist area. There is no shoulder or neck pain. Question Which of the following radiographs should be ordered?

Correct answer: Right elbow AP and Lateral, Right forearm AP and Lateral, Right wrist AP & Lateral Explanation AP and lateral views of the injured elbow should be taken. Because the patient is complaining of pain to the arm and wrist in case of the fall on his outstretched hand (FOOSH), AP and lateral views of the forearm and wrist should also be considered because of possible injury to these structures. A third view of the elbow, the lateral oblique view might be obtained and can help diagnose subtle lateral condyle fractures and assess displacement. It is not always obtained. In pediatric patients, there are multiple growth centers in the elbow. In children younger than 2.5 years it may be beneficial to view the contralateral elbow to compare the anatomic positions of the growth centers along with the injured side.

Question A patient presents with a motor deficit on the right side of the body, decreased sensation and pain on the left side of the body, and diminished vibratory and position sense on the right side of the body. What type of spinal cord lesion is present?

Correct answer: Right hemisection of spinal cord Explanation A hemisection of the spinal cord results in a Brown-Sequard's syndrome. The characteristic deficits of the Brown-Sequard's syndrome include a motor deficit and deficit of position sense and vibratory sense ipsilateral to the side of the lesion. There is a pain and temperature deficit that is contralateral to the side of the lesion. These deficits occur because the fibers in the cortical spinal tract and in the posterior columns decussate above the level of the spinal cord. The fibers that serve pain and temperature sensation decussate upon entering the spinal cord. The anterior spinal artery syndrome affects the spinal cord bilaterally and spares only the posterior columns. A lesion involving the posterior columns should spare both motor sensation and pain and temperature sensation bilaterally. Syringomyelia results in a central cord syndrome. Typically, motor sensation and pain and temperature sensation are affected first, with the upper extremities affected more severely than the lower extremities.

Case A 73-year-old man presents with inability to actively raise his left non-dominant arm to retrieve plates from the kitchen cabinet. This began a month ago after his shoulder pain improved. He had a history of pain in that shoulder for over 6 months that kept him from sleeping on the left side and the pain would wake him often. There was no specific injury he can recall although he felt a pop a month ago while taking out the trash. On inspection you notice the back of the shoulder appears sunken when compared to the other shoulder. Question What is the most likely diagnosis?

Correct answer: Rotator cuff tear Explanation The key here is the inability to actively raise the arm. Typically elderly patients with rotator cuff tears report a prodrome of chronic shoulder pain for several months and maybe a episode where they feel a pop when performing some heavier than usual activity (taking the trash out). A sunken shoulder indicates atrophy due to the tear. Impingement syndrome would be painful, but active motion is usually normal. Adhesive capsulitis is limited active and passive range of motion limitation. Rotator cuff tear has no passive range of motion limitation. Glenohumeral dislocation is painful with any motion, but the disorder is not with the rotator cuff, but the joint congruity. A proximal humerus fracture would have a history of trauma.

ase A 21-year-old man presents with a 3-month history of pain in his left shoulder. He is a left-handed pitcher for his college baseball team. The pain began when he started spring training a few months ago, and it has gradually worsened since that time. In addition to the pain, the patient is now having difficulty lifting his left arm above his head. He denies any injury to the shoulder itself, and he states that ibuprofen provides some relief. On physical exam, the patient has tenderness to palpation of the lateral left shoulder just under the acromion, limited abduction of the left shoulder, a negative "drop arm" sign, and a negative "crossover test". Question What condition best explains the patient's symptoms?

Correct answer: Rotator cuff tendinitis Explanation The patient above is most likely suffering from rotator cuff tendinitis. Distinguishing between the various soft tissue disorders that can cause shoulder pain (tendinitis, subacromial bursitis, and tears) can be difficult because these conditions can cause similar symptoms and can often co-exist. However, knowledge of musculoskeletal anatomy and physiology can help in making a more precise diagnosis. The muscles of the rotator cuff include the supraspinatus, infraspinatus, teres minor, and subscapularis, otherwise known as the SITS muscles. The tendons of 3 of the rotator cuff muscles form a musculotendinous unit where they insert into the proximal humerus. Repetitive movements of the shoulder, such as throwing or swimming, can lead to inflammation of the tendons of the rotator cuff muscles. The most commonly affected muscle is the supraspinatus, which inserts directly under the acromion. Therefore, a patient with a rotator cuff tendinitis, in addition to the inability to lift the arm above shoulder level (abduction and external rotation), will have tenderness with palpation of the area just under the acromion on the affected side. The "drop arm" sign is elicited by having the patient fully abduct the affected arm to shoulder level, or up to 90 degrees, and then lower it slowly. If the patient is unable to hold the arm fully abducted, this is a "positive drop arm" test. Though the "drop arm" sign is used to assess the status of the rotator cuff, a positive drop arm sign indicates an actual complete tear in the rotator cuff, not just tendinitis. The patient in the presentation above had a "negative drop arm" test. Bicipital tendinitis, inflammation of the long head of the biceps tendon, can also present with anterior shoulder pain that may resemble rotator cuff tendinitis and may coexist with it. However, in bicipital tendinitis, tenderness is usually maximal in the bicipital groove along the anterior aspect of the humerus. Inflammation or arthritis of the acromioclavicular (AC) joint of the shoulder is not a common cause of shoulder pain. However, when it is present, it is usually the result of direct injury to the shoulder with resulting degenerative changes. This condition usually presents with localized tenderness over the AC joint or pain with adduction of the affected shoulder. The "crossover test" is also used to assess the AC joint. Therefore, a "positive crossover test" is when adduction of the patient's affected arm across the chest elicits pain. The patient above had a "negative crossover test". Adhesive capsulitis usually occurs in patients 50 - 70 years of age, and it refers to fibrosis of the glenohumeral joint capsule. It is manifested by diffuse, dull, aching pain in the shoulder and progressive restriction of active and passive range of motion. There is usually no localized tenderness on exam.

Case A 35-year-old man presents with right shoulder pain that is becoming progressively worse. Although he visits the gym 3 times a week, over the past month he has not been able to increase the amount of weight he lifts secondary to the shoulder pain. He has not tried anything to alleviate the pain. He reports that the pain is at its worst at night while he is trying to sleep. He also reports pain while in the shower washing his hair, or when using the shoulder press machine at the gym. He denies any history of recent trauma or sports related injury. However, upon questioning, he reports that about 1 month ago he and his wife painted their entire house in 1 weekend. On exam, there is no notable swelling, atrophy, redness, or bruising of the shoulders. Point tenderness is noted over the right lateral deltoid muscle. Active ROM of the right shoulder at 80 degrees of abduction elicits pain. Patient has a negative drop arm test, negative apprehensive test, and a positive Neer impingement test of the right shoulder. Question What is the most likely diagnosis?

Correct answer: Rotator cuff tendonitis (tendinopathy) Explanation The most likely diagnosis is rotator cuff tendonitis (tendinopathy). It is associated with pain and difficulty abducting or rotating the arm. Degree of trauma depends on the type of injury. With full thickness tears, the degree of trauma is significant (e.g., a major fall, MVA, or shoulder dislocation). Partial thickness tears can be caused by lesser degree of trauma, and overuse injuries can cause tears, as well. In addition, tendonitis injuries are often associated with overuse. Symptoms are usually pain and weakness made worse by overhead activities, overhead abduction, and external rotation. Pain is usually located over the anterior and lateral aspects of the shoulder and may radiate into the lateral deltoid. The pain may worsen with sleeping on the affected extremity. The patient may describe a "catch" as the arm is brought into the overhead position. In addition, a typical painful arc usually occurs between 70 degrees and 110 degrees of abduction along with a positive impingement test, which is forced forward elevation of the arm. The pain results as the greater tuberosity impinges on the acromion. The drop test is associated with rotator cuff tears. Acromioclavicular sprain is common in younger persons and athletes. The classic cause is the direct blow to the acromion with the humerus in the adducted position. This drives the acromion medially and inferiorly. Falling on an outstretched hand or elbow can also cause this injury. A cross body adduction test is positive with this condition. Adhesive capsulitis is often associated with patients with diabetes and thyroid disorders. It can result from immobility following a shoulder injury. Patients have slow, gradual onset of shoulder pain that can be severe, especially at night. The pain is generalized and referred to the superolateral aspect of the shoulder and upper arm. Examination reveals no swelling at the glenohumeral joint. Passive and active ROM is greatly limited in all movements. Fracture of the clavicle is one of the most common injuries. It is usually caused by a fall on the lateral aspect of the shoulder. Less commonly, it is caused by a direct blow or by falling on the outstretched hand. Although complications are uncommon, they can occur. With this type of injury, lung and neurovascular compromise can occur. AP X-ray is the best way to ascertain a clavicle fracture. Glenohumeral dislocations in younger individuals are most commonly caused by direct trauma and sports injuries. Patients with this type of injury are usually unwilling to move the affected arm, and they tend to cradle it with the unaffected arm. Anterior shoulder dislocations account for about 80-90% of all dislocations. An excessive external rotation or abduction force usually causes this type of injury. A bulge may be noticeable where the humeral head rests, with an emptiness beneath the acromion. Posterior shoulder dislocation usually happens when the humeral head is driven posteriorly with force. Usually, the patient experiences a positive apprehensive test. References

Question A 55-year-old woman presents with weakness, bone pain, and lethargy. A 24-hour urine sample reveals a spike of M protein. Based on the most likely diagnosis, which additional finding on bone marrow aspirate would support the diagnosis?

Correct answer: Russell bodies Explanation Russell bodies are the accumulation of immunoglobulin in plasma cells. This is a characteristic finding in patients with multiple myeloma. Teardrop-shaped erythrocytes (dacrocytes) are commonly present in patients with myelofibrosis and myeloid metaplasia. These tear drop cells are found also in patients with beta thalassemia major. Lewy bodies are a characteristic feature present in the brains of patients with Parkinson's disease. It is also seen in a type of dementia called Lewy body dementia. A "starry sky" pattern is seen in patients with Burkitt's lymphoma. This is the result of macrophages engulfing cellular debris. Schistocytes are fragmented pieces of red blood cells. These are commonly found in patients with hemolytic anemia or microangiopathic diseases such as disseminated intravascular coagulation.

Case A 5-year-old boy presents with an ankle injury. While on his school's playground, he was playing on the swings and decided to jump off. An X-ray of the ankle demonstrates a fracture line through the metaphysic plate of the superior aspect of the distal fibula, ending before the epiphyseal plate. Question Ico-delete Highlights What type of Salter-Harris fracture does this child have?

Correct answer: Salter-Harris type II Explanation The injury described is classified as a Salter-Harris type II growth plate fracture. In a Salter-Harris type II fracture, the fracture lies above the growth plate, sparing the epiphysis. Salter-Harris is a system used to classify factors affecting the growth plate, or "physis." Five types have been characterized: Type I is a transverse fracture through the growth plate (physis). Type III is a fracture through the growth plate and epiphysis, sparing the metaphysis. Type IV is a fracture through all 3 elements of the bone: the growth plate, metaphysis, and epiphysis. Type V is a compression fracture of the growth plate resulting in a decrease in the perceived space between the epiphysis and diaphysis on X-ray.

Case A 4-year-old girl presents with a 3-day history of left elbow pain and swelling. The history reveals that 4 days prior she fell onto the elbow and sustained a superficial abrasion. The following day the elbow became red and swollen and the child began to complain of increased pain. On the day prior to presentation, the child had a fever of 103° F and refused to move her left arm due to the pain. On physical exam, the child appears mildly ill. She has a temperature of 102.5 F. Her pulse is 140, and her blood pressure is normal. Her exam shows a markedly swollen left elbow with a 1 x 1 cm black, crusted lesion over the olecranon process. There is erythema that extends 5 cm above the elbow to 8 cm below the elbow. The child refuses to flex or extend the joint, and when you try to passively move the elbow, she screams. A photograph of the elbow is shown. Question What is the most likely diagnosis?

Correct answer: Septic arthritis Explanation Septic arthritis is the most likely diagnosis in this particular patient. Septic arthritis can affect all ages but is most common in the pediatric age group. Septic arthritis almost always affects only 1 joint and is most likely to occur in the hip or knee. The predominant symptom is joint pain made worse by movement. Fever is often present, as is swelling, warmth, and redness of the involved joint. The diagnosis of septic arthritis requires analysis of joint fluid. Correctly diagnosing patients with septic arthritis is crucial in order to prevent the devastating complications of joint destruction and systemic sepsis. Erysipelas is a type of superficial cellulitis with lymphatic involvement, most often occurring on the face or lower extremity. The classic findings are a sharply demarcated area of erythema with a raised border. It usually remains confined to the dermis and lymphatic system; it rarely affects deeper structures. Cellulitis and bursitis may produce similar findings as the patient in the question, but maneuvers that stress the joint are not as painful as described in this case. A fracture could account for the inability to move the joint and for the swelling, but it would not explain the fever, warmth, or redness of the joint. Should a fracture be found on radiograph, this patient would have an open fracture; this would be an orthopedic emergency.

Case A 33-year-old man presents with a red finger on his right hand. He has a history of severe diabetes Type I. Further details from the patient include that he got a severe gash over the knuckle of the right pointer finger while working on an engine motor 2 days ago. He washed it out and covered it as soon as he could. Since this occurred, he has noted a significant increase in pain, redness, and severe restriction of movement in this finger; these characteristics have worsened vastly in the last 12 hours. He admits to "feeling warm" for the past day as well as having significant sweating at night, but he has not taken his temperature. He has tried to take some acetaminophen 500 mg 2 tablets every 8 hours; there has not been any noticeable relief. Pertinent physical examination findings include an oral temperature of 101.5° F, significant erythema extending distally to the DIP joint and proximally toward the dorsal surface of the right hand, severe restriction of flexion, as well as an inability to point the joint when asked. Question Based on the history and physical examination, what is the most likely diagnosis?

Correct answer: Septic arthritis Explanation The patient in the above scenario is highly likely to be suffering from a case of non-gonococcal acute bacterial arthritis, which is otherwise known as septic arthritis (SA). SA cases present with acute pain, swelling and heat of the affected joint that can be worsening with each passing hour. Chills and fever are common in around 80% of patients who present with SA. Restricted motion and loss of function are also found. In terms of the physical examination, fever is present in 1/3 - 1/2 of patients with SA; erythema, restricted movement and warmth may also be present. The patient's past medical history of having type I diabetes also increases the risk and likelihood of SA. Degenerative joint disease, also known as osteoarthritis (OA), is an incorrect response. Components seen in patients with OA include progressive onset of pain that is exacerbated by activity and relieved at rest. OA signs and symptoms are usually not found in an acute time period, but rather over months to years of progressing pain of the affected joint. Morning stiffness is common in patients with OA, although it will resolve within a 30 minute time period. Physical examination findings include limited range of motion of that affected joint and crepitus, to which the patient above has both. Patients may also have tenderness to palpation of the joint, joint effusion, or even palpable osteophytes. Psoriatic arthritis is an incorrect response. Psoriatic arthritis has distinct signs and symptoms including, but not limited to, psoriasis preceding the arthritis, 'sausage' appearance of finger and toes, symmetric involvement, as well as nail pitting. Acute gouty arthritis has an extremely sudden onset and very commonly occurs at night. Precipitating factors include excessive alcohol intake, changes in medications, or fasting episodes for surgical procedures. A specific injury does not initiate the signs and symptoms of gouty arthritis. The most commonly affected joint is the metatarsophalangeal joint of the great toe, which is sometimes referred to as 'podagra'; attacks can occur in the feet, ankles, or knees as well. The affected joints are progressively swollen and exquisitely tender; also, the overlying skin can be tense, warm, and dusky red. The patient will commonly also have a fever of 39°C or higher. Although some factors are similar to the patient in this case, he has many more similarities to the diagnosis of SA. Osteopenia is much more common diagnosis in older women that have experienced menopause; occurrence is higher in Caucasian women that possess other risk factors, such as tobacco or alcohol abuse, excess/deficiency in hormones, malignancy, and even some genetic disorders. Both disease states are typically asymptomatic until an actual fracture occurs. Loss of height may also be seen. References McPhee SJ, Papadakis MA, Rabow, MW. 2014 Current Medical Diagnosis & Treatment. 53rd ed. New York, NY: McGraw Hill Medical; 2014: 824-826, 826-827, 829-830. Septic arthritis. Dynamed website. Availabe at: http://web.b.ebscohost.com.libproxy1.usc.edu/dynamed/detail?vid=15&sid=697f3a38-56b7-468c-baf7-b6051faf0fb9%40sessionmgr111&hid=123&bdata=JnNpdGU9ZHluYW1lZC1saXZlJnNjb3BlPXNpdGU%3d#db=dme&AN=116123&anchor=top. Accessed on January 20, 2015.

Case A 66-year-old woman presents with back pain. She has a 5-month history of taking prednisone and has been a smoker for the last 15 years, which she has difficulty quitting. Upon physical exam, her back is very tender upon palpation. When the patient walks, she appears very bent over and unable to stand upright. A laboratory finding on her DXA was a T score of -3.1. X-ray was conducted as well, and demonstrates a small fracture in her back. Question What is the most likely diagnosis?

Correct answer: Severe osteoporosis Explanation The clinical picture is suggestive of severe osteoporosis. The patient has risk factors of both smoking and taking prednisone. Additionally, upon her physical exam, she appears to be tender upon palpation and unable to stand up. Severe osteoporosis is defined by a T score less than -2.5 with a fracture, which is confirmed from both her DXA and x-ray. Osteomalacia is not the correct answer, as osteomalacia is softening of the bones that presents with pain and weakness. This patient does not present with weakness. Osteopenia is not the correct answer because a T-score between -1.0 and -2.5 is diagnostic osteopenia; however, this patient has a T score below -2.5, which defines osteoporosis. Osteoporosis is not the correct answer, because in this patient, it states the patient has a fracture and a T score less than -2.5, which describes severe osteoporosis not osteoporosis. Severe osteoporosis is more associated with a fragility facture, which also makes osteoporosis incorrect. Osteosarcoma is not the correct answer, as it is a type of bone cancer that usually first presents with pain at night, which is not present in this patient.

Case A 4-year-old girl presents 30 minutes after falling from a slide (about a 4-foot fall) on her outstretched right dominant arm with her elbow fully extended. There was no loss of consciousness, but there was immediate extreme pain; there is an obvious deformity at the elbow. On presentation, the girl is holding her right elbow and she will not allow passive movement. Question Ico-delete Highlights What finding would be expected on exam?

Correct answer: Severe pain with flexion of the elbow Explanation The correct response is severe pain with flexion of the elbow. This is the classic age and mechanism of injury for a supracondylar humerus fracture. Pain is worse with flexion or extension, especially in displaced fractures (as indicated by the deformity in this child's case). This injury typically has a large effusion or edema and ecchymosis; it is often deformed (if fracture is displaced) and has crepitus. Patients will not usually allow you to passively move the elbow. The axillary nerve would not be injured in this case; it is proximal to the elbow and rarely involved.

Case You are evaluating a 66-year-old woman at a health maintenance visit. She had no complaints and no history of chronic illness except postmenopausal osteoporosis, with a bone mineral density greater than 3 standard deviations below the mean. She takes supplemental calcium and is being treated with denosumab IM every 6 months. Her only recent fracture was a compression fracture at T-6, diagnosed 5 months ago. Although she had previously enjoyed tennis, dancing, and gardening, she has drastically reduced her activity for fear of suffering further fractures. She asks if she should resume any sort of regular physical activity. Past medical history is otherwise unremarkable except for anemia, which is now resolved. Vital signs are normal, as is the remainder of the examination. Question What advice would you give?

Correct answer: She should work toward resuming her former exercise program. Explanation Inadequate physical activity contributes to the development of osteoporosis. Regular weight-bearing and muscle strengthening exercises are recommended to maintain and improve bone strength, as well as to improve agility, strength, and posture and to reduce the risk of falls. Such a regimen may also lead to some increase in bone density. Here, tennis and jogging are weight-bearing activities, to which a muscle strengthening activity such as weight training can be added. Activities of daily living alone are inadequate physical activity, which is a contributing factor in the development of osteoporosis. A history of a vertebral compression fracture is not a contraindication to increasing physical activity. Exercise is not contraindicated in osteoporosis and is important in its prevention and treatment. Both muscle strengthening exercise, such as weight training, and weight-bearing exercises, such as tennis, dancing, walking, and jogging, are recommended for a patient with postmenopausal osteoporosis. Treatment with denosumab is not a contraindication to exercise.

Case A 47-year-old grossly obese woman presents with left non-dominant shoulder pain and limited range of motion. The pain began about a month ago, and her shoulder has progressively lost range of motion since it started. She cannot reach overhead with that arm. There is no history of trauma, it does not wake her at night, and she can sleep on the left side. Her past medical history is significant for Diabetes Type I and hypothyroidism. She is on regular insulin and NPH insulin as well as Synthroid (levothyroxine). On exam, she is tender to palpation at the deltoid insertion and diffusely over the entire shoulder. She has pain with motion; it is worse at the end point of range. Her active range of motion is from 0° to 60° (abduction), 0° to 70° (forward flexion), and 0° to 20° (external rotation). Her passive range of motion is only 10° better in all directions. Question Ico-delete Highlights What should the patient expect with regards to recovery with this disorder?

Correct answer: She will need 1 to 2 years to recover full motion Explanation You should advise patients with Adhesive Capsulitis that they should expect, on average, a 1- to 2-year recovery period to regain full motion and for pain to fully subside. Although physical therapy is an adjunct to recovery, it may shorten the recovery; however, it will only shorten it to about a year. Most patients will recover fully over this time period, and they will not have to put up with it for life. Some orthopedic surgeons will perform manipulation under anesthesia, but only after a full attempt for self-recovery of at least one year. Steroids will help with the pain, but not the motion recovery.

Case A 12-year-old African-American boy presents with a 12-hour history of fever, chills, and knee pain. Plain radiograph is suspicious for osteomyelitis, and blood cultures are positive for Salmonellae. Question Ico-delete Highlights What is the patient's past medical history most likely to reveal?

Correct answer: Sickle cell anemia Explanation Salmonellae has been identified as the most common cause of hematogenous osteomyelitis in the setting of hemoglobinopathies, such as sickle cell anemia. Diabetes mellitus is also associated with hematogenous osteomyelitis, but in these patients, the causative organism is commonly S. aureus, not salmonellae. Acute lymphoblastic leukemia, Osgood-Schlatter disease, and osteogenesis imperfect do not have a specific association with osteomyelitis.

Question 1 of 250 Options Mark Note Flashcard Calc Labs Questions Feedback Help You can select text in the case or question to highlight it. Case Ico-delete Highlights Following a fall, a 6-year-old boy has restricted movement of the right hand. On examination, the right shoulder appears swollen. X-ray shows a fracture in the proximal humerus. However, at the site of fracture, there is a lucent defect extending up to the epiphyseal plate. The bone is not expanded. The patient undergoes a surgical procedure. Grossly, the lesion is actually cystic and contains clear yellow fluid. Microscopically, the cyst wall is made of fibrous tissue with occasional giant cells. Question What is the most likely diagnosis in this case?

Correct answer: Simple cyst of bone Explanation Simple cyst of the bone is seen mainly in the proximal humerus and proximal femur in children. The patients generally present with a pathological fracture. X-ray shows a lucent defect situated centrally that extends up to the epiphyseal plate. Bone involved is not expanded. Grossly, the cyst has a thin fibrous wall and it is filled with clear yellow fluid. Microscopically, the cyst wall is thin, fibrous, septa, with occasional giant cells. A peculiar calcified matrix like cementum may be present. Treatment is by aspiration of the cyst followed by injection of methylprednisolone acetate. If this fails, then surgery is advised. An aneurysmal bone cyst is a non-neoplastic locally aggressive lesion of the bone typically seen in first 2 decades of life. There is a slight female preponderance for developing them. It involves the metaphyses of the long bones. X-ray shows characteristic purely lytic lesion involving the metaphyses of a long bone with an eccentric, blow out appearance and extension into the soft tissue, which is limited by a thin shell of new bone. Grossly, it shows cavities separated by septa. The spaces contain blood or serum. Microscopically, there are cysts of various sizes separated by septa. The septa are composed of loosely arranged spindle cells and benign giant cells. The spaces are lined by cuboidal cells. The septa show capillary proliferation. Areas may show relatively solid proliferation of cells. It is treated by simple curettage but they can recur. Telangiectatic osteogenic sarcoma is a histological variant of osteogenic sarcoma. It is characterized by prominent blood-filled cysts that appear similar to aneurysmal bone cysts radiographically and pathologically. Pathological fractures are very frequent. However, the septum that separates the bloody cysts contains malignant stroma. They are associated with a more aggressive course than conventional osteogenic sarcoma. A ganglion is a small cyst that is always located near a joint capsule or tendon sheath. It is commonly seen around the wrist joint. It arises because of cystic or myxoid degeneration of connective tissue. The cysts lack a true cell lining. The lesion may be multilocular, and it enlarges by coalescence of adjacent areas of myxoid change. Synovial cysts are caused by the herniation of synovium through a joint capsule. Sometimes, it occurs due to massive enlargement of the bursa. The cyst will have a synovial lining, and it may be hyperplastic and contain inflammatory cells and fibrin. The synovial cyst that forms in the popliteal space is called as Baker's cyst. These cysts are associated with joint diseases leading to increased intra-articular pressure, such as degenerative joint disease, neuropathic arthropathy, and rheumatoid arthritis.

Question A 45-year-old female patient is seen in your office and complains of chronic dry mouth and dry eyes. She tells you that she has difficulty shedding tears and is very tired. Upon examination of her mouth, you note apparent decay in many of her teeth. This patient is likely to have which of the following diseases?

Correct answer: Sjögren's syndrome Explanation Sjögren's syndrome is an autoimmune disorder that is characterized by chronic dry mouth, dry eyes, vaginal dryness, skin dryness, and fatigue. The lack of saliva often results in rampant tooth decay. Sjögren's syndrome often goes undiagnosed, being attributed to "natural" changes related to aging and menopause. 90% of the 200,000 Americans diagnosed with Sjögren's syndrome are women. However, the disease also attacks men. There is no cure for this disease, but several treatments can alleviate some symptoms. These include the use of artificial tears and saliva to treat dryness and nonsteroidal anti-inflammatory drugs and corticosteroids to treat inflammation. Maintaining an exercise program can also help.

Case A 63-year-old Caucasian woman presents to review her recent imaging results. She has a past medical history of gastroesophageal reflux disease and mild seasonal allergies. She takes a daily omeprazole and an as-needed cetirizine. She smokes 15 cigarettes daily and has done so for the past 33 years. Family history is significant for her mother having a broken hip when she was 70 and an aunt who had 2 broken vertebrae at ages 68 and 71. The dual-energy X-ray absorptiometry (DXA) results are as follows: T-score lumbar spine: -2.4, T-score total hip: -2.5, T-score femoral neck: -2.6. Question Ico-delete Highlights Considering the patient's diagnosis, what would be the most crucial lifestyle modification she could make to improve her future results?

Correct answer: Smoking cessation Explanation Smoking cessation is the correct response. In the pathology known as osteoporosis, the ultimate issue that can occur is bones becoming weak or brittle to the point where a fall or even minor stresses such as coughing could cause fractures. This chronic disease state is especially in common in Caucasian and Asian women past menopause. Certain specific risk factors increase a patient's risk of developing osteoporosis and therefore influence how successful treatment is if/when it is initiated. These risk factors include both smoking and alcohol consumption. Other common causes include aging, high-dose corticosteroid administration, and sex hormone deficiency. Osteoporosis is often asymptomatic until a fracture has occurred. There are many non-invasive techniques that can be used to assess if osteoporosis is present and how severe the disease state currently is in the patient. These studies include dual-energy X-ray absorptiometry (DXA), single-energy X-ray absorptiometry, (SCA), quantitative CT, and ultrasound. The DXA is usually extremely accurate in diagnosing osteoporosis and has become the standard for evaluating bone density; it is what was utilized in our patient above. Patients with a T score that is -2.5 or more in the spine, total hip, or femoral neck are diagnosed as being osteoporotic. Our patient's T scores fell just around that cut-off number. Although not completely understood, the use of cigarettes over a long period of time has been found to have a detrimental effect on bone mass. It is felt that perhaps these consequences are due to toxic effects on the osteoblasts or possibly more indirectly via estrogen metabolism. Active tobacco use contributes to the progression of osteoporosis and therefore should be considered the very first modification this patient should make in regard to her diagnosis. Smoking can also have secondary effects on bone mass, including respiratory illness, frailty, decreased exercise, poor nutrition, and the need for additional medications that may affect bone mass in the long run (e.g., glucocorticoids). Although the other choices are not incorrect and are plausible answers to this question, smoking cessation is the most important action that should be taken by this patient.

Case Ico-delete Highlights A 48-year-old African American woman presents with a concern about osteoporosis. Her elderly father-in-law recently suffered a fractured hip due to severe osteoporosis. The patient has been having intermittent bilateral hip pains and is worried that she also has osteoporosis. She gets the hips pains more with prolonged standing, walking, and heavy lifting. At today's exam, she reports the pain as mild (1 out of 10, on 1-10 scale). A summary of her past medical history is shown in the table: Medications Fluticasone/salmeterol (Advair HFA) inhaler daily for asthma control; ibuprofen 400 mg as needed for arthritis pain Medical history Asthma, mild osteoarthritis OB/GYN history G2P2, has regular menses Social history Patient admits to tobacco use (1 pack of cigarettes daily). She denies use of alcohol and illicit drugs. She is divorced with 2 children at home. She walks for exercise and works in a potato processing facility, which involves frequent lifting of moderate weight. Family history Positive for breast cancer in patient's mother; hypertension in her father; asthma in 1 brother Question What aspect of this patient's history puts her most at risk for development of osteoporosis?

Correct answer: Social history Explanation The patient's social history is the most worrisome risk factor for development of osteoporosis. Tobacco use conveys a greater risk of developing osteoporosis. This patient's medication history is not worrisome. Prolonged use of oral corticosteroids is associated with decreased bone density and development of osteoporosis, but the nasal and/or inhaled steroids (such as her fluticasone) deliver a minimal systemic dose and are not generally associated with osteoporosis. The patient's age is not a risk factor for osteoporosis. Increased age (and postmenopausal status for women) is associated with increased risk of osteoporosis. The patient's current symptoms of mild hip pain are not associated with osteoporosis. Typically, patients with osteoporosis are asymptomatic, even in fairly severe cases. Ethnic and racial groups shown to be highest risk for osteoporosis are Caucasians, Asians, and Native Americans. African Americans seem to have a lower risk; research has not yet determined the reason. The patient's family history is negative for osteoporosis. The other conditions in her family history are not linked with heritable osteoporosis risk. The father-in-law with osteoporosis does not confer any genetic risk to this patient.

Question A 17-year-old male high school football player presents after being tackled and slammed onto his right dominant shoulder forcefully 2 hours ago during a game. He had immediate pain, but was able to continue punting. He has full active and passive range of motion, but some pain (4/10) with abduction. There is no obvious deformity and the skin over the shoulder is intact and not tented. What would you expect to find on a physical exam?

Correct answer: Tenderness over the AC (acromioclavicular) joint Explanation Acromioclavicular (AC) separations typically occur as a result of a direct blow to the very lateral shoulder, usually a fall onto the shoulder. This AC separation is probably a Type I. Type I will have tenderness over the AC joint, no deformity and fair motion with some pain but the patient may still be able to play football with the shoulders padded and limited contact. Pain occurs with abduction mainly. They will typically hold their arm adducted and supported. Tenderness over the greater tuberosity would be seen with rotator cuff pathology or proximal humerus fractures. Crepitus would be seen with fracture or Type III AC separations. Nerve and/or vascular injuries are very rare with this injury.

Case Ico-delete Highlights You are evaluating a 32-year-old man who presents with an 11-day history of progressive, atraumatic, left hip pain. His PMH is unremarkable with the exception of intravenous drug use. He complains of increasing pain that originally was aggravated with movement, but now he has pain at rest. He is ambulatory with a limp and exam shows no edema, crepitus, rash, or erythema. Pain is reproduced with hip ROM. Distal CMS is fully intact. Vital signs show a BP of 124/93, HR 134, RR 20, Temp 39.4, and O2 sat of 96% RA. Laboratory studies show a leukocytosis and elevated ESR. Question What finding would you expect on his hip X-ray?

Correct answer: Soft tissue swelling with periosteal elevation Explanation The answer is soft tissue swelling with periosteal elevation.1 This patient presents with osteomyelitis of the hip. Osteomyelitis is infection of the bone, with accompanying inflammation and eventual bone destruction. Risk factors for osteomyelitis include IV drug use, implanted devices and hardware, immunocompromised states, open fractures, and chronic joint destruction, such as in patients with rheumatoid arthritis. Acutely, patients may have fever, malaise, a mild leukocytosis, and elevated inflammatory markers such as ESR and CRP. Additional work up should include blood cultures, bone aspiration, and imaging studies. The initial imaging modality should be an X-ray. Early in the process, the X-ray may be completely normal or show soft tissue swelling with periosteal elevation. Late findings will include bone demineralization, lytic regions, and eventual bone sclerosis. A fracture of the greater trochanter in a 32-year-old without trauma is extremely unlikely. Severe bone demineralization is a late finding of osteomyelitis. Roughly 30%-50% of bone demineralization is necessary for this to be seen on x-ray. In a patient with an 11-day history of symptoms, it is unlikely you will find this on x-ray at this time. Tophi are deposits of urate crystals seen in patients with longstanding gout. An osteochondroma is a benign cartilage growth.

Question A newborn boy was observed to have a small cyst located midline in the lumbar region of his back. No neurological symptoms were observed, and the sac contained only meninges and cerebral spinal fluid. X-ray revealed the absence of a vertebral arch at L5. What is this type of anomaly?

Correct answer: Spina bifida with meningocele Explanation The failure of the 2 halves of the vertebral arches to fuse during development results in an anomaly referred to as spina bifida. The most minor, clinically insignificant type (spina bifida occulta) is asymptomatic and may be revealed only with imaging of the vertebra involved. Sometimes a small dimple or tuft of hair overlying the defect may be present at the level of the malformation, usually in the lumbar or sacral regions. More serious anomalies, categorized as spina bifida cystica, show a cyst-like protrusion at the level of the defect. When the cyst or sac contains meninges and cerebrospinal fluid, it is referred to as spina bifida with meningocele. If the spinal cord and/or nerve roots are included with the meninges and CSF in the sac, it is referred to as spina bifida with meningomyelocele. The most severe type of vertebral arch defect is one in which not only did the arches fail to fuse in the midline, but the neural plate failed to fold and fuse at the level of the anomaly, resulting in a flattened plate of nervous tissue, called spina bifida with myeloschisis. Spina bifida cystica show varying degrees of neurological deficits.

Question An 8-year-old boy has had worsening symptoms of bladder incontinence and increased frequency of urinary tract infections. He was born with myelomeningocele and had subsequent repair and shunting. He has been using a wheelchair much more frequently lately because his back, legs and right hip have been hurting, and he has felt a popping sensation in his hip. Based on these symptoms, what is the most likely diagnosis?

Correct answer: Spinal cord tethering Explanation Spinal cord tethering may cause progressive neurologic deficits, including decreased muscle strength or increased tone, deteriorating gait, change in sensory level, urologic dysfunction or orthopedic deformities such as hip dislocation, pes cavus or scoliosis. Although anatomical evidence of some degree of tethering is evident in almost all myelomeningocele patients, surgical untethering is reserved only for those having progressive signs and symptoms. Diagnosis is thus based on clinical grounds and not imaging studies. Natural progression of myelomeningocele is partly dependent on the level of the lesion. In children without severe developmental delays and hypotonia, 89% of preadolescents or 100% with low lumbar or sacral and 63% with higher lesions are ambulators when they receive multidisciplinary orthopedic and neurosurgical management. After adolescence ambulation decreases to 50% as it becomes more energy efficient to use a wheel chair. Orthopedic procedures are common. 85% of school age children can be free of urinary incontinence in social situations. Long term mortality can be 35-50% by adulthood. Shunt malfunction signs and symptoms include cognitive changes such as change in school performance or worsening behavior, onset or change in frequency of seizures, decrease in muscle strength or loss of previously acquired motor skill or increase in spasticity, change in ambulation, change in urinary or bowel function, change in lower cranial nerve function, pain in the back especially around the myelomeningocele repair site or worsening in scoliosis or lower extremity orthopedic deformities. Syringomyelia is a dilated spinal cord central canal filled with CSF. This is present in up to 80% of patients with myelomeningocele but only 2.5% show symptoms such as upper extremity weakness, spasticity or ascending motor loss in the lower extremities. Extension into the brain stem (syringobulbia) can cause lower cranial nerve and brainstem dysfunction. Chiari II malformation consists of the caudal displacement of the cerebellar tonsils and vermis, caudal medulla and sometimes the fourth ventricles into the cervical spinal canal. Although present in most children with myelomeningocele, it causes severe enough symptoms in 15-35% of patients to require surgical treatment. Symptoms are most common during infancy with disorders of swallowing and may include choking on foods and liquids, nasal regurgitation during drinking and frequent vomiting or significant gastroesophageal reflux. In addition, repeated aspiration pneumonia, dysarthria, apnea or cyanotic spells, inspiratory stridor and a hoarse or high-pitched cry. Older children and adults with symptoms have weakness or spasticity of the upper extremities, headache or neck pain, cerebellar dysfunction, oculomotor changes, and scoliosis.

Case A 68-year-old man presents due to lower back pain. The patient reports the pain has been present for the past 3 months and seems to be getting worse. Upon physical exam, the patient appears to get relief of pain when bending forward. Upon standing and extension of the lumbar spine, the patient reports pain. He denies decreased range of motion in the shoulders, neck, and hips. The spine is not in an S or C shape. T2 weighted imaging shows disc degeneration. X-rays show symmetrical joint spaces. C-reactive protein (< 1.0 mg/L) and ESR (<40 mm/hr) are in normal range. Gram stain is negative for Staphylococcus aureus. Question What is the most likely diagnosis?

Correct answer: Spinal stenosis Explanation Spinal stenosis is the correct answer because the patient reports a decrease in pain when leaning forward and worsened pain from standing and extension of the lumbar spine. Additionally, the T2 weighted imaging shows disc degeneration. Osteoarthritis is not the correct answer because the X-rays show symmetrical joint spaces; therefore, the diagnosis of osteoarthritis is incorrect. Polymyalgia rheumatica is not the correct answer because the C-reactive protein and ESR are in normal range and both would have been elevated if polymyalgia rheumatica were the diagnosis. Scoliosis is not the correct answer as the spine would have appeared in an S or C shape upon physical exam. Septic arthritis is not correct because patient's Gram stain is negative for Staphylococcus aureus.

Case A 52-year-old man presents to the emergency department with fever, chills, and left foot pain. His foot has been hurting for the past few weeks, but has worsened over the last few days. There is no known injury to the area, but the patient notes that the pain began after he started wearing a new pair of shoes. Past medical history includes hypertension and type 2 diabetes, but he has not followed up with his physician recently. Upon examination, there is an ulcer on his heel with surrounding erythema and edema. X-ray reveals erosion and periosteal changes at the site of the ulcer. Question Which organism is the most common cause of this infection?

Correct answer: Staphylococcus aureus Explanation Staphylococcus aureus is correct. The patient has osteomyelitis. Staphylococcus auerus is the most common cause of osteomyelitis of the foot in diabetic patients. Escherichia coli, Pseudomonas aeruginosa, Serratia marcescens, and Salmonella enterica are incorrect. While all these organisms may cause osteomyelitis, Staphylococcus aureus is the most common cause.

Case A 49-year-old Caucasian man is well known to your practice; he presents with pain in his left lower extremity. Upon further questioning, the patient describes doing construction work 3 months prior when he jumped onto his feet from a height of about 5 ft. Since this episode, he has noted issues of increasing left sided hip and knee pain. He describes the pain as radiating into the left groin and front middle thigh area. The pain is relieved with sitting, and it is aggravated by walking and climbing up stairs. The patient denies any paresthesias, numbness, bowel/bladder dysfunction, fever, night sweats, or chills. Radiographic interpretation includes the presence of a crescent sign as well as marked irregularity of the left femoral head with sclerosis. Question Ico-delete Highlights Given these findings, what would you expect to see on a radiographic image taken of the left hip?

Correct answer: Subchondral collapse Explanation The correct response is subchondral collapse. Avascular necrosis, sometimes also referred to as osteonecrosis, is osseous cell death resulting from vascular compromise. Common sites that are affected include the proximal or distal femoral head or even the ankle, shoulder, or elbow. Common causes include but are not limited to corticosteroid use, alcoholism, trauma, systemic lupus erythematosus, pancreatitis, gout, or even sickle cell disease. Radiographic findings will include mild density changes early on, sclerosis and a more progressed disease state will lead to the pathognomonic crescent sign. When repair begins at the interface between necrotic and viable bone, this produces a sclerotic margin. Eventual mechanical failure of trabecular bone at this interface results in progressive microfracture and collapse of the adjacent dead subchondral trabeculae and this leads to the subchondral radiolucent area along the fracture line, which is the crescent sign.Later disease will show joint-space narrowing and degenerative changes in the acetabulum. Lytic lesions are most commonly seen in patients with multiple myeloma. A fractured femoral head may be seen as the result of a high-impact trauma involving significant force, falls in older patients, or various illnesses or disease states that significantly affect bone integrity, such as vitamin D deficiency, systemic lupus erythematosus, or cancer. Both of these potential answers are not as likely. A bone cyst is described as a fluid-filled hole that develops inside a bone. These most commonly occur in children and young adults and very rarely cause significant symptoms. The cause of bone cysts is unclear; usually, they will not cause serious health issues. This does not match what is described in the original question. A bone tumor is an area of abnormal growth of cells within the bone and may either be malignant or benign. Causes of bone tumors are generally unknown but are seen commonly in patients with a history of genetic abnormalities, radiation, or injury. More often than not, no specific cause is found; again, this is most likely not the correct answer.

Case A 22-year-old woman presents for evaluation of knee pain. The patient appears in moderate distress and is unable to fully bear weight as she moves to the exam table. Her physical exam reveals a swollen, tender left knee. She is tender at the lateral femoral condyle, lateral tibial plateau, and tibiofemoral joint line. Lachman and anterior drawer tests are positive. Posterior drawer, Apley grind, patellar grind, and McMurray tests are negative. There are no sag or apprehension signs. Valgus and varus stress tests are normal. An X-ray is obtained and shows no fractures. Question What inciting event is most likely in this patient's history?

Correct answer: Sudden landing after jumping during basketball Explanation This patient's physical exam findings demonstrate a deficiency in the anterior knee, specifically the anterior cruciate ligament (ACL). ACL injuries most often occur acutely while playing sports, such as basketball, skiing, and tennis; however, they can occur with other mechanisms as well. Female gender is a risk factor. The patient may give a history of hearing a "pop", along with acute pain as well as buckling or instability of the knee. Of the choices listed, sudden landing after jumping during basketball is the most likely to produce an ACL injury. A dashboard injury to the knee during a motor vehicle accident could produce several injuries, but it would be most likely to produce a posterior dislocation and disrupt the posterior cruciate ligament (PCL). A PCL injury would likely yield physical exam findings of a positive posterior drawer and sag sign (in which the anterior tibia sags posteriorly). Fever and prior Neisseria gonorrhoeae infection with knee pain suggests septic arthritis as the cause. Septic arthritis would produce swelling and tenderness, but it would not be expected to demonstrate instability, such as this patient's positive anterior drawer and Lachman tests. Frequent squatting and climbing stairs can contribute to knee pain, and would be suggestive of a patellofemoral syndrome or possibly osteoarthritis (although the latter is unlikely in a 22-year-old patient). If this patient's knee pain had been caused by squatting and stair-climbing, her physical exam might have demonstrated a positive apprehension test and patellar grind test. A lateral blow to the knee is associated with disruption of the medial and/or lateral collateral ligaments. Here, we would expect the physical exam to have a positive valgus and/or varus stress test. A lateral knee injury would not typically cause ACL injury.

Case A 4-year-old girl presents after a fall from a 4-foot slide 30 minutes ago. She fell with her right dominant arm outstretched and her elbow fully extended. There was no loss of consciousness, but there was extreme pain. She immediately grabbed her right forearm and her right elbow. There is an obvious deformity at the elbow. Question What is the most likely diagnosis?

Correct answer: Supracondylar humerus fracture Explanation Supracondylar fractures of the distal humerus are the most common elbow fractures in children between the ages of 2 and 12. The typical mechanism of this type of fracture is a fall on an outstretched arm, usually from a height. Proximal humerus and distal radius fractures will not show deformity at the elbow unless also associated with injury at the elbow. Fracture of the lateral epicondyle of the humerus accounts for 1/5th of elbow fractures in children. Fracture of the medial condyle of the humerus is also uncommon.

Case A 23-year-old woman presents with a painful right arm. Tissue pressure in the arm is 55 mmHg. There is also swelling and tenderness to the touch. Patient has extreme pain when the fingers of the right hand are flexed. There is no erythema present and no purulent exudate is seen. Question What is the treatment for this patient's symptoms?

Correct answer: Surgery Explanation Surgery is the correct response. Compartment syndrome is compression of the blood vessels and nerves that can occur in the extremities. As a result, nerve and muscle damage can result due to decreased blood flow to the affected area. The muscles of the extremities are separated into compartments by fascia. The fascia cannot expand, and when pressure increases inside of the compartment, the enclosed tissues can become compressed. The pressure can increase to the extent that the blood flow becomes blocked. If the pressure is not released, this can lead to permanent damage. Necrosis of the tissue can occur and will require amputation of the limb. Compartment syndrome can result from trauma (including long bone fractures), burns, surgery, and hemorrhage from a large blood vessel. It can also be the result of casts or bandages that have been placed too tightly. It is most common in the forearms and lower legs; however, any part of an extremity may be affected. A chronic form of compartment syndrome can be seen in runners. Symptoms include pain, decreased sensation, pallor of the skin, and weakness. For example, the patient may feel pain in the calf when the ankle is flexed. The pain may not be relieved with pain medication or when the limb is elevated. This type of pain is a hallmark of compartment syndrome. Diagnosis is usually made through examination. The patient will experience pain when the affected muscle is moved. The skin over the affected area may appear tight and shiny. There may also be swelling. Measuring the pressure in the compartment will confirm the diagnosis. If the pressure within the compartment is greater than 45 mm Hg, the diagnosis is confirmed. Treatment is usually surgery. An incision is made through the fascia in order to release the pressure. The incision is left open and closed later. If the pressure is relieved quickly, the patient's prognosis is good. If the pressure is not relieved, nerve damage can result in approximately 12 - 24 hours. Permanent nerve damage is a complication that can result if the condition is left untreated.

Case A 28-year-old woman presents with what she describes as a "lump on her hand". She first noticed this lump on her right lateral wrist about 6 months ago. She states initially it was about an inch in diameter and not painful. Over the last 2 months, the lump has now grown to about 3.5 inches in diameter and is extremely painful when the patient writes, types, or anything that involves the right wrist. She has taken ibuprofen 200 mg PO every 8 hours as needed, with only mild relief. Physical examination reveals somewhat limited range of motion of the right wrist due to the pain and size of the mass, as well as a translucent appearance when shining a penlight through it. Question Considering the most likely diagnosis at this time, what treatment regimen is most appropriate?

Correct answer: Surgical excision Explanation The patient above is most likely experiencing signs and symptoms of a ganglion, also known as a ganglion cyst. These are the most common mass or lump found in the hand. These are benign and in majority of patients are completely harmless and asymptomatic. These fluid filled cysts have a tendency to appear quickly, disappear quickly, and even change size. Treatment is not indicated for asymptomatic ganglion cysts, but in the event that one is painful, interferes with the patient's function, or has an unacceptable appearance, treatments may be available. In the scenario of our patient above the most reasonable treatment at this time is surgical excision. This is due to the tremendous signs and symptoms the patient is currently experiencing and the very high possibility of recurrence with other nonsurgical means of treatment. Observation would be a reasonable option if the mass were completely asymptomatic and the patient wasn't bothered by the appearance of it. Radiation and chemotherapy are not appropriate answers due to the fact that a ganglion cyst is not a cancerous mass. Aspiration may be a nonsurgical option for treatment, but aspiration on its own is frequently unsuccessful in preventing the return of the ganglion cyst. This is likely due to the fact that the connection to the joint or tendon sheath, where the cyst originates, is not removed. Other nonsurgical options that may be tried include the use of splints or anti-inflammatory medications. References

Case A 23-year-old woman presents with a rash and swollen joints. She had been healthy previously, and the only medication she takes is acetaminophen. A review of systems includes recent fevers and a 5 lb weight loss in the past month; she is also experiencing photosensitivity and hematuria. She denies oral ulcers, nasal congestion/discharge, ear pain, pleuritic symptoms, chest pain, neural symptoms, bruising, and bleeding. On physical exam, her temperature is 101°F; her blood pressure is 130/85 mm Hg. She has a malar rash as well as diffusely swollen, warm, and tender joints. Her cardiopulmonary exam is normal. She has no costovertebral angle tenderness. Trace bipedal edema is noted. Diagnostic studies reveal: Erythrocyte sedimentation rate 50 mm/hour Hemoglobin 11 g/dL Anti-double-stranded DNA antibodies Positive Antihistone antibody Positive Platelet count 200,000 cells/mL White blood cell count 12,000 cells/mm3 Chest X-ray Normal Urinalysis No bacterial growth. Numerous dysmorphic red blood cells. 10 white blood cells/high power field. Red blood cell casts. 510 mg albumin/g creatinine. Complement 3, 4 Both reduced Anti-Neutrophil Cytoplasmic Antibody Negative Question What is the most likely diagnosis?

Correct answer: Systemic lupus erythematosus Explanation This woman most likely has systemic lupus erythematosus (SLE), given her symptoms of hematuria, photosensitivity, and arthritis, in addition to the findings of urinary red cell casts, proteinuria, and hypocomplementemia. In a large meta-analysis looking at 200,000 patients with SLE, the most common clinical features included arthritis in 72%,hematological abnormalities in 62%,malar rash in 50%,photosensitivity in 48%, renal involvement in 38%, oral ulcers in 34%, serositis in 30% and neurological involvement in 14%. (11) Renal involvement is frequent in SLE, and it may include a variety of glomerular diseases. Hypocomplementemia may occur due to complement consumption in the glomeruli. Focal proliferative and diffusely proliferative glomerulonephritis are two forms of glomerulonephritis that might be seen in this patient if biopsied. If proven, she might be treated with immunosuppressive therapy, such as prednisone and cyclophosphamide. Patients with SLE nephritis may recover, relapse, progress to a different form of lupus nephritis, or progress to end-stage renal disease. Drug-induced lupus does not typically cause renal or neural involvement. A variety of drugs cause a lupus-like picture with positive anti-histone antibodies, but no anti-double-stranded DNA antibodies are seen; normocomplementemia is noted in this condition. Wegener's granulomatosis is a small vessel vasculitis of unknown pathogenesis that causes inflammation and necrotizing granulomas in the upper and lower respiratory tract and kidneys as well as a rash. Palpable reddish-purple lesions on the lower extremities and positive anti-neutrophil cytoplasmic antibody (ANCA) should raise suspicion of vasculitis. Renal involvement is very common; necrotizing glomerulonephritis with negative immune staining may be seen on renal biopsy. Microscopic polyangiitis/polyarteritis is a small vessel vasculitis and is a differential cause of rash, arthralgias, and renal failure. Arthritis (true joint swelling noted on physical exam) is not usually noted. Patients may be ANCA-positive. Henoch-Schönlein purpura (HSP) is a vasculitic syndrome seen more often in children than adults; it may cause petechiae, purpura, gastrointestinal bleeding, and hematuria/glomerulonephritis. Complement counts and anti-double-stranded DNA are normal, and elevated IgA antibody levels may be seen. In a classification scheme revised by the Systemic Lupus International Classification Clinic (SLICC) in 2012, patients who have "4 SLICC criteria including at least one immunological criterion (e.g., elevated ANA) and at least one clinical criteria (e.g.,malar rash) or biopsy-proven nephritis compatible with systemic lupus in the present of either ANA or anti-dsDNA" can be classified as having systemic lupus erythematosus.

Case A 21-year-old man presents with a 3-month history of pain in his left shoulder. He is a left-handed pitcher for his college baseball team and states that the pain began at spring training and has gradually worsened since that time. In addition to the pain, the patient is now having difficulty lifting his left arm above his head. He denies any injury to the shoulder itself and states that ibuprofen provides some relief. Question What physical exam finding in this patient would be most indicative of rotator-cuff tendinitis?

Correct answer: Tenderness to palpation just under the acromion Explanation The correct response is tenderness to palpation just under the acromion. The muscles of the rotator cuff include the supraspinatus, infraspinatus, teres minor, and subscapularis, otherwise known as the SITS muscles. Three of these muscles - the supraspinatus, infraspinatus, and teres minor - insert on the greater tuberosity of the humerus. Repetitive movements of the shoulder, as with throwing or swimming, can lead to inflammation of the tendons of the rotator cuff muscles; the most commonly affected muscle is the supraspinatus, which inserts directly under the acromion. Classically, a patient with rotator cuff tendinitis will have tenderness to palpation just under the acromion on the affected side as well inability to lift the arm above shoulder level (abduction and external rotation). The "crossover test" is used to assess the acromioclavicular (AC) joint of the shoulder. Inflammation or arthritis of the AC joint usually presents with localized tenderness or pain with adduction of the affected shoulder. A "positive crossover test" is when adduction of the patient's affected arm across the chest elicits pain. Bicipital tendinitis, inflammation of the long head of the biceps tendon, can also present with anterior shoulder pain that may resemble rotator cuff tendinitis and may coexist with it. However, in bicipital tendinitis, tenderness is usually maximal in the bicipital groove along the anterior aspect of the humerus. The Speed's test is used to evaluate for biceps tendon injury. The patient keeps the elbow extended and arm supinated. In this position, the patient elevates the arm against resistance. Pain with elevation is indicative of a inflammation or tear in the biceps tendon. Palpable crepitus during range of motion maneuvers is suggestive of osteoarthritis of the joint, not tendinitis, though both can co-exist.

Case A 32-year-old man presents with pain in his right shoulder. He tells you he plays softball every weekend and does a lot of hoeing and shoveling around his house. The shoulder has been sore for some time, but now it hurts to the point where he tries to avoid using it. The drop arm test is negative, and there is no redness, warmth, or obvious swelling, but the patient is unable to lift his arm up to 90 degrees without pain. Question What is the most likely diagnosis?

Correct answer: Tendinitis Explanation Tendinitis is an inflammation of the tendon tissue, or the tendon sheath (tenosynovitis). The cause is often unknown. As the vascularity of tendons decreases with age, the incident of tendinitis will increase. Symptoms usually include painful tendons on movement. Swelling can be seen or felt on palpation. Along the tendon, localized tenderness of variable severity is present on palpation. Generally, there is not the warmth or redness of bursitis. During the drop arm test, the client is asked to fully abduct the arm (straight out laterally from the side), then slowly lower the arm to his side. If he is unable to hold his arm against gravity, the drop arm test is positive. The drop arm test detects whether or not there are any tears in the rotator cuff. If there are tears in the rotator cuff, or supraspinatus tendon, the arm will drop to the side from a position of 90 degrees. The client will not be able to lower it smoothly in rotator cuff tear and will not be able to raise it to 90 degrees in tendinitis. Bursitis is an inflammation of the bursa. A bursa is a closed sac containing a small amount of synovial fluid (usually < 1 cm). True bursitis has fluid that can be aspirated. Tendinitis is often mislabeled as bursitis. Bursitis around the knee is more common in those who kneel often (like maids and clergymen). To distinguish between tendinitis and bursitis, note that bursitis will often not only feel painful on use, but have swelling, redness, and warmth. Analysis of bursal fluid can indicate the reason for inflammation. Fibromyalgia is a soft tissue, non-articular pain disorder characterized by chronic, generalized musculoskeletal aches, pains, and stiffness that occur primarily in muscles and their attachments. It is associated with specific sites of exaggerated tenderness. Gout occurs most commonly in men (95%). Presentation is usually acute, with a swollen, red big toe. Women may develop gouty arthritis in the perimenopausal phase as estrogen levels drop. In many cases, if the disorder is left untreated, urate crystals are deposited, usually in and around the joints of the extremities. The needle-shaped monosodium urate (MSU) crystals can be deposited in the parenchyma of organs, such as the kidney. In some clients, deposits of uric acid form within the urinary collecting tubules.

Case A 32-year-old woman presents with a 3-month history of dominant right hand pain as well as paresthesia of the thumb, index, and middle fingers. She works in a retail sales office; she spends much of the day typing on a keyboard. Question What associated physical finding may indicate an advanced version of her most likely diagnosis?

Correct answer: Thenar atrophy Explanation The correct response is thenar atrophy. Compression of the median nerve in carpal tunnel syndrome (CTS) results in paresthesias, pain, and sometimes paralysis in the median nerve distribution pattern (palmar surface of the thumb, index, middle, and radial side of the ring fingers). When the nerve is severely compressed for a long period of time, it can also result in the motor component of the nerve being compromised. The thenar muscles, if not supplied with adequate nerve motor impulse, will begin to atrophy. Hypothenar atrophy would be an indication of ulnar nerve compression seen in entrapment syndromes either at the elbow or wrist. Weakness of digital extension is an indication of radial nerve problems, which is more likely to be seen with humeral shaft fractures (although it is not very common). The 2-point discrimination test distance widens with CTS to over 15 mm of spread. The radial artery lies outside of the carpal tunnel and therefore is unaffected in CTS.

Case A 51-year-old woman with a 1-year history of back pain and weakness of the lower extremities presents for evaluation. Neurological examination shows decreased sensation below T12 dermatome level. She denies any bowel or bladder changes. An MRI reveals a 2 x 0.5 cm cystic intradural extramedullary mass at the level of T12. The lesion shows intermediate signal intensity on T1-weighted sagittal and axial images after intravenous contrast injection. Question What fact is true concerning the majority of the patient's type of tumors?

Correct answer: They mostly develop at the lower portion of the spinal cord Explanation Ependymoma is the most likely diagnosis. The majority of tumors are found in the lower portions of the spinal cord. Ependymomas make up approximately 60% of spinal cord tumors. They are relatively slow-growing, and they may develop anywhere along the entire length of the spinal cord; however, a large percentage of these tumors are found at the lower sections of the cord. They develop from ependymal cells and can be seen in all age groups; the most common age group is between 20 and 40 years. Men and women are affected equally. Tumors may metastasize here from other parts of the body, or they may be made of the same types of cells. Spinal tumors may be intradural extramedullary, intramedullary, or extradural. Astrocytomas are the second most common type of spinal cord tumor. They are found more often in children than in adults. Other types include developmental tumors, oligodendrogliomas, lipomas, and hemangioblastomas. The usual presentation is pain; classically, it is aggravated in supine or flat position. As the tumor advances, abnormal sensation and eventually weakness follow, until finally normal bladder and bowel function are lost. Surgery is the preferred treatment of choice. Radiation therapy is sometimes given if total surgical removal of the tumor is not possible, or if the tumor appears to grow rapidly

Question What is seen during the early stages of osteoarthritis?

Correct answer: Thickening of articular cartilage Explanation Most descriptions of osteoarthritis emphasize the loss of articular cartilage that occurs as a result of this disease. During early stages of this disease, however, the cartilage is thicker than normal. There is an increase in water content of the joint as well as an overall increase in proteoglycan synthesis and hypertrophic repair of articular cartilage.

Case A 42-year-old woman works full-time as a data entry clerk and often puts in many hours of overtime. She is experiencing numbness and tingling in her right thumb, index finger, middle finger, and half of her ring finger. The numbness and tingling initially comes and goes, but it is constantly present after a few months. Question On physical examination, what sign should be elicited?

Correct answer: Tinel's sign Explanation This patient has symptoms consistent with carpal tunnel syndrome. In carpal tunnel syndrome, the median nerve is compressed in the wrist. The distribution of her symptoms indicates distribution of the median nerve. Tinel's sign is produced by lightly tapping on the anterior surface of the wrist over the median nerve. A positive Tinel's sign is said to be present if the tapping produces tingling. Tinel's sign can be seen with carpal tunnel syndrome. Trendelenburg's sign is a dropping of the buttock when the leg on the same side is raised. Trousseau's sign is positive if occluding the brachial artery produces carpal spasm. Cullen's sign is ecchymosis around the umbilicus, which can sometimes be seen with acute pancreatitis. Grey Turner's sign is ecchymosis of the flanks, which can sometimes be seen with acute pancreatitis. References DeGowin RL, LeBlond RF, Brown DD. D

Case An obese 45-year-old man presents with right hip pain that extends to the groin; the pain started after helping a neighbor move the week prior to presentation. Any attempts at physical activity cause symptoms to worsen. The patient's medical history is significant for diabetes, depression, and alcohol abuse. After physical examination and diagnostic testing, he is diagnosed with a hip strain. Question Due to concern over the possibility of the patient developing avascular necrosis of the hip, what recommendation should you make to the patient?

Correct answer: To treat alcohol abuse Explanation Alcohol abuse is a risk factor for developing avascular necrosis of the hip. Treating the alcohol abuse decreases the risk. Avascular necrosis is death of the bone in the femoral head. Physical therapy will not prevent this from occurring in a patient with a risk factor (e.g., alcohol abuse). Treating the hip strain will help the patient now, but treatment will not decrease the patient's risk of getting avascular necrosis. Although weight loss is beneficial for patient's overall health, it would not reduce the risk of developing avascular necrosis. Since the problem is the patient's current alcohol abuse putting him at an increased risk for avascular necrosis, starting NSAIDs would not be preventative.

Case A 35-year-old man had orthopedic knee surgery 3 days ago. You have ordered meperidine hydrochloride (Demerol) every 4 hours. You have had to increase the dosage several times because the patient complains of pain every 3 hours. Question What does this likely represent?

Correct answer: Tolerance to meperidine Explanation Tolerance to a drug is defined as decreasing response to repeated constant doses of the medication or the need for increasing doses to maintain a constant response. Toxic effect occurs when a drug is overdosed. In the case of a narcotic, it could result in respiratory depression. Since the patient is still in pain and pain is one of the strongest respiratory stimulants (you breathe faster when you are in pain), this is very unlikely in this case. Addiction is the state of being given up to some habit, especially strong dependence on a drug. There is little chance of developing an addiction when a narcotic is administered for a short time following an operation. Dependence is a state in which there is a compulsion to take a drug, either continuously or periodically, in order to experience its psychic effects or to avoid the discomfort of its absence. The latter can result in physical symptoms. The patient is not showing any symptoms of hypersensitive reaction, which could be urticaria, rhinitis, bronchospasm, dermatitis, and/or angioedema.

Case A 49-year-old Caucasian man presents with pain in his left lower extremity. During questioning, the patient states that while doing construction work 3 months prior, he jumped from an elevated height of 6 feet; ever since this episode, he has noted increased issues with left-sided hip and knee pain. He describes the pain as radiating into the left groin and front middle thigh area. The pain is relieved with sitting and aggravated by walking and climbing up stairs. The patient denies any paresthesias, numbness, bowel or bladder dysfunction, fever, night sweats, or chills. Pertinent medical history includes a 20-year extensive history of alcohol. A radiograph interpretation shows the presence of a crescent sign, as well as marked irregularity of the left femoral head with sclerosis. Question Considering the most likely diagnosis, what would be the ultimate clinical intervention necessary for this patient?

Correct answer: Total hip replacement Explanation Avascular necrosis, sometimes also referred to as osteonecrosis, is osseous cell death resulting from vascular compromise. Common sites that are affected include the proximal or distal femoral head or even the ankle, shoulder, or elbow. Causes include, but are not limited to corticosteroid use, alcoholism, trauma, systemic lupus erythematosus, pancreatitis, gout, or even sickle cell disease. Radiographic findings will include mild density changes early on and sclerosis, while a more progressed disease state will lead to the pathognomonic crescent sign, which is subchondral collapse. Later disease will show joint-space narrowing and degenerative changes in the acetabulum. The age of this patient would make the treating healthcare provider highly pursue conservative efforts to help preserve the affected joint. Unfortunately, with progressive process of this disease, most likely the patient in the above scenario will need to have a total hip replacement. The initial treatment plan usually includes avoidance of weight bearing of the affected joint for several weeks. Vascularized or nonvascularized bone grafting is sometimes used to help preserve the femoral head but only delays the ultimate clinical intervention of a total hip arthroplasty. Daily oral corticosteroids and bisphosphonates are both incorrect and in fact on occasion may lead to the development of osteonecrosis.

Case A 34-year-old woman presents to your clinic with a 3-month history of a mildly tender mass on the dorsocentral aspect of her dominant right wrist. She says the lump "comes and goes," but this time it has stayed and become tender. She denies any known trauma of the wrist or hand. Question What examination technique would help to show that this mass is a ganglion cyst?

Correct answer: Transilluminating the mass Explanation A prominent ganglion cyst of the dorsal wrist will transilluminate, while a solid tumor will not. The active range of motion is usually not affected with a ganglion cyst except for tenderness that may limit the motion. Although ganglions are usually smooth on the surface, tumors can be too. Radiographs are taken to rule out any bony pathology, but they won't reveal the ganglion. MRI is used for locating the mass and its origin. Most ganglions and tumors are tender to firm palpation or pressure.

Case A 21-year-old man presents with a 3-month history of pain in his left shoulder. The pain began when he started spring training and has gradually worsened. In addition, the patient is now having difficulty lifting the arm above his head. He denies injury to the shoulder itself; he states that ibuprofen provides some relief. On physical exam, the patient has tenderness to palpation of the lateral left shoulder just under the acromion, limited abduction of the left shoulder, and a negative "drop arm" sign and "crossover test". Question What is the next appropriate step in the management of this patient?

Correct answer: Trial prescription of non-steroidal anti-inflammatory medications (NSAIDs) Explanation The correct response is a trial prescription of non-steroidal anti-inflammatory medications (NSAIDs). The patient above is most likely suffering from rotator cuff tendinitis. Repetitive movements of the shoulder, such as throwing or swimming, can lead to inflammation of the tendons of the rotator cuff muscles. A patient with a rotator cuff tendinitis, in addition to an inability to lift the arm above shoulder level (abduction and external rotation), will have tenderness with palpation of the area just under the acromion on the affected side. Though the "drop arm" sign is used to assess the status of the rotator cuff, a positive drop arm sign indicates an actual complete tear in the rotator cuff, not just tendinitis. The patient in the presentation above had a "negative drop arm" test. For most rotator cuff disorders, rest from activities that increase pain is helpful. In addition, NSAIDs and moist heat can afford some relief, and they are usually the first-line treatment for these conditions. Corticosteroid injections and physical therapy can also be used in patients who do not get full relief from NSAIDs, or in those who have recurring issues. Opiate or narcotic analgesics are usually never indicated in the management of these inflammatory conditions.

Case A 20-year-old man presents after taking a fall during a skiing trip 2 days ago. The chief complaint is pain in the right thumb. On examination, you note gross laxity in the first metacarpophalangeal joint when moving the right thumb into abduction. An X-ray reveals no fracture. Question What structure is injured?

Correct answer: Ulnar collateral ligament of the thumb Explanation The clinical picture is of a gamekeeper's thumb. With this injury, the ulnar collateral ligament is ruptured, which causes laxity when moving the thumb into abduction. In cases of radial collateral ligament damage, there would be no laxity when moving the thumb into abduction. The flexor and extensor tendons are not damaged in a gamekeeper's thumb. Median nerve damage (carpal tunnel syndrome) would present with aching, numbness, or tingling in the thumb, some of the fingers, and sometimes part of the hand.

Case A 24-year-old man presents due to increasing pain in his right groin and buttock and difficulty walking. The pain has been present and worsening for about 1 month. He further reports that he had quite significantly injured himself also near his right hip in a fall while snowboarding about 3 months ago. He did not seek care immediately and had difficulty ambulating for a week, then saw a chiropractor to "put his lower back back in." The pain and gait problems eventually improved. Now he is concerned because the pain has returned with no new history of trauma. He is otherwise healthy and takes no regular medicines. He denies any chronic health conditions and has an unremarkable family history for musculoskeletal and rheumatologic conditions. On physical exam, the patient was noted to walk into the exam room with a slight limp. Examination of the hip, buttock, and groin region reveals no edema, erythema, or ecchymosis. The exam does not produce one particular point of tenderness with palpation, but pain is elicited with right hip motions, particularly internal rotation. Diagnostic testing revealed a "crescent sign" and confirmed the suspected condition. The patient will be scheduled for definitive treatment within a week. Question What approach is best to prevent further complications and maintain his current health until definitive treatment occurs?

Correct answer: Utilize crutches for any ambulation. Explanation This patient's condition is suspicious for avascular necrosis (or the more preferred term, osteonecrosis) of the hip. Avascular necrosis (AVN) is a relatively common complication following a traumatic hip dislocation when the vascular supply to the femoral head is damaged and bone death occurs. There are many theories about the exact mechanism; other causes, such as vascular, congenital, and autoimmune diseases, have been implicated with osteonecrosis. If the "crescent sign" is seen on plain radiography (X-ray), AVN is confirmed. Definitive treatment is surgical, with a variety of approaches, from core decompression to bone grafting. Until surgery occurs, symptomatic patients should utilize crutches for any ambulation to prevent collapse of the femoral head. The patient should not begin aggressive physical therapy prior to surgical repair. As noted above, bearing weight on the joint may predispose the patient to bone collapse. There is also no reason to initiate antibiotics prior to surgical treatment. AVN is not caused by infection, in contrast to osteomyelitis. Likewise, the provider should not initiate bisphosphonate therapy, even though the long-term goal is for the patient to regrow new bone. In fact, bisphosphonate therapy has been linked with causing osteonecrosis. There are some studies proposing benefit with using bisphosphonates, but not enough definitive data yet to recommend their use. The provider should not initiate high-dose corticosteroids, either. As with bisphosphonates, corticosteroids have been linked with causation of osteonecrosis. The primary mechanism of AVN does not appear to be inflammatory, so the steroids would not benefit the patient's healing.

Case A 17-year-old male college freshman presents with chronic fatigue, back pain, and stiffness. The pain has been present for the past several months, but it appears to be worsening. The back symptoms are worst at night and first thing in the morning; they improve somewhat during the day. He is having difficulty staying productive at school because he is always tired. Question MRI of the lumbosacral spine reveals bilateral inflammatory changes of the sacroiliac joints and fusion of the L3-L5 vertebrae. What extra-articular condition is associated with this disease?

Correct answer: Uveitis Explanation The correct response is uveitis. The patient is presenting with signs and symptoms indicative of ankylosing spondylitis (AS). Characteristic symptoms include back pain that is worse in the morning and improves with exercise. Back stiffness and extreme fatigue are also reported. AS is more common in male patients. As with other inflammatory arthritides, patients usually present at a younger age (18-35 years) relative to patients with mechanical causes of back pain. Unlike mechanical back pain, the pain associated with inflammatory spondylitis usually improves with movement. AS is a clinical diagnosis, but many patients will have elevated c-reactive protein and erythrocyte sedimentation rates. Diagnostic imaging classically reveals inflammatory and sclerotic changes affecting the sacroiliac joints and spine. Vertebral fusion is also seen as a result of progressive joint erosion. Extra-articular conditions associated with AS include iritis and uveitis (inflammations of the iris and uvea, respectively) as well as inflammatory bowel disease. Heart disease is also more common in patients with inflammatory arthritides. Diabetes mellitus and peptic ulcer disease are relatively more common in patients with rheumatoid arthritis. Psoriatic skin lesions are seen in patients with psoriatic arthritis. Scleroderma is an autoimmune disease that does not coexist with increased frequency in any of the inflammatory arthritides.

ase A 16-year-old boy is currently an inpatient 7 days after a motor vehicle accident that resulted in multiple lower extremity fractures. The fractures were surgically corrected recently. He is currently immobilized below the waist, but casts will not be placed until the majority of his edema has resolved. He is stable, the edema is resolving, and he is recovering well, with the exception of new onset of increased edema and erythema over his left thigh. X-rays show evidence of acute osteomyelitis. The patient does not have any allergies, and a culture from the infected bone has revealed methicillin-resistant Staphylococcus aureus. Question What treatment regimen is the best choice to treat his infection following surgical debridement?

Correct answer: Vancomycin (Vancocin)1 g IV q 12 hours for 6 weeks Explanation The correct answer is Vancomycin (Vancocin) 1g IV q 12 hours for 6 weeks for several reasons. Methicillin-resistant Staphylococcus aureus (MRSA) has to be treated with an antibiotic that MRSA is sensitive to, such as Vancomycin. Vancomycin is a glycopeptide antibiotic typically used to treat Gram-positive bacteria. The other reason that this answer is correct is due to the length of time noted in the treatment. Patients with acute osteomyelitis need at least 4-6 weeks of antibiotic therapy. Studies have also been done on animals that show optimal treatment length to be 4-6 weeks in duration. Nafcillin (Unipen) 2g IV q 4-6 hours for 6 weeks is not the correct answer. Nafcillin is a narrow spectrum beta-lactamase penicillin that is used to treat Gram-positive bacteria, including staphylococci, that are resistant to other penicillins. If the patient's culture had shown methicillin-sensitive Staphylococcus aureus (MSSA), then Nafcillin would have been an appropriate antibiotic choice. However, Nafcillin is not effective against MRSA. 6 weeks would have been an appropriate length of time if the culture had shown MSSA. Oxacillin (Bactocill) 2 g IV q 4-6 hours for 2 weeks is not the correct answer. Oxacillin is also a narrow spectrum beta-lactamase penicillin that is used to treat Gram-positive bacteria, including staphylococci, that are resistant to other penicillins. If the culture had shown MSSA, then Oxacillin would have been an appropriate antibiotic choice. In addition, the answer is not correct; the length of treatment (2 weeks) would not be long enough. Ciprofloxacin (Cipro) 400mg IV q 12 hours for 4 weeks is not the correct answer. Ciprofloxacin is a fluoroquinolone antibiotic used to treat a wide variety of infections. However, it is typically used to treat Gram-negative organisms rather than the Gram-positive organism in this question. It is possible for osteomyelitis to be caused by Gram-negative bacilli, and this would be a good treatment option in that case. Linezolid (Zyvox) 600mg IV q 12 hours for 2 weeks is not the correct answer. Linezolid is in the oxazolidinone class of medications and is typically used to treat infections that have resistance to other antibiotics, such as vancomycin-resistant enterococci (VRE) or MRSA. Therefore, Linezolid would be a good antibiotic choice in this case. However, the length of treatment (2 weeks) would not be long enough to effectively treat the infection.

Case A 50-year-old obese woman presents with severe left knee pain. She states the pain began about 8 months ago but has grown significantly worse in the last 3 months. The patient denies any trauma or event that initiated the pain. She notes stiffness in the knee first thing in the morning that lasts around 5-10 minutes. The knee pain is worsened with activity and is relieved with rest. The patient's medication list includes lisinopril 10 mg once daily for high blood pressure. She has a documented medication allergy to acetaminophen; she states this makes her break out in hives. Physical examination findings reveal a Caucasian female with a BMI of 40. There is limited range of motion of the left knee and severe crepitus. Question Considering the most likely diagnosis for this patient, what lifestyle modification would be most beneficial in relieving her symptoms?

Correct answer: Weight loss Explanation This patient is most likely suffering from degenerative joint disease, also known as osteoarthritis (OA). OA is the most common form of knee arthritis. Common characteristics of OA include onset after 40 (usually in patients over 55) along with obesity; typically, these patients are genetically predisposed. Other components seen in patients with OA initially have an insidious onset of pain that will rapidly progress. Characteristics of this pain will be soon recognized as being exacerbated by activity and relieved at rest. Morning stiffness is common in patients with OA, although it will resolve within a 30 minute time period. Common symptoms patients with OA experience include buckling or giving way of that affected joint due to the bony areas impinging upon each other. Physical examination findings include limited range of motion of that affected joint and crepitus; the patient above has both. Patients may also have tenderness to palpation of the joint, joint effusion, or even palpable osteophytes. Changes that occur in the articular cartilage due to this pathology are irreversible. Conservative treatment is what should be initiated first. This should include activity modification, therapeutic exercises, weight loss, proper footwear, and avoidance of high impact activities. Use of a cane may also be mechanically advantageous for the patient, especially if it is used in the hand opposite the affected side. Knee braces or sleeves may also be utilized and provide minor subjective pain relief. Initial medication intervention of choice should be oral acetaminophen, topical capsaicin, or a traditional NSAID. It is well documented that obesity puts extra strain on the knee joints. Patients should be advised to lose 10% of their body weight in order to improve their health. It can be difficult to lose even that amount of weight. Healthcare providers should stress to patients that even minor weight loss could help people suffering from signs and symptoms due to osteoarthritis. Some studies even provide weak evidence that overweight people with knee osteoarthritis can improve their mobility and relieve pain even if they lose just 5% of their body weight within a month period. Most patients find it difficult to lose that much weight just by dieting and are more successful if they combining dieting with exercise therapy. Perseverance and motivation must be stressed. Bed rest would be inappropriate in this clinical case scenario. Screening labs is incorrect as well. Blood pressure control would be beneficial to this patient overall, but it would not directly improve osteoarthritis. The recommended therapeutic exercises for osteoarthritic patients often include muscle strengthening, flexibility, and resistance workouts.

Case A 12-year-old boy presents with a 3-month history of intermittent, moderate right knee pain and mild right hip pain that has been increasing in frequency and severity over the past week. There is no history of trauma or associated symptoms. Recently, he has begun to experience stiffness in the right hip and has developed a limp. Examination reveals normal vital signs, with height at 64" and weight at 182 lbs (>95th percentile). He walks with an antalgic gait with the right foot externally rotated. There is pain on passive range of motion of the right hip, but not the right knee. Internal rotation of the right hip is limited. External rotation of the right hip with the knee flexed produces external rotation of the right lower leg. Radiographs of the right knee are normal, but radiographs of the right hip reveal a moderate increase of the angle of the femoral head to shaft. Question What statement concerning this condition is true?

Correct answer: Without surgical intervention, there is an increased risk of avascular necrosis of the femoral head Explanation Slipped capital femoral epiphysis (SCFE) most usually appears in overweight male teenagers and presents with hip and/or knee pain, hip stiffness, altered gait, and decreased range of motion. Unless surgically corrected, thereby relieving the compromise of the blood vessels to the capital femoral epiphysis, there is an increased risk of avascular necrosis of the femoral head. This is not a systemic condition that would affect other joints. It is common in this age group for hip disease to present with referred pain to the knee. A complaint of knee pain at this age warrants a thorough evaluation of the ipsilateral hip. The angular femoral deformity will not remodel spontaneously. Surgery should be performed imminently to relieve compromise of the vasculature of the capital femoral epiphysis.

Question A forearm fracture just distal to the elbow damages the radial nerve. What will the patient experience?

Correct answer: Wrist drop from the loss of wrist extension Explanation The radial nerve innervates the wrist extensors, so damage to the nerve will not allow wrist extension, which leads to wrist drop. Pronator and opposition muscles, as well as sensory aspects of the ventral forearm, are all mediated by the median nerve. Finger abduction is controlled by the ulnar nerve.

Case A 17-year-old football player (punter) was tackled and forcefully slammed onto his right dominant shoulder 2 hours ago during a game. He had immediate pain, but it has subsided; he now experiences the same level of pain when he carries a heavy object. He wants to continue punting. He has full active and passive range of motion, but some pain (4/10) with abduction. There is no obvious deformity, and the skin over the shoulder is intact and not tented. There is no crepitus on palpation while moving the arm. Most of his tenderness seems to be on the superolateral right shoulder. Question What recommendation(s) should be made to this patient?

Correct answer: You can continue if the pain allows it, but further injury might make it worse Explanation Patients with a Type 1 Acromioclavicular separation may resume playing as pain allows, but typically that takes 4 weeks. The patient will run a risk of further injury if he continues to play through the pain and convert a Type 1 into a Type 2 or worse. The patient playing because the coach or teammates really need him should not be advised. This injury, although considered minor, can be made worse if the player were to be hit or land on his shoulder again. Never inject cortisone into an AC joint with a ligament injury. This only weakens the ligament further, and it can increase the chance of converting a Type 1 into a Type 2 or worse. Since this is most likely a Type 1 injury, surgery is not necessary.

Case Ico-delete Highlights A 49-year-old, right-hand dominant woman presents with a 2-week history of progressive pain in her right thumb and wrist area. She says that her thumb seems to "stick" in place upon movement. She states that she has never experienced this before. You ask her if she has been using her hands more often than normal, and she tells you she recently began to crochet a sweater for a family member. On physical examination, her pain is markedly exacerbated when she places her thumb into the palm of her hand, and you passively move her hand in an ulnar direction. Question What is the most likely diagnosis?

Correct answer: de Quervain's tenosynovitis Explanation The clinical picture is suggestive of de Quervain's tenosynovitis. It occurs commonly in middle-aged women, precipitated by repetitive use of the thumb. The inflammation thickens the tendon sheath abductor pollicis longus and extensor brevis on the thumb side of the wrist. This can cause pain, swelling, and may cause the thumb to lock in place upon movement. The Finkelstein test (full flexion of thumb into palm followed by ulnar deviation of the wrist) will produce pain and is diagnostic of de Quervain's tenosynovitis. Wrist arthritis causes pain on movement with evidence on radiographs, which is not seen in this patient. A scaphoid fracture would have tenderness in the anatomical snuffbox, which is not described in this patient. Dorsal wrist ganglion presents with a palpable mass. Thumb arthritis presents with swelling over the carpometacarpal joint and pain on joint compression.

Case A 39-year-old woman presents after getting hurt at her karate class the night before. She had been doing kick-jumps and landed on the outside of her left plantarflexed and inverted foot with all of her weight. There was a popping sound, and sharp pain set in immediately. The instructor had ice put on the ankle and elevated it right away. Before leaving the facility, a compression wrap was applied. Treatment was sought about 14 hours later because of pain and swelling. On examination, swelling in the area in front of the lateral ankle is visible. The anterior drawer test shows a difference of 5 mm between injured and non-injured ankle. The talar tilt test reveals no instability, and weight-bearing ability is only slightly restricted. Question What ligament is most likely affected? Refer to the image.

Explanation Considering the mechanism of injury, the most likely affected ligament is the anterior talofibular ligament. This is usually the result of landing on a plantarflexed and inverted foot when jumping, stepping in a hole, or walking on rough ground. The anterior talofibular ligament is the weakest ligament within the lateral ankle complex. It tears first, followed by the calcaneofibular and posterior talofibular ligaments. The anterior drawer test (image 2) measures the anterior translation of the talus and determines the integrity of the anterior talofibular ligament. Since translation can be up to 9 mm, comparison to the non-injured ankle is very important. If the foot is dorsiflexed and inverted when injured, the calcaneofibular ligament is hurt. In this case, the talar tilt test (image 3) shows instability. External rotation force on a dorsiflexed foot results in injury of the syndesmosis. The delta ligament tears in eversion and external rotation injuries. Anterior drawer test to measure the integrity of the anterior talofibular ligament is performed by steadying the lower leg with one hand while the other hand grasps the heel and pulls the foot forward. Physiologically, the movement is between 2 and 9 mm. The test is considered positive when the difference between the uninjured and injured ankles is more than 4 mm. Talar tilt test to measure the integrity of the calcaneofibular ligament is performed by steadying the lower leg with one hand while the other hand grasps the heel and inverts the foot. Physiologically, the movement ranges from 5° to 23°, and it is also essential to compare injured and non-injured ankle. The test is considered positive when the difference is more than 6°.

ase A 47-year-old man presents with a history of worsening burning left heel pain for the last few weeks. He states that it is worse in the morning after getting out of bed and beginning to walk; it improves with exercise. He denies any foot numbness, tingling, or known injury. He states that he recently began a jogging regimen. Question What is the most likely diagnosis?

hide Correct answer: Plantar fasciitis Explanation The correct answer is plantar fasciitis, as it occurs as a result of excess stress on the Achilles tendon, which attaches to the plantar fascia. Classic presentation is burning heel pain that is worse when bearing weight after prolonged periods of rest (e.g., sleep). Heel contusion is a soft tissue injury resulting from a trauma. Heel contusions cause pain, but the pain is unlikely to be as severe as what is described in this case. Calcaneal fracture typically occurs after an injury, and the pain would likely be more consistent with weight bearing. Tarsal tunnel syndrome is due to entrapment of the posterior tibial nerve, and it presents with medial malleolus pain that may radiate. Pain is typically worse during sleep, walking, and dorsiflexion. Paresthesias are common. Achilles tendon rupture is often described as the feeling of being kicked or shot in the area of the Achilles tendon; subsequently, jogging and walking normally would not be possible, as the tendon is crucial for this movement.

Case A 40-year-old male carpenter presents with a 9-month history of worsening bilateral knee pain. He feels stiff in the morning when he first wakes up, but this subsides quickly. By the end of the workday, he has significant pain in his knees and has to go home and elevate his legs for some relief. This seems to be worse in colder weather. There is tenderness over the medial aspect of both knees, with the right knee being more tender and edematous. Varus deformity is noted. An arthrocentesis shows the synovial fluid white blood cell count to be 1200/mcL. On plain radiographs, subchondral cysts are present. Question What is the most likely diagnosis?

orrect answer: Osteoarthritis Explanation Osteoarthritis is correct because this often presents in a pauciarticular manner in middle-aged people. There can be morning stiffness, but it lasts less than 30 minutes. Osteoarthritis is worse with activity and is relieved by rest. On examination, there can be tenderness, crepitus, and joint effusion. Synovial fluid has less than <2000/mcL white blood cells. Plain radiographs can show subchondral cysts, osteophytes, subchondral sclerosis, and joint space narrowing. Rheumatoid Arthritis is incorrect because morning stiffness lasts at least 1 hour; there is symmetric joint swelling; and the hands are most commonly affected in early disease. Bilateral Meniscus Tears is incorrect because there was no acute injury. These tears commonly occur in twisting injuries. Onset of pain usually occurs within 1 day from injury. Infectious Arthritis is incorrect because this commonly occurs in only 1 joint. Furthermore, swelling, warmth, and redness would be noted on physical examination. Pseudogout is incorrect because the synovial fluid would show phagocytosed crystals within polymorphonuclear leukocytes. The total leukocyte concentration is 15,000-30,000/mm3, with a high concentration of neutrophils. The knee is commonly affected by pseudogout.

Case An 18-year-old woman presents to the emergency department with her mother due to pain in her right leg after a car accident. She points to her lower leg and describes the pain as severe. Upon physical exam, there appears to be pressure and pain in the lower right leg. Additionally, when asking the patient to stand on the leg, she reports weakness and extreme tenderness. Upon use of the Stryker IC pressure monitor system, the patient's pressure was 35 mm Hg. Question What is the most likely diagnosis?

orrect answer: Acute compartment syndrome Explanation The clinical picture is suggestive of acute compartment syndrome because the patient complained of pain and pressure. Additionally, the patient has a pressure of 35 mm Hg on the Stryker IC pressure monitor, which helps confirm compartment syndrome and indicates a need for intervention. Carpal tunnel syndrome is not correct because the patient does not describe pain in her hand. Carpal tunnel syndrome is compression of the median nerve, which is not located in the leg. Chronic exertional compartment syndrome is not correct because this condition is caused when a patient has muscle pain with exercise. A muscle in the body is surrounded by fascia, which ends up building up too much pressure in a muscle compartment, causing pain. The symptoms in the case result from more acute and severe pressure in the leg. Rhabdomyolysis is not correct because rhabdomyolysis is a breakdown of skeletal muscle cells rather than an increase of compartment pressure. Stress fracture is not correct because the patient has a pressure of 35 mm Hg, which is higher than what would present in a patient with just a stress fracture. Additionally, a patient would report point tenderness, not overall pain and pressure. References

Case A 25-year-old sexually active woman presents with a 2-day history of pain and swelling of her dorsal right wrist and fingers. For the past week, this pain and swelling has been in different joints of her body, including her left knee, left elbow, and right ankle. On exam, you note edema, effusion, and erythema over the dorsal right wrist with the wrist held in 15 degrees of extension. It is very tender on palpation and has virtually no range of motion secondary to the pain. You aspirate synovial fluid for lab studies. The lab studies reveal the following information: Synovial fluid: WBCs - 57,000 cells/mcL; Gram stain: too numerous to count WBCs with no bacteria seen; Culture: pending. Question Based on this history, physical, and lab findings thus far, what should be the initial treatment for this woman?

orrect answer: Admit to the hospital for IV ceftriaxone (Rocephin) Explanation Gonococcal septic arthritis, which this woman most likely has based on age, history, and physical and lab findings, should be admitted for IV ceftriaxone (Rocephin) until culture results return and have sensitivities determined. Oral treatment, especially with doxycycline, is ineffective in treating gonococcal septic arthritis. One should not wait for culture results to start treatment. Surgical arthroscopic irrigation may be needed if after 24-48 hours there is no improvement on the IV antibiotic treatment. 4 to 5% of gonococcal isolates produce a β-lactamase that confers penicillin resistance; therefore, it is not first-line treatment.

Case A 28-year-old man presents first thing Monday morning due to a left ankle sprain. The patient describes walking in the woods the day before while wearing flip flops and inadvertently stepping into a hidden hole. The patient immediately noticed extreme pain with weight bearing on the left ankle, making it extremely difficult for him to walk out of the woods. Ambulation is still exceptionally difficult at this time. He describes it as an 8-9/10 on a 1-10 pain scale. The patient admits to having ankle sprains before, but this one is much more severe and debilitating. He has also noted significant swelling and exquisite tenderness to the touch. He has treated his ankle with elevation, ice for 20 minutes at a time, and ibuprofen 200 mg every 8 hours. Physical examination reveals a moderate degree of ecchymosis of the left ankle with substantial mechanical instability and moderate restriction of range of motion. Question Based on the history and physical examination findings, what would be the most appropriate clinical intervention at this time?

orrect answer: Bracing of the ankle Explanation The scenario above is describing the Grade II ankle sprain. This injury typically involves an incomplete tear of a ligament. Patients will experience moderate pain, swelling, tenderness, and ecchymosis. There will be mild to moderate joint instability during exam and some restriction of the range of motion, as well as loss of function. Ambulation and weight bearing are painful. Grade I ankle sprain results from only mild stretching of a ligament with potentially microscopic tears. Patients will only present with mild swelling and tenderness; there will not be evidence of joint instability, and the patient will be able to bear weight or ambulate fairly easily. Grade III ankle sprain involves a complete tear of a ligament. There will be severe pain, swelling, tenderness, and ecchymosis. Significant instability will be seen on exam, as well as loss of function, inability to bear weight, or ambulate whatsoever. Immediate intervention of an ankle sprain should follow the MICE mnemonic: modified activities, ice, compression, and elevation, which our patient has been somewhat following. Subsequent treatment commonly involves protected weight bearing with crutches and the use of an ankle stabilizer brace. This is considered especially in those patient suspected of having either a Grade II or III ankle injuries. Early motion is essential, so the response of no weight bearing is incorrect. Patients are often encouraged to perform appropriate home and physical therapy exercises. A cast for this injury would also not be an appropriate choice. It may be considered for a high ankle sprain, which is not the case in the above scenario. Observation only is also not the correct answer.

Case A 64-year-old woman presents with malaise and a severe unilateral headache, as well as pain and stiffness in her neck, shoulders, and back. Her appetite is poor, and she has recently lost weight. She has an oral temperature of 100.5° F, her hemoglobin is 11.8 g/dL, and the sedimentation rate is 104 mm/hr. A superficial temporal artery biopsy confirms the suspected diagnosis. Question Which of the following is considered first-line treatment for this suspected disorder?

orrect answer: Corticosteroids Explanation The correct response is corticosteroids. Giant cell (cranial/temporal) arteritis (GCA) is associated with polymyalgia in 25% of individuals and is a disease commonly seen in the elderly population. Common in the elderly, it is rare under 55 years. The symptoms include headache, scalp, temporal artery tenderness (e.g., when combing hair), jaw claudication, amaurosis, and fugax. A decrease or loss of temporal artery pulsations may occur in patients with cranial or temporal arteritis. Although polymyalgia rheumatica often responds to anti-inflammatory agents or low-dose steroids, temporal arteritis should be treated with high-doses of corticosteroids in an attempt to prevent complications such as blindness, which is the immediate risk for this patient. The diagnosis is confirmed by temporal artery biopsy, which shows characteristic granulomatous inflammation. Other vessels are also involved in this disease. Untreated patients may develop sudden unilateral or bilateral blindness. Indomethacin is a NSAID and is used in inflammatory conditions such as osteoarthritis and bursitis and is a first-line medication for the treatment of acute gout. Gold thiomalate is used to treat rheumatoid arthritis in certain cases. Penicillamine is used to treat primary biliary cirrhosis, lead toxicity, and Wilson disease

ase The mother of a 3-year-old boy states that her child does not walk normally. The mother feels that the child has been falling more frequently; she also tells you that his symptoms are worsening quickly. On exam, the child uses his hands to push himself into an upright position from sitting on the ground. He walks with a waddling gait. There is hypertrophy of the bilateral calves, lumbar lordosis, and hyporeflexia. There is also cognitive delay. His labs reveal an increased creatine kinase. Question What is the most likely diagnosis?

orrect answer: Duchenne muscular dystrophy Explanation Duchenne muscular dystrophy symptoms are usually noted at age 3-5 in boys. The disease is present at birth, but symptoms often are not noted until the child starts walking. A waddling gait, hypertrophy of the calves, lumbar lordosis, and hyporeflexia are all common exam findings with this disease. The way the child pushed himself into an upright position is known as Gower's sign. There is also an increase in creatine kinase. This is the most common form of muscular dystrophy in children. Facioscapulohumeral muscular dystrophy usually starts later in the teenage years. Weakness is usually noted in the face, arms, and around the shoulders. Facial weakness is usually the initial sign of this disease. Myotonic muscular dystrophy typically involves many other systems in addition to the musculoskeletal system. It is the most common adult form of muscular dystrophy. People with this disorder have long faces and drooping eyelids. Neck muscles and distal limb muscles are the first muscles to be involved. Typical onset of Becker muscular dystrophy symptoms is later than Duchenne, normally occurring after age 15. It also has a milder course than Duchenne muscular dystrophy. It resembles Duchenne, but it is far less common. Hypertrophy of the calves would also be a finding with this disease.

Case An 82-year-old man with rheumatoid arthritis, HIV, and diabetes mellitus has developed fever, severe pain, and swelling to his right knee. He denies any history of trauma, injuries or accidents, sexual activity, and denies prior episodes. A joint aspiration was done which revealed the following finding. Question What would be the most likely additional manifestation in this patient?

orrect answer: Limited range of joint motion Explanation This patient's presentation is most consistent with a diagnosis of septic arthritis. The joint aspiration of a septic arthritis typically reveals opaque to turbid synovial fluid with bacteria, a leukocytosis, and high lactate levels. Clinically, patients present with severe pain, effusion, and resistance to joint motion. Patients with an infected joint typically present with the triad of fever (40-60% of cases), pain (75% of cases), and impaired range of motion. The findings of an overlying cellulitis, such as warmth and redness of the surrounding skin, may also be present. Unless an osteomyelitis in proximity is also present, both plain radiographs and MRI scanning will reveal only a joint effusion. Arthrocentesis demonstrates an exudative effusion, with a leukocyte count greater than 50 × 109 cells/L. Bacterial cultures are frequently positive, with Staphylococcus species the most frequent causative organisms. These symptoms may evolve over a few days to a few weeks. Fever is usually low-grade (< 102°F), with rigors present in only 20% of cases. Spiking fevers and chills are much more common with crystalline arthritis. The pattern of joint involvement is an extremely important diagnostic feature. Of cases of nongonococcal suppurative arthritis, 85-90% are monoarticular. If the disease affects more than one joint, S aureus is most commonly implicated. Polyarticular arthritis is usually observed in gonococcal disease, various viral infections, Lyme disease, reactive arthritis, and various noninfectious processes. A positive McMurray's sign is suggestive of a meniscal tear. Numbness and tingling are manifestations consistent with neurologic pathology.

Case A 23-year-old man presents with a 2-week history of fever and back pain. The pain radiates from the midline to one side. The patient also has a 1-week history of right lower limb weakness and urinary incontinence. The patient gives history of trauma to the back following a motor accident 1 month prior to presentation. Examination by touch and percussion demonstrates localized pain in the spine; you also note neurological losses on the right side of the body that include decreased sensation and weakness. The patient's temperature is 101°F, and his skin is moist. Question What investigative procedure has the greatest diagnostic accuracy in diagnosing the patient's condition?

orrect answer: Magnetic resonance imaging (MRI) Explanation The patient's physical exam is worrisome for the diagnosis of epidural abscess. Gadolinium-enhanced magnetic resonance imaging (MRI) of the spine is the most accurate test to confirm the diagnosis of epidural abscess. It is useful in distinguishing epidural abscess from the adjacent compressed thecal sac and other potential compressive lesions, such as a herniated disc. Epidural abscess is a disorder that is characterized by inflammation and a collection of infected material (pus) between the dura (the outer membrane covering the brain and spinal cord), and the bones of the skull or spine. The infection is usually bacterial (staphylococcus is common) but may be fungal or viral. The condition is very rare. Pus forms as a collection of fluid, destroyed tissue cells, white blood cells, and live and dead microorganisms. The infection spreads through the bones and membranes of the spine. The risks include boils (furuncles) on the skin, particularly on the back or scalp. Associated predisposing conditions include a compromised immune system, such as in patients with diabetes mellitus, AIDS, chronic renal failure, alcoholism, cancer, or following epidural anesthesia, spinal surgery, or trauma. Abscess may also result from the spread of any infection through the bloodstream from another body location. Symptoms include fever, chills, lower backache (mild, progressive, and typically radiating to an extremity), sudden or rapid progression of neurologic symptoms, loss of movement of an area of the body, weakness or paralysis, loss of sensation of an area of the body, numbness in a specific, limited area (localized), loss of bladder or bowel control, and male impotence. An examination often shows localized tenderness over the spine. A neurologic examination may indicate spinal cord compression, with involvement of the lower body (paraplegia), or the entire trunk, arms, and legs (quadriplegia). The extent of neurologic losses corresponds with the location of the lesion. A CSF examination may be normal or may contain a few white blood cells and slightly elevated protein levels. Myelography, which was commonly used previously to diagnose spinal epidural abscess, is no longer recommended because of the risk of introducing infection. Lumbar puncture also entails the risk of spreading bacteria into the subarachnoid space with consequent meningitis; therefore, it should not be performed. CT scans are performed only when MRI cannot be performed. Plain radiographs occasionally demonstrate osteomyelitis, but they are of almost no utility. The goals of treatment are the relief of spinal cord compression and the eradication (cure) of the infection. The therapeutic method of choice is laminectomy for surgical decompression, combined with antibiotics. Conservative treatment alone is justifiable only for specific indications. Broad-spectrum antimicrobials or a combination of antibiotics are the most commonly used antimicrobials. Corticosteroids may occasionally be prescribed to reduce swelling and compression of the spinal cord. References

Case Ico-delete Highlights A 45-year-old man presents with a 2-day history of left-sided chest pain. He describes it as an aching pain that is aggravated by movement, and he denies any history of trauma to the chest. On examination, there is tenderness on palpation over his left chest wall. Question What is the most likely cause of his chest pain?

orrect answer: Musculoskeletal pain Explanation Musculoskeletal chest pain, which can be unilateral or bilateral, is usually described as an aching pain. It is aggravated by movement. On examination, tenderness may be elicited on application of pressure over the affected chest wall. Herpes zoster pain is usually described as sharp or burning. On examination, there is usually a unilateral vesicular rash with a dermatomal distribution. In some cases, the pain may precede the lesions. Angina pain is usually described as tightness or pressure on the chest and lasts for less than 10 minutes. It is usually left-sided or retrosternal and may radiate to the jaw, neck, and shoulder. It is precipitated by physical exertion or emotional stress. It is relieved by rest and sublingual nitroglycerin. Spontaneous pneumothorax pain is of sudden onset and patients present within a few hours of its onset. It is usually pleuritic in nature. On examination, the patients may be dyspneic with decreased breath sounds on the affected side. Pericarditis pain is usually described as a sharp retrosternal pain that is often worse when the patient is supine. It is relieved by sitting upright, and it is aggravated by deep inspiration and changes in position. It may be episodic and can last for hours or days. On examination, a pericardial friction rub may be auscultated.

Case A 45-year-old man presents back to his orthopedic office for a 1-month history of worsening pain in his left groin and buttock. He had a surgical hip fracture repair approximately 4 months ago, 2 days after a fall in a bowling alley. His post-operative course was unremarkable, with normal healing and return to activities. However, he now notes increasing difficulty with walking and has had to limit activities again due to pain. He has tried over-the-counter (OTC) pain medications without relief. His past medical history is unremarkable, aside from the surgical fracture repair. He has no chronic medical conditions, no other surgeries, and takes no chronic medications. He has no medication allergies. On physical exam, the patient is asked to walk across the room. He limps and appears to be in pain. The entire hip region is evaluated, and no erythema, ecchymosis, or edema is noted. The incision site is healing well, with no tenderness and no fluctuant tissue. With range of motion tests, the patient reports pain, especially with internal rotation of the left hip. He is sent for several tests. Blood culture is negative. X-ray shows a crescent sign over the left femoral head. An MRI shows a low-signal intensity on the left femoral head, and increased uptake is seen also in this region with nuclear bone scan. The fracture site shows normal post-operative changes. Question What medication would be most helpful as an adjunctive treatment for this patient's complication from his hip fracture?

orrect answer: Oral ibuprofen Explanation This patient's condition is suspicious for avascular necrosis (or the more preferred term, osteonecrosis) of the hip. Avascular necrosis (AVN) is a relatively common complication, following a traumatic hip dislocation and/or fracture, when the vascular supply to the femoral head is damaged and bone death occurs. Some potential conditions on the differential may include arthritis, malignancy, and osteomyelitis. The history and physical exam may be similar in all these conditions, but the history of hip fracture and the diagnostic studies support a diagnosis of AVN. Definitive treatment is surgical, with a variety of approaches, from core decompression to bone grafting. The role of medical treatment is primarily analgesia, so, of the choices provided, oral ibuprofen would be a reasonable adjunct. If the patient reports that he tried OTC ibuprofen, it would be important to elicit the dose and duration of treatment. Many patients have not correctly utilized OTC analgesics. Additionally, if the patient prefers a medication that cannot be obtained OTC, an alternative non-steroidal anti-inflammatory drug, or another class of analgesic, could be prescribed. Intravenous vancomycin would be useful if this patient had osteomyelitis, which would have been more likely if the blood cultures were positive. The MRI findings may be similar with AVN, but osteomyelitis does not exhibit the crescent sign on x-ray, either. A joint injection of corticosteroid will not fix the underlying problem and may exacerbate the surgical repair. Combination chemotherapy (along with surgical resection) may be a treatment approach for osteosarcoma. It is unlikely this patient has a malignancy. The imaging does not show characteristic findings of a bone cancer. Oral methotrexate may be used in a variety of conditions. If this patient's joint pain was secondary to an autoimmune arthritis, methotrexate may be useful. However, his presentation refutes autoimmune arthritis as the cause of his pain.

Case A 36-year-old woman presents with a tibial fracture following a motor vehicle accident. She is admitted to the hospital. Overnight, she develops increased pain despite maximal doses of IV narcotics. Question What is the most reliable early physical exam finding for her condition?

orrect answer: Pain out of proportion to exam findings Explanation The patient is suffering from compartment syndrome. Compartment syndrome occurs when pressure increases inside of a muscle compartment. It typically develops a few hours after soft tissue injury. Pain is typically out of proportion to exam findings and is unresponsive to opioid analgesia. The symptoms of compartment syndrome have classically been described by the 5 Ps: pain, pulselessness, paresthesia, pallor, and poikilothermia, but not all patients will develop this constellation of symptoms. Absent peripheral pulses may be present in a patient with compartment syndrome, but arterial circulation may remain intact until late in the presentation of compartment syndrome; a patient with intact peripheral pulses may still be suffering from compartment syndrome. Pallor of the involved limb may be present in a patient with compartment syndrome, but it is a late finding that does not develop until arterial circulation is affected. Paresthesia of the involved limb is another late finding of compartment syndrome. It develops when nerve ischemia is present. Fever is not a classic symptom of compartment syndrome.

Question Membranous glomerulonephritis is associated with which of the following conditions?

orrect answer: Systemic lupus erythematosus Explanation The causes of membranous glomerulonephritis are enumerated as follows: (Renal vein thrombosis is frequent.) * Chronic infections include malaria, hepatitis B * Systemic diseases like SLE * Heavy metals like Hg, Gold intoxications * Drugs like penicillamine, captopril * Solid tumor like melanoma, lung cancer, etc. The causes of focal segmental glomerulosclerosis are enumerated as follows: (Renal vein thrombosis is infrequent.) * Vesicoureteral reflux * HIV infected patients * Obesity * Heroin addict * Solitary or ablated kidney

Case A 32-year-old woman presents with a 3-month history of right hand pain and paresthesia of the thumb, index, and middle fingers. She works in a retail sales office and spends much of the day typing. Question What associated physical finding may indicate an advanced case of this suspected diagnosis?

orrect answer: Thenar atrophy Explanation When compression of the median nerve is severe and long-standing, atrophy of the thenar muscles—which are supplied by the motor branch of the median nerve—can occur, indicating possible denervation of the muscles. Hypothenar atrophy would indicate ulnar nerve long-standing compression, usually because of a space-occupying lesion, such as ganglion, lipoma, ulnar artery aneurysm, or muscle anomaly. Weakness on extension of the DIP joints would indicate injury or entrapment of the radial nerve. Widening of the gap (NOT decrease of the gap) in 2-point discrimination testing would be expected. The radial artery is not within the carpal tunnel and would unlikely be decreased unless there was injury to the wrist.

Case A 24-year-old man presents with radial-sided wrist pain following a fall on his outstretched hand 3 days ago. He did not seek immediate medical attention due to a lack of swelling. Despite self-treatment with ice and analgesics his wrist still hurts, especially with ulnar deviation and while trying to open a jar or grip the steering wheel of his car. On examination, his tenderness seems to be in the anatomic snuffbox of the wrist. Radiographs are normal in all views. Question Ico-delete Highlights What is the proper treatment for this patient at this time?

orrect answer: Thumb spica splint and referral to an orthopedic surgeon Explanation If a patient with a suspected scaphoid fracture has radiographs that are read as "normal'", but clinically there is a fracture, place the patient in a long arm thumb spica splint and refer the patient to an orthopedic surgeon. Elastic bandage and volar splints are not adequate immobilization for a possible scaphoid fracture. Immediate referral is not warranted because a positive diagnosis has not been identified. Routine referral is not warranted for the same reason; however, if after a positive diagnosis there is also evidence of non-union or a worsening fracture gap, a referral to an orthopedic surgeon is definitely warranted.

Case You are evaluating a 16-year old Caucasian male high school junior (his baseball team's starting shortstop) for left hamstring soreness of 4 days duration. He felt a slight pull during infield practice. He was late that day and skipped his normal warm-up and stretching routine. Despite rest, the soreness has persisted. He would like some medication for the discomfort but noted that several oral analgesics and anti-inflammatories had caused gastric upset in the past. Past medical history is otherwise unremarkable, growth and development are normal, and immunizations are current. Vital signs are normal. Examination reveals mild tenderness to palpation of the left hamstrings, aggravated by extension at the knee. Question Which of the following is the best choice of drug therapy?

orrect answer: Topical diclofenac gel Explanation Pharmacological management of minor sports injuries, such as strains and sprains, with topical NSAIDs provides a viable alternative to oral medication. Topical NSAIDs are generally effective in relieving the pain of minor sprains and strains when applied to the affected area and are well-tolerated, even by those who have experienced unacceptable side effects to oral NSAIDs, such as gastrointestinal upset. Topical NSAIDs have anti-inflammatory, antipyretic, and analgesic qualities. Side effects are generally limited to local site reactions, such as erythema and pruritus. Intramuscular corticosteroids are not indicated for a minor soft tissue strain. An opiate is not an acceptable choice for treatment of a minor soft tissue strain. Oral corticosteroids are not indicated for a minor soft tissue strain. While oral NSAIDs are often used to treat minor sports injuries, here, the history of gastric upset with oral anti-inflammatories makes topical NSAIDs the better choice.


Kaugnay na mga set ng pag-aaral

Life Insurance Policies-Provisions, options and Riders Chapter 3

View Set

Communication in Healthcare: Mindfulness: Mindfulness & Reflection, Implicit Bias

View Set

Module 11: Organizing for Long-Term Success

View Set

MKG 310 Test 3, MKG 310 Summary Questions

View Set

Style is how you ___ what you ____

View Set

Chapter 3 - Business in the Global Economy

View Set

USAFA History 300 GR2 Terms (Fall 2018)

View Set